You are on page 1of 200

Refresher Manual for the

SAT
*

Version 7.1

00 Refresher SAT FM & Intro 1 11/15/05 3:28:25 PM


Copyright © 2006 by The Princeton Review, Inc. All Rights Reserved.

No part of this manual may be reproduced for distribution to a third party in any
form or by any means, electronic or mechanical, including photocopying, record-
ing, or any information retrieval system, without the prior consent of the publisher,
The Princeton Review.

Permission to reprint this material does not constitute review or endorsement by


the Educational Testing Service, of this publication as a whole or of any other sam-
ple questions or testing information it may contain.

This Manual is for the exclusive use of Princeton Review course students, and is
not legal for resale.

Educational Testing Service and ETS are registered trademarks of the Educational
Testing Service. SAT is a registered trademark of the College Board.

The Princeton Review is not affiliated with Princeton University or the Education-
al Testing Service.

866.TPR.PREP/ www.PrincetonReview.com

00 Refresher SAT FM & Intro 2 11/15/05 3:28:25 PM


Acknowledgments

These people rock:

Joan Afton, Jennifer Arias, Siddiq Bello, Fred Bernstein, Carol Brenneisen,
Lisa Buchman, Morgan Chase, Rob Cohen, Mariwyn Curtin, Jon Dohlin, Ken Dow,
Dan Edmonds, Alicia Ernst, Louise Favier, Michael Freedman, Len Galla, Jodie
Gaudet, Jay Glick, Andrea Goldfein, Andra Gordon, Effie Hadjiioannou, Peter
Hanink, Clayton Harding, Scott Karp, John Katzman, Meher Khambata, Jane
Lacher, Illeny Maaza, Tom Meltzer, Nikki Moss, Jefferson Nichols, John Pak, Chee
Pae, Isabel Parlett, Magda Pecsenye, Maria Quinlan, Valli Rajah, Carmine Raspaolo,
Joe Reddy, Jennifer Robbins, Jeff Rubenstein, Joe Sampson, Nick Schaffzin, Jon
Spaihts, Joshua Shaub, Graham Sultan, Rachael Unite, Eric Wertzer, Stephen
White, Jeannie Yoon, and the staff and students of The Princeton Review.

Special thanks to Adam Cadre, Alex Schaffer, Christine Parker, Dave Ragsdale,
and John Fulmer for their enormous contributions to this manual.

Special thanks to Adam Robinson, who conceived of and perfected the Joe
Bloggs approach to standardized testing, and many of the other techniques in this
manual.

Version 7.1

00 Refresher SAT FM & Intro 3 11/15/05 3:28:26 PM


00 Refresher SAT FM & Intro 4 11/15/05 3:28:26 PM
Contents

Introduction . ...........................................................................................1

Math Introduction....................................................................................5

Plugging In..............................................................................................11

Geometry................................................................................................17

Other Approaches..................................................................................25

What If I’m Stuck?..................................................................................35

POOD Review.........................................................................................45

Math Homework.................................................................................... 53

Critical Reading Introduction................................................................81

© The Princeton Review, Inc. | 

00 Refresher SAT FM & Intro 5 11/15/05 3:28:26 PM


Refresher Manual for the SAT

Reading Comprehension........................................................................85

Sentence Completions...........................................................................97

Critical Reading Homework................................................................ 103

Writing Introduction.............................................................................121

The Essay, Revisited............................................................................ 125

Improving Sentences........................................................................... 135

Error IDs and Improving Paragraphs................................................. 143

Grammar Homework........................................................................... 153

Answers & Explanations......................................................................161

vi | © The Princeton Review, Inc.

00 Refresher SAT FM & Intro 6 11/15/05 3:28:26 PM


Introduction

Welcome Back!
Many of you are here because you took a course this summer and you are refresh-
ing your skills for an upcoming test. Others among you have already taken the SAT
and some of you have not yet improved as much as you would like. Fortunately, you
now have another chance!

Regardless of why you are here, we’re are going to do our best to help you im-
prove your score. If you have a problem—you’ve forgotten a technique, or perhaps
never even learned it—let us know. We’re here to help!

How the Refresher Course is Different


Because you’ve already taken one of our courses, there’s no need for us to teach
you our techniques from scratch—you already have a pretty good idea of how to
use them! Instead we’ll re-familiarize you with the techniques and give you more
guided practice in using them efficiently.

© The Princeton Review, Inc. | 

00 Refresher SAT FM & Intro 1 11/15/05 3:28:26 PM


Refresher Manual for the SAT

What Score Improvement Can You Expect for This Course?


This will obviously vary. If you have already gone up more than 150 points, it is
still possible for you to squeeze out some more points. The lower your score is, the
more likely you are to improve dramatically. There are a number of ways that you
can improve your score. If you aren’t doing all of the following, you’re not getting
the highest score that you can.

• Answering the right number of questions


• Choosing the best questions to answer
• Using POE and guessing aggressively
• Practicing the techniques
• Learning more vocabulary

You can’t raise your score if you don’t put in the work. Come to class, ask ques-
tions, do your homework, take the diagnostic tests, analyze your performance....
Put in the work now, so you never have to take the SAT again!

Structure of the SAT


The SAT now has three Math sections, three Critical Reading sections, two multi-
ple-choice Grammar sections, and one essay. There is also one, 25-minute experi-
mental section, which could be reading, math, or grammar. The total testing time
is now 3 hours and 45 minutes.

Math


 
%ASY  %ASY 
 
0ROBLEM -EDIUM   
3OLVING   
 
%ASY 
$IFFICULT   
  
 -EDIUM  
%ASY   

  -EDIUM 
 
 

'RID )N -EDIUM  


 
$IFFICULT 
  $IFFICULT 
$IFFICULT   
  

1UESTIONS 1UESTIONS 1UESTIONS


0ROBLEM3OLVING'RID )N 0ROBLEM3OLVING 0ROBLEM3OLVING
-INUTES -INUTES -INUTES

 | © The Princeton Review, Inc.

00 Refresher SAT FM & Intro 2 11/15/05 3:28:27 PM


Introduction

Critical Reading

Easy 1 1
2 Easy 2
Medium
Sentence 3 3
4 Sentence 4
Completions Difficult
5 Completions Medium 5
6 6
Short 7 7
Difficult
Reading 8 8
9 9
10 Short 10
11 Reading 11
12 12 Easy 1
13 13 Sentence 2
3
Medium
14 14 Completions 4
5
15 15 Difficult 6
Long 16 16 7
8
Reading 17 Long 17 9
18 18 10
Reading 11
19 19 Long 12
20 20 Reading 13
14
21 21 15
22 22 16
23 23 17
18
24 24 19

24 Questions 24 Questions 19 Questions


25-Minute Section 25-Minute Section 20-Minute Section

Grammar
1 New! Grammar questions
Easy . are also ordered roughly
Improving .
. by difficulty.
Sentences Medium .
.
Difficult 11
1
12 .
Easy . Easy .
. .
. .
Error . .
Medium . .
ID . .
. . Improving
. Medium . Sentences
. .
Difficult . .
29 .
30 .
. .
Improving . .
Paragraphs . Difficult .
. .
35 14
35 Questions 14 Questions
25-Minute Section 10-Minute Section

© The Princeton Review, Inc. | 

00 Refresher SAT FM & Intro 3 11/15/05 3:28:29 PM


Refresher Manual for the SAT

Summary
No matter when you first prepped for the SAT, use your TPR techniques! A consis-
tent approach to the SAT will get you far.

 | © The Princeton Review, Inc.

00 Refresher SAT FM & Intro 4 11/15/05 3:28:30 PM


Math Introduction



 
%ASY  %ASY 
 
0ROBLEM -EDIUM   
3OLVING   
 
%ASY 
$IFFICULT   
  
 -EDIUM  
%ASY   

  -EDIUM 
 
 

'RID )N -EDIUM  


 
$IFFICULT 
  $IFFICULT 
$IFFICULT   
  

1UESTIONS 1UESTIONS 1UESTIONS


0ROBLEM3OLVING'RID )N 0ROBLEM3OLVING 0ROBLEM3OLVING
-INUTES -INUTES -INUTES

If you think reading is important only on the Critical Reading section, you’re
wrong. Half of all Math section errors are caused by misreading—these are often
your “careless mistakes.” To help eliminate those “careless mistakes”:

• don’t do the math in your head—write everything down


• re-read the question before you bubble to be sure you are answering the
right question

© The Princeton Review, Inc. | 

01 Math Introduction 5 11/15/05 3:28:34 PM


Refresher Manual for the SAT

PACING
Your target score is the score you will aim for on your next diagnostic or the real
SAT. As your diagnostic score goes up, so will your target score.

So attempt this many questions


You need
To get: 20- 8- 16- Total # of
to earn:
(scaled question question Grid-Ins question questions
(raw
score) PS PS PS to attempt
points)
350 7 6 2 2 2 12
400 12 7 3 3 4 17
450 19 9 4 4 6 23
500 25 11 5 5 8 29
550 32 14 6 6 10 36
600 38 16 6 7 13 42
650 44 18 7 8 15 48
700 47 all all 9 all 53
750 52 all all all all 54
800 54 all all all all 54

Unless you’re shooting for a 700 or higher, do not do every question!

As you can see, you only need to do about half of the questions to get a 500. If
you’re doing more questions than you need to, you’re actually hurting your score.
By rushing through too many questions, you’ll have less time to concentrate on
each question, and you’ll make more careless errors.

Look at the pacing chart and memorize the number of questions you need to
answer to get the score you want. You should use all of your time to work on those
questions, rather than sitting around with ten minutes left at the end of every sec-
tion. Take more time per question and get more of them right. Slow down and
score more.

PERSONAL ORDER OF DIFFICULTY


Remember that the order of difficulty on the SAT is not perfect. You have your own
personal strengths and weaknesses, and you know a lot of strategies that take ad-
vantage of the way ETS writes the test.

Keep ETS’s order of difficulty in mind, but always remember that you control
which questions you answer, and the order in which you answer them. Go through
the section looking for questions that look easier for you.

6 | © The Princeton Review, Inc.

01 Math Introduction 6 11/15/05 3:28:35 PM


Math Introduction

PROCESS OF ELIMINATION (POE)

Always look for ways to eliminate incorrect answers. If you can elimi-
nate even one, guess!

How many answer choices can you eliminate before solving the question?

3. If the sum of 5 consecutive integers is 100, what is


the middle number?
(A) 18
(B) 20
(C) 95
(D) 98
(E) 100

10. When a certain used-clothing store sells clothes, it


keeps 30 percent of the money, and gives the rest to
the original owner. If the original owner of an outfit
receives $98 from a sale, how much did the store
charge for the outfit?
(A) $66
(B) $100
(C) $136
(D) $140
(E) $163

JOE BLOGGS, HAVEN’T WE MET BEFORE?

Joe Bloggs gets difficult questions wrong because he’s predictable.


Don’t be predictable. Don’t fall for ETS traps. Don’t be Joe.

How many JB answers can you eliminate before solving the problem?

18. If a car’s odometer reads 73,333 miles, what is the


LEAST number of miles that the car must travel be-
fore four digits on the odometer are identical again?
(A) 99
(B) 444
(C) 666
(D) 1,111
(E) 4,444

© The Princeton Review, Inc. | 7

01 Math Introduction 7 11/15/05 3:28:35 PM


Refresher Manual for the SAT

19. A shop owner buys apples at wholesale cost and sells


each apple at a profit of 20 percent. If the shop owner
charges the same amount for each apple, and sells
all but 5 percent of his inventory, what percent profit
will he make on his investment?
(A) 10%
(B) 14%
(C) 15%
(D) 18%
(E) 25%

BALLPARKING
Ballparking will also help you eliminate answer choices.

10. Which one of the following numbers would be in-


creased by approximately 100 percent if the order of
its digits were reversed?
(A) 203
(B) 1,002
(C) 1,992
(D) 4,005
(E) 8,004

If you can eliminate any of the answer choices, guess.

8 | © The Princeton Review, Inc.

01 Math Introduction 8 11/15/05 3:28:36 PM


Math Introduction

THE PRINCETON REVIEW’S GUIDE TO GRIDDING

How to Grid
• Write your answer in the spaces at the top before gridding.
• Grid in answers as far to the left in the grid box as possible.
• Don’t reduce fractions if they already fit in the grid.
• Don’t round decimals.
• Don’t grid in mixed fractions.

18. Of 15 people who entered a store in a one-hour


period, two bought nothing, seven bought exactly one
item, and the rest bought two or more items. What
percent of the people who entered the store that hour
bought at least two items? (Ignore the percent sign
when gridding in your answer.)

What You Can’t Grid In (Even If You Wanted To)


• Negatives
• Square roots
• π
• Variables
• % signs
• $ signs

© The Princeton Review, Inc. | 

01 Math Introduction 9 11/15/05 3:28:36 PM


01 Math Introduction 10 11/15/05 3:28:36 PM
PLUGGING IN

Plugging In is the most important math technique. Plugging In turns algebra and
geometry problems into arithmetic problems. It’s the one thing that can help your
score the most.

WHEN TO PLUG IN
Remember that “in terms
8. Charlene is c years old and is 5 years younger than
of” is a needless phrase
Derrick. If Derrick is half as old as Blaine, then, in
that ETS uses to confuse
terms of c, how many years old is Blaine? you. But, it’s our signal to
(A) c – 10 Plug In!
(B) c + 5
(C) c + 10
(D) 2c + 5
(E) 2c + 10

• Do not Plug In numbers that appear in the answer choices or in the


question.
• Do not Plug In zero or one.
• Do not Plug In the same number for two different variables.

© The Princeton Review, Inc. | 11

02 Plugging In 11 11/15/05 3:28:41 PM


Refresher Manual for the SAT

12. Steve usually jogs h kilometers every day. On Mon-


day and Wednesday of last week, however, Steve
jogged twice his usual daily distance, and on Sat-
urday Steve jogged half his usual daily distance. If
Steve jogged his usual daily distance on each of the
other days, how many total kilometers, in terms of h,
did Steve jog last week?
(A) 9h
(B) 8.5h
(C) 7h
(D) 6.5h
(E) 3.5h

Choose numbers that make the arithmetic as easy as possible.

Always check all 5 answer g


17. If is an integer, which of the following must also
choices when you Plug In. h
be an integer?

h
(A)
g

(B) g

(C) gh

g2
(D)
h
g2
(E)
h2

Remember that you can plug in on any problem that has variables in the an-
swers.



Don’t forget the rules
of geometry when
Plugging In!

12. In the fi gure above, what is the value of b in terms of


a?
(A) 90 + a
(B) 90 + 2a
(C) 180 – 2a
(D) 360 – 2a
(E) 2a

12 | © The Princeton Review, Inc.

02 Plugging In 12 11/15/05 3:28:42 PM


Plugging In

15. In a laboratory setting, the population of a certain


bacteria doubles every 3 hours. If the initial popula-
tion was 12, which of the following expresses the
population after h hours?
h

(A) 2 × 3 2
h

(B) 3 × 12 2

(C) 12 × 23h
h

(D) 12 × 2 3

(E) 12 × 2 h

Sometimes, you can even plug in on Grid-Ins!

18. A merchant reduces the original price of a pair of


shoes by 10 percent. After several weeks, the mer-
chant increases the new price of the shoes by 50 per-
cent. The resulting price of the shoes is what percent
greater than the original price of the shoes? (Disre-
gard the percent sign when gridding your answer.)

© The Princeton Review, Inc. | 13

02 Plugging In 13 11/15/05 3:28:45 PM


Refresher Manual for the SAT

PLUGGING IN THE ANSWERS


On the SATs you’ll have plenty of chances to PITA. Just remember the following
rules:

Label your answers.

Starting with answer choice C, work the steps of the problem.

Look for something in the problem that tells you if the answer is correct.

When you fi nd the correct answer, STOP.

16. The combined cost of items X, Y, and Z is $225. If Y


costs $10 more than X, and Z costs $10 more than Y,
then what is the cost of X ?
Label → ________ ________ _______

(A) $85
(B) $80
(C) $75
(D) $65
(E) $55

PLUGGING IN TIMED DRILL


See how well you do on this drill. Just remember all the different ways you can
plug in. If you’re having trouble, see what other techniques you can apply. It’s all
about POOD!

Time: 10 minutes

# of Questions to
Target Score
Attempt
< 450 3 or 4
460–550 4 or 5
560–650 6 or 7
> 650 All

6. Alex is three times as old as Betty. In fi ve years, Alex


will be twice as old as Betty. How old is Betty now?
(A) 5
(B) 10
(C) 15
(D) 20
(E) 30

14 | © The Princeton Review, Inc.

02 Plugging In 14 11/15/05 3:28:46 PM


Plugging In

a 3
7. If ab = 48 and = , which of the following could
b 4
be the value of a ?
(A) 4
(B) 6
(C) 8
(D) 12
(E) 16

10. In the figure above, if x = 4y, what is the value of y ?


(A) 4
(B) 16
(C) 18
(D) 36
(E) 72

1 1
11. If = t and t 2 = , what is r in terms of s ?
s2 r
(A) 4
s
(B) s
(C) 4s
(D) s 2
(E) s 4

13. Jerome won the lottery. He paid one quarter of his


winnings to the government in taxes. He gave $3,000
to his mother and $1,000 to his friend. If he had
$11,000 left, how much money did he originally win?
(A) $12,000
(B) $15,000
(C) $18,000
(D) $20,000
(E) $45,000

16. If x is k percent of y, what percent of y is kx ?

(A) k
%
100
100
(B) %
k
(C) k%

(D) 100k%

(E) k 2 %
© The Princeton Review, Inc. | 15

02 Plugging In 15 11/15/05 3:28:49 PM


Refresher Manual for the SAT

A C

18. In right triangle ABC above, AB is twice BC. If


AB = x, what is the area of  ABC in terms of x ?

x 2
(A)
8
x 3
(B)
8
x2 2
(C)
4
x2 3
(D)
4
x2 3
(E)
8

19. A car salesman sells half of the cars in his showroom


in one week. The next week, he sells one-third of the
remaining cars. At the end of the two weeks, what
fraction of the original number of cars did he sell?

1
(A)
6
1
(B)
2
2
(C)
3
3
(D)
4
5
(E)
6

16 | © The Princeton Review, Inc.

02 Plugging In 16 11/15/05 3:28:52 PM


Geometry
Reference Information

l h •r
r h c 60 x s 45 s 2
• h b 2x
w
w 30 45
A = lw
b l a s
A= r 2
x 3
C=2 r A = 1 bh V = lwh V = r 2h c 2 = a 2 + b2
2 Special Right Triangles
The number of degrees of arc in a circle is 360.
The sum of the measures in degrees of the angles of a triangle is 180.

Remember that you’re given many of the formulas you’ll need for the geometry.
Now all that’s left is to be able to use the information you’re given to your advan-
tage.

Zƒ Xƒ
.OTE&IGURENOTDRAWNTOSCALE

10. In the figure above, if z = 110, then z + y = Write any info from the
(A) 200 problem on the figure.
(B) 180 Work from what you know
(C) 130 to what you don’t know.
(D) 110
(E) 90

What information are you given?

What rules can help you?

© The Princeton Review, Inc. | 17

03 Geometry 17 11/15/05 3:28:56 PM


Refresher Manual for the SAT

"

% &
! #

13. Quadrilateral ABCD shown above has an area of 72.


If ED = BF = 6, what is the length of AC ?
(A) 6
(B) 12
(C) 18
(D) 24
(E) 36

Write down the formula you need:

Now fill in what you already know:

What do you need to find to finish the problem?

No figure given? No problem! Just draw your own.

Draw figures that are 14. In a right triangle, one leg has length x and the other
missing. Fill in anything 4
has a length of x . Which of the following express-
you already know. 3
es the length of the hypotenuse in terms of x ?

3
(A) x
5
3
(B) x
4
4
(C) x
5
5
(D) x
4
5
(E) x
3
Just keep applying the basic geometry advice, even if a question involves a num-
ber of steps.

18 | © The Princeton Review, Inc.

03 Geometry 18 11/15/05 3:28:58 PM


Geometry

12. If Circle O has a radius of 4, what is the ratio of the


circumference of Circle O to the area of Circle O ?
(A) 1:4
(B) 1:2
(C) 1:1
(D) 2:1
(E) 4:1

What should you do first?

What should you do next?

" #

ƒ
! $

6. In the figure above, if AB = BC = 8, what is the area Draw a line to create basic
of ABCD ? shapes that have nice
formulas.
(A) 144
(B) 128
(C) 96
(D) 64
(E) 48

10. What is the slope of the line that passes through the
points with coordinates (2, 6) and (3, 5) ?

(A) –1

1
(B) –
2
1
(C)
2
(D) 1

(E) 2

© The Princeton Review, Inc. | 19

03 Geometry 19 11/15/05 3:28:59 PM


Refresher Manual for the SAT

GEOMETRY Timed DRILL


Test out your geometry skills. Choose the problems that you know how to do first.
Then see what you can eliminate on the others.

Time: 10 minutes

Target Score # of Questions to


Attempt
< 450 3 or 4
460–550 4 or 5
560–650 6 or 7
> 650 All

s#

s" $s

Aƒ Bƒ
s Bƒ s Bƒ s
% ! &
.OTE&IGURENOTDRAWNTOSCALE

3. In the figure above, if b = 25, what is the value of a ?


(A) 15
(B) 75
(C) 90
(D) 105
(E) 155

ƒ 3

1 Zƒ

ƒ 4

4. In the figure above, if PT = TQ, QS = SR, then z =


(A) 10
(B) 40
(C) 60
(D) 90
(E) 110

20 | © The Princeton Review, Inc.

03 Geometry 20 11/15/05 3:28:59 PM


Geometry

L L


X  ƒ

10. In the figure above, if l1 || l2 , then x =


(A) 180
(B) 110
(C) 60
(D) 40
(E) 30

/
1 s 2

0 4 3

11. In the figure above, side PS of rectangle PQRS is


tangent to the circle with center O at point T. If
QR = k, what is the area of rectangle PQRS in terms
of k ?

k2
(A)
4
k2
(B)
2

(C) k 2

(D) 2 k 2

(E) 4 k 2

© The Princeton Review, Inc. | 21

03 Geometry 21 11/15/05 3:29:01 PM


Refresher Manual for the SAT

Y
L

 X
L

L

16. Lines l1 , l2 , and l3 have slopes of a, b, and c, respec-


tively, and l2 is parallel to the x axis. Which of the
following is a negative number?
(A) b+c
(B) a+b
(C) c–a
(D) b
(E) 2c

17. If the volume of a rectangular solid is 64, its length is


twice its width, and its width is twice its height, then
what is its length?
(A) 2
(B) 4
(C) 8
(D) 16
(E) 32

18. Cubes A and B have faces with areas x and y, respec-


tively. If the ratio of x to y is 1 to 9, what is the ratio
of the volume of cube A to the volume of cube B ?
(A) 1:3
(B) 1:9
(C) 1 : 27
(D) 1 : 81
(E) 1 : 729

22 | © The Princeton Review, Inc.

03 Geometry 22 11/15/05 3:29:03 PM


Geometry

19. Square ABCD (not shown) has vertex A at (–1, 1),


vertex B at (–1, 4), and vertex D at (2, 1). What is the
slope of the line passing through vertex C and the
origin?

(A) –2

1
(B) –
2
3
(C)
2
(D) 2

(E) 4

© The Princeton Review, Inc. | 23

03 Geometry 23 11/15/05 3:29:04 PM


03 Geometry 24 11/15/05 3:29:04 PM
Other Approaches

Want a better score? ETS expects you to complete problems the way that you were
taught in school. But, anytime that you do what the test writer expects, you don’t
get the best score that you could.

Arithmetic
Don’t do these the way that ETS expects!

17. A factory produces an average of 50 televisions per As soon as you see the
day for 4 days, and an average of 20 televisions per word average, draw an
day for the next 8 days. What is the average number average pie.
of televisions produced per day by the factory over
the entire 12-day period?
(A) 12
(B) 20
(C) 30
(D) 35
(E) 36

© The Princeton Review, Inc. | 25

04 Other app 25 11/15/05 3:29:08 PM


Refresher Manual for the SAT

Use the ratio box for part 11. A fruit vendor sells 8 bananas for every 5 mangoes. If
to part comparisons. the vendor sells 24 bananas, what is the total number
of pieces of fruit sold?
(A) 13
(B) 15
(C) 24
(D) 37
(E) 39

m
m
m

9. A team won 36 of the 60 games it played. If there


were no games tied or forfeited, what percent of the
games did the team lose?
(A) 24%
(B) 36%
(C) 40%
(D) 50%
(E) 60%

Translate the question into math:

What percent of the games did the team lose?

What’s the best formula to 14. Kumar finds that the number of mistakes that he
use for inverse variation? makes on a 50-question test varies inversely with the
amount of time that he spends studying. If Kumar
made 10 mistakes on his last 50-question test and
studied for 3 hours, how many mistakes can he ex-
pect to make on his next such test if he studies for 5
hours?
(A) 5
(B) 6
(C) 15
(D) 17
(E) 30

26 | © The Princeton Review, Inc.

04 Other app 26 11/15/05 3:29:08 PM


Other Approaches

7. A store is encouraging business by giving away door How do you discover a


prizes. A prize is given to the second customer of the pattern?
day and to every fifth customer after that. Which one
of the following customers will receive a prize?
(A) 61
(B) 65
(C) 68
(D) 73
(E) 82

15. For all x ≥ 1 , let f ( x ) = (1 + x − 1 )2 . For which of What technique will help
the following values of x does f ( x ) = 9 ? with this question?
(A) 2
(B) 3
(C) 4
(D) 5
(E) 9

10. All of the students enrolled in a certain school district What’s the best way to
are between the ages of 5 and 19, inclusive. If a handle this question?
student whose age is x enrolls in this school district,
which of the following most accurately expresses all
possible values for x ?

(A) x − 19 ≤ 5
(B) x − 12 ≤ 7
(C) x − 24 ≤ 14
(D) x − 12 ≤ 5
(E) x − 19 ≤ 7

© The Princeton Review, Inc. | 27

04 Other app 27 11/15/05 3:29:10 PM


Refresher Manual for the SAT

Graphs
Here are some ways that ETS tests graphs. What’s the easiest way to do each of
these problems?

y
7

4
y = f (x)
3

x
O 1 2 3 4 5 6 7 8 9 10 11 12 13

16. The figure above shows the graph of y = f (x). The


function g is defined as g (x) = 2f (x + 3). If g (x) = 6,
which of the following could be the value of x ?
(A) 1
(B) 2
(C) 3
(D) 7
(E) 8
Y

X
, .
.OTE&IGURENOTDRAWNTOSCALE

14. The figure above shows the graph of y = f ( x ) for


−4 ≤ x ≤ 4 . If f ( x ) = − x 2 + 16, what is the area of
LMN(not shown)?
(A) 32
(B) 24
(C) 16
(D) 8
(E) 4

28 | © The Princeton Review, Inc.

04 Other app 28 11/15/05 3:29:12 PM


Other Approaches

s 
X
/

11. The graph of y = f ( x ) is shown in the figure above.


Which of the following shows the reflection of the
graph of y = f ( x ) across the x-axis?

Y Y

s
n 
(A) X (D) /
X
/
s
 n

Y Y

  n 
s s
(B) /
X (E) /
X

(C) /
 n
X

© The Princeton Review, Inc. | 29

04 Other app 29 11/15/05 3:29:13 PM


Refresher Manual for the SAT

O x

A •
(–1, –1)

8. The graph of the equation f ( x ) = x + 1 − 1 is shown


above. For which of the following equations would
point A be located at (1, 1) ?

(A) f ( x ) = x − 1 + 1
(B) f ( x ) = x − 1 − 1
(C) f ( x ) = x + 1 + 1
(D) f ( x ) = x − 1
(E) f ( x ) = x + 1

Other Approaches TIMED Drill


Try these.
Time: 10 minutes

Target Score # of Questions to


Attempt
< 450 3 or 4
460–550 5 or 6
560–650 7 or 8
> 650 All

7. At a certain time of day, a man 6 feet tall casts a


shadow 8 feet long. If, at the same time, a lamppost
casts a shadow 20 feet long, then how many feet tall
is the lamppost?
(A) 12
(B) 14
(C) 15
(D) 22
(E) 25
30 | © The Princeton Review, Inc.

04 Other app 30 11/15/05 3:29:15 PM


Other Approaches

List S: 4, 9, 9, 4, 1, 9, 5

8. The median of the numbers in list S will remain un-


changed if which of the following numbers is added
to the list?
(A) 4
(B) 5
(C) 6
(D) 7
(E) 9

13. Will is selecting clothes to wear to school. He must


choose one pair of sneakers, one pair of jeans, and
one shirt. If Will has four pairs of sneakers, five pairs
of jeans, and twelve shirts, how many different outfits
could he wear?

14. If the average (arithmetic mean) of the degree mea-


sures of two angles of a right triangle is 70, which of
the following must represent the degree measure of
one of the three angles of the triangle?
(A) 20
(B) 25
(C) 30
(D) 35
(E) 40

15. In 1995 the average price of CD players was $300. In


1999 the average price of CD players was $240. By
what percent did the average price of a CD player
change between 1995 and 1999?
(A) 60%
(B) 40%
(C) 25%
(D) 20%
(E) 5%

© The Princeton Review, Inc. | 31

04 Other app 31 11/15/05 3:29:15 PM


Refresher Manual for the SAT

16. Alice has a full box of purple, white, and green tennis
balls. There are twice as many purple balls as white
balls, and three times as many green balls as purple
balls. If Alice pulls out one ball at random, what is
the probability that it will be purple?

2
(A)
9
1
(B)
3
2
(C)
5
3
(D)
5
2
(E)
3
x +1
17. Let x @ y be equal to . For which of the follow-
y+2
ing is x @ y the greatest?
(A) x = 6, y = –6
(B) x = 6, y = –1
(C) x = 5, y = 7
(D) x = 5, y = 2
(E) x = 4, y = 4

18. In the first four months of their season, the Cooper-


stown baseball team won 3 games for every 4 it lost,
with no game ending in a tie. In the remainder of the
season, the team played 7 games and won all of them.
If at the end of the season its ratio of wins to losses
is 1 to 1, what is the total number of games that the
team has played?

32 | © The Princeton Review, Inc.

04 Other app 32 11/15/05 3:29:17 PM


Other Approaches

y = x2

J K

x
0

y = bx2 – 25

Note: Figure not drawn to scale.

20. The figure above shows the graphs of y = x 2 and


y = bx 2 − 25 for some constant b. If the length of JK
is equal to 10, what is the value of b ?
(A) 1
(B) 2
(C) 4
(D) 10
(E) 25

© The Princeton Review, Inc. | 33

04 Other app 33 11/15/05 3:29:18 PM


04 Other app 34 11/15/05 3:29:18 PM
What If I’m Stuck?

Even if you don’t know how to solve a problem, there are things that you can try.

POE

14. A compact disc is placed in a player that randomly Just ask Joe! Then,
eliminate his answer.
selects and plays songs from the compact disc. The
compact disc contains 3 ballads, 4 instrumental

pieces, x dance tracks, and no other pieces. If the

probability that the first song played will be a ballad


1
is , what is the value of x ?
4
(A) 1
(B) 4
(C) 5
(D) 9
(E) 12

© The Princeton Review, Inc. | 35

05 What if Stuck 35 11/15/05 3:29:22 PM


Refresher Manual for the SAT

.OTE&IGURENOTDRAWNTOSCALE

What does Joe do? 15. In the figure above, three lines intersect as shown. If
a = 80, what is the value of b + c ?
(A) 100
(B) 180
(C) 260
(D) 340
(E) It cannot be determined from the
information given.

Joe also helps you to avoid picking answers too quickly on problems that you know
how to do. Careless errors can cost you a lot of points.

1
Are any answers the 8. A candy store offers a 33 % discount on any pur-
wrong size? 3
chase of three boxes of chocolates. Sheila pays $12

for three boxes of chocolates. What was the amount

of her discount, in dollars?


(A) 3
(B) 4
(C) 6
(D) 9
(E) 12

36 | © The Princeton Review, Inc.

05 What if Stuck 36 11/15/05 3:29:22 PM


What If I’m Stuck?

" #

! $

.OTE&IGURENOTDRAWNTOSCALE

Note: Figure not drawn to scale.

16. Rectangle ABCD has length 6 and width 8. What is Use the figure. Ballpark!
the area of the shaded region?

(A) 25π − 48
2
25π
(B) − 24
2

(C) 10π

(D) 25π

(E) 50π − 24

ETS loves to include partial answers. Sometimes you can eliminate something
that is wrong even if you only work the first step of the problem.

16. A factory produced 3,000 chairs on Monday, of How many partial


which 70 percent were painted. Of the chairs that answers can you find in
were painted, 40 percent were painted blue. How this problem?
many more chairs were NOT painted than were
painted blue?
(A) 60
(B) 360
(C) 840
(D) 900
(E) 1,260

© The Princeton Review, Inc. | 37

05 What if Stuck 37 11/15/05 3:29:24 PM


Refresher Manual for the SAT

What answer could you 18. A radio antenna and a three-foot pole are installed 42
get by just doing simple feet apart on a level flat roof. If a wire runs from the
operations with the top of the pole to the top of the antenna, and the wire
numbers? Would the rises four feet for every three feet it travels horizon-
answer really be that tally, what is the height of the antenna in feet?
easy?
(A) 42
(B) 45
(C) 56
(D) 59
(E) 70

More Plugging In
Plugging In and Plugging In The Answers can save the day on lots of different
kinds of questions. How can you use the technique on these?

z+2 8
20. If = , then what is one possible value of z ?
z−3 z

(A) −6
(B) −4
(C) −2
(D) 2
(E) 12

x 2 3 4 5
y 7 10 13 16

4. The table above represents a relationship between


x and y. Which of the following linear equations
describes the relationship?
(A) y=x+1
(B) y=x+5
(C) y = 3x
(D) y = 3x + 1
(E) y = 4x – 1

38 | © The Princeton Review, Inc.

05 What if Stuck 38 11/15/05 3:29:24 PM


What If I’m Stuck?

18. At a certain company, an employee’s salary s in dol-

lars is given by the function s(y) = 1,500y + 30,000,

where y is the number of years the employee has

worked at the company. If the value v in dollars of the

car that the employee owns is given by the function


s − 18, 000
v( s ) = , and Kelly is an employee of this
2
company whose car has a value of $30,000, how

many years has Kelly worked for this company?


(A) 34
(B) 32
(C) 18
(D) 12
(E) 8

13. Let the function f be defined by f (x) = 2x – 9. If


f (a) = b, what is the difference between f (2a) and
f (a), in terms of b ?
(A) 2b – 9
(B) 2b
(C) b
(D) b+9
(E) 2b + 9

© The Princeton Review, Inc. | 39

05 What if Stuck 39 11/15/05 3:29:25 PM


Refresher Manual for the SAT

TIMED Drill
Time: 15 minutes

Target Score # of Questions to


Attempt
< 450 6 to 8
460–550 9 to 11
560–650 12 to 14
> 650 All

3. If a square of area 25 has vertices at


Nn (–3, –1), (2, –1), and (–3, 4), what
are the coordinates of the remaining
vertex?
N 
 (A) (–3, –2)
(B) (–2, 4)
(C) (1, 4)
(D) (2, –3)
(E) (2, 4)

1. The areas are given for each of the


small rectangles in the figure above.
What is the area of the entire figure? 75°
a° 2a°
(A) n+6
(B) n+7 30°
(C) 2n – 7
(D) 5n + 7
(E) 5n + 17 4. In the figure above, what is the value
of a ?
2. If the price of postcards ranges from
(A) 60
40 cents to 70 cents each, what is
(B) 85
the greatest number of postcards that (C) 170
can be purchased with $3.00 ? (D) 180
(A) 8 (E) 255
(B) 7
(C) 6
(D) 5
(E) 4

40 | © The Princeton Review, Inc.

05 What if Stuck 40 11/15/05 3:29:25 PM


What If I’m Stuck?

5. If y = f ( x ) , which of the following


could be the graph of y = f ( x ) ? .5-"%2/&4/!34%233/,$"9
4/34)4#/ n
Y

.UMBERSOF4OASTERS3OLD
n

(A) X n


INTHOUSANDS
n
n

     

6. According to the graph above, the


 X greatest annual percentage change in
(B) the number of toasters sold by Tostit
Co. took place between the years
(A) 1996 and 1997
(B) 1997 and 1998
(C) 1998 and 1999
(D) 1999 and 2000
Y (E) 2000 and 2001

 X
(C)

Note: Figure not drawn to scale.


(D) X

7. The laminated block shown above
consists of a layer of wood between
two layers of plastic. If each plastic
layer is one-third as thick as the
wooden layer, and the thickness of
each layer is an integer, what is one
Y
possible height of a stack of such
blocks?
(E) X (A) 18

(B) 24
(C) 35
(D) 39
(E) 42

© The Princeton Review, Inc. | 41

05 What if Stuck 41 11/15/05 3:29:27 PM


Refresher Manual for the SAT

8. If x is divisible by 5, what is the 11. Set A consists of the positive odd


remainder when 3(x + 2) is divided numbers, set B consists of the
by 5 ? integers between 2 and 12 inclusive,
set C consists of the positive prime
(A) 0
numbers less than or equal to 25,
(B) 1
and set D consists of the distinct
(C) 2
positive integer factors of 30. If
(D) 4
(E) 6 X = A ∩ B , and Y = C ∩ D , which
of the following is X ∪ Y ?
9. If r and s are nonzero integers, and (A) {1, 2, 3, 5, 7, 9, 11}
r + s = 0, which of the following (B) {2, 3, 5, 7, 9, 11}
must be true? (C) {3, 5}
(D) {2, 7, 9, 11}
(A) 2r = 2s (E) {2, 3, 5}

(B) rs = r2
12. Lori is forming a team consisting
(C) 2r – s = r + s of a team leader, a researcher,
and an assistant to develop a new
(D) r2 + s2 = 0 laser. If she has 7 scientists from
which to choose, then how many
2
(E) r = 1 arrangements of scientists are
s2 available?
10. If x is 3 times y, and y is 4 more than (A) 21
x, then what is the sum of x and y ? (B) 35
(C) 210
(A) –8
(D) 314
(B) –4
(E) 343
(C) 0
(D) 4
(E) 7

42 | © The Princeton Review, Inc.

05 What if Stuck 42 11/15/05 3:29:28 PM


What If I’m Stuck?

13. A certain artist’s income is directly


proportional to the number of
paintings she sells, and the number
of paintings she sells is directly ! "
proportional to the number of art
shows she has per month. She must
distribute at least 30 flyers for each
show. Last month she earned $2,400
by selling 120 paintings during the 'AZEBO 3TATUE &OUNTAIN
4 art shows she had. If she needs to
earn $3,600 this month, what is the
minimum number of flyers she can
distribute? 15. The diagram above shows all paths
in a garden between gazebo A and
(A) 6 gazebo B. If each path segment is
(B) 60 30 feet long, what is the length in
(C) 120 feet of the longest path that can be
(D) 150 walked from gazebo A to gazebo B
(E) 180 without passing a statue or retracing
any path segment?
14. The wheels of a train each have a
radius of 9 inches. If the train is (A) 60
traveling on a track with light posts (B) 180
placed every 600 feet, how many (C) 240
revolutions will each wheel make (D) 300
(E) 360
between one light post and the next?

200
(A) (approximately 63.66)
π
400
(B) (approximately 127.32)
π

(C) 50π (approximately 157.08)

(D) 100π (approximately 314.16)

(E) 200π (approximately 628.32)

© The Princeton Review, Inc. | 43

05 What if Stuck 43 11/15/05 3:29:29 PM


05 What if Stuck 44 11/15/05 3:29:29 PM
POOD Review

It’s time to put it all together. Remember that good pacing and careful problem
selection have a huge impact on your score. Try these problems. If you can’t fig-
ure out how to solve it, can you eliminate any answer choices by ballparking? Are
there Joe Bloggs answers? Did you try to Plug In or PITA? Good test takers are
flexible in their approach.

6. Three squares, each with a perimeter of 12, are joined Do it


to form a rectangle. What is the perimeter of the Skip it
resulting rectangle?
(A) 18
(B) 24
(C) 27
(D) 36
(E) 48

© The Princeton Review, Inc. | 45

06 POOD Review 45 11/15/05 3:29:33 PM


Refresher Manual for the SAT

Do it 7. During a certain period in the growth of an organism,


Skip it the number of cells contained in its body grows expo-
nentially. If the organism contained 1,500 cells at the
beginning of this period, and the number of cells in
the organism doubles every 12 hours, how many cells
does the organism contain after x hours?
(A) 15002x
x
12
(B) 1500 × 2
x
(C) 1500 × 2
x
12
(D) 2 × 1500
12
x
(E) 2 × 1500

Do it 8. In a dog show, 30 percent of the male dogs and 15


Skip it percent of the female dogs won prizes. If 30 male
dogs and 20 female dogs participated in the com-
petition, what percent of the dogs in the show won
prizes?
(A) 12%
(B) 22.5%
(C) 24%
(D) 45%
(E) 90%

46 | © The Princeton Review, Inc.

06 POOD Review 46 11/15/05 3:29:34 PM


POOD Review

9. Jackie takes 5 days to read a short book. If she reads Do it


10 pages on the first day, nothing on the second day, Skip it
10 more pages on the third day, nothing on the fourth
day, and 20 pages on the fifth day, which of the fol-
lowing graphs could be used to show her progress
through the book?

OFPAGES
(A)

X
     
$AY

OFPAGES
(B)

X
     
$AY

OFPAGES
(C)

X
     
$AY

OFPAGES
(D)

X
     
$AY

OFPAGES
(E)

X
     
$AY

© The Princeton Review, Inc. | 47

06 POOD Review 47 11/15/05 3:29:35 PM


Refresher Manual for the SAT

M: {2, 5, 7}
N: {2, 3, 7, 9}

Do it 10. Which of the following is the average


Skip it (arithmetic mean) of the members of M ∪ N ?
(A) 4

1
(B) 4
2

(C) 5

1
(D) 5
5
2
(E) 5
3
1
Do it 11. If x −2 = 81 , what is the value of x 2 ?
Skip it
1
(A) −
9
1
(B)
9
1
(C)
3

(D) 3

(E) 9

Do it 12. A right triangle has a perimeter of 24 and a hypote-


Skip it nuse of 10. If the length of all three sides are integers,
what is the triangle’s area?
(A) 6
(B) 12
(C) 14
(D) 24
(E) It cannot be determined from the
information given.

48 | © The Princeton Review, Inc.

06 POOD Review 48 11/15/05 3:29:37 PM


POOD Review

13. If the y = x − 5 is graphed in the xy-coordinate sys- Do it


tem, what is the distance between the x-intercepts? Skip it
(A) −5
(B) 0
(C) 2.5
(D) 5
(E) 10

14. If 4
x 5 − 227 = 2 , then x = Do it
Skip it
(A) 5
−211
(B) 2
(C) 3
(D) 16
(E) 243

15. In a figure-skating competition, each contestant Do it


receives 10 scores. After the lowest and highest Skip it
scores are removed, the average (arithmetic mean) of
the remaining 8 scores is calculated. The average of
Jamie’s ten scores is b and the average of her lowest
and highest scores is d. In terms of b and d, what is
the average of Jamie’s eight remaining scores?

(A) 10 b − 2 d

b−d
(B)
8
5b − d
(C)
5
8b − d
(D)
4
5b − d
(E)
4

© The Princeton Review, Inc. | 49

06 POOD Review 49 11/15/05 3:29:40 PM


Refresher Manual for the SAT

Do it 16. A bicyclist travels at a rate of m miles per hour for z


Skip it hours. If z is greater than 3, then the bicyclist would
have to increase his rate by how many miles per
hour, in terms of m and z, in order to travel the same
distance in 3 fewer hours?
(A) m(z – 3)

mz
(B)
z−3

(C) z(m – 3)

z−3
(D)
mz
mz
(E) −m
z−3
Do it 17. An empty fuel-storage tank with a capacity of x gal-
Skip it lons is filled completely by a supply pump at a rate
of 5 gallons per minute. The tank is then immediately
drained by an exhaust pump at a rate of 4 gallons per
minute. If the entire process takes 18 hours, what is
the value of x ?
(A) 2,000
(B) 2,400
(C) 3,000
(D) 3,600
(E) 5,400

Do it 18. If the graph of f (x) = x2 + mx + n intersects the x-axis


Skip it exactly one time, and f (–3) = 0, what is the value of
m+n?
(A) 15
(B) 9
(C) 3
(D) 0
(E) −3

50 | © The Princeton Review, Inc.

06 POOD Review 50 11/15/05 3:29:41 PM


POOD Review

2 3 Do it
Skip it
A

B

1 4

19. The area of rectangle QRST in the figure above is 48.


6 a+b
If = , what is the value of a ?
a 6

(A) 2
(B) 3.6
(C) 6.4
(D) 8
(E) 10

20. The first two terms of a sequence are 7 and 12. Be- Do it
ginning with the third term, each term is 3 less than Skip it
the sum of the two numbers immediately preceding it.
For example, the third term is 16, because
(7 + 12) – 3 = 16. How many of the first 100 terms in
this sequence are odd numbers?
(A) 33
(B) 34
(C) 50
(D) 66
(E) 67

© The Princeton Review, Inc. | 51

06 POOD Review 51 11/15/05 3:29:41 PM


06 POOD Review 52 11/15/05 3:29:41 PM
mAth homeWorK

DENVER DAVE’S HOMEWORK REVIEW MODEL


Remember: Review EVERY question you work on whether you get it right or wrong,
and categorize each question as follows.

Nailed it. Mark a question with an N if you got this question right
and totally understood both it and the underlying con-
cepts. You know that you could get this type of question
right regardless of how ETS presents it.
Got it/Guessed. Mark a question with a G if you got this question right
either by guessing or in some mysterious way you can’t
remember afterwards. While we’ll take luck on test day,
not so much during the class. The point is to be able
to replicate the process by which you correctly answer
questions on test day. Be sure to review these questions
so that you know EXACTLY how it is answered.
Missed it—Doh! Mark a question with an M if you missed it, but under-
stood it upon review. This is the best kind of learning. If
you make a silly mistake once and realize why you did so,
you are that much closer to NOT doing it next time!
Faster Way? Mark a question with an F if you got the question right
but want to know if there is a faster way to solve it.
Huh? Mark a question with an H if you missed this question
and can’t fi gure it out on your own. This is defi nitely the
kind of question you ask about during homework review
at the beginning of each class.

On the homework pages that follow:

• Do the questions, then check your answers against the answer


key.

• Mark each question with an N, G, M, F, or H, depending on


what category it falls into.

• Get your hands on a brightly colored highlighter and


CLEARLY MARK those questions you want to ask your teacher
about in the next class. This will make them easier for you to
fi nd.

© The Princeton Review, Inc. | 53

07 Math HW 53 11/15/05 3:29:47 PM


Refresher Manual for the SAT

PLUGGING IN

6. The difference between (p + 5) and 8. The number of clients a certain


(p – 7) is company serves triples for each year
the company is in business. After 5
(A) 2
years in business, the company now
(B) 12
serves 10,935 clients. If the company
(C) p–2
originally served c clients when it
(D) 2p – 2
(E) p + 12 first formed, what is the value of c ?
(A) 5
6. In a certain store, small puppets cost (B) 15
$1 each and large puppets cost $2 (C) 45
each. If the store sold 25 puppets (D) 729
for a total of $30, how many small (E) 3,645
puppets were sold?
9. Alicia is 5 years younger than Jane,
(A) 25
who is currently j years old. In terms
(B) 20
of j, how old will Alicia be in 8
(C) 15
years?
(D) 10
(E) 5 (A) j+3
(B) j–1
6. On Tuesday, two-thirds of a large (C) j–3
block of ice melted. On Wednesday, (D) j–5
one-half of the remaining ice melted. (E) j–8
If the block of ice then weighed 60
pounds, how much did it weigh, in s$
pounds, at the beginning of the day
on Tuesday? !
s Xƒ
(A) 540
(B) 480
Aƒ '
(C) 360 #s s s%
(D) 180 Bƒ
(E) 20
s
7. Printer A prints at a constant rate of x &
pages per hour. Printer B’s constant
rate is one-third that of Printer A. If
9. In the figure above, CD intersects
the two printers worked together
for three hours, how many pages, in AF at A and AF intersects CE at
terms of x, will they print? G. What is the value of x in terms of
a and b?
(A) 4x
(A) 90 + a − b
(B) 3x
(B) 90 + a + b
(C) 2x (C) 180 + a − b
4 (D) 180 + a + b
(D) x
3 (E) 180 − b
(E) x

54 | © The Princeton Review, Inc.

07 Math HW 54 11/15/05 3:29:48 PM


Math Homework

9. If a is a positive integer, which of the 11. Marlene is m feet tall and Albert
following must be a positive even is w inches tall. If Marlene is taller
integer? than Albert, which of the following
2 formulas expresses, in feet, how
(A) (a + 2)
much taller Marlene is than Albert?
(B) 2a + 1
(A) m – 12w
(C) 2a

(D) 3a + 2 (B) m – w
a
(E)
(C) w − 12m
3
10. Ernie is three times as old as Bert 12
and four years younger than Roger. 12m − w
If Bert is b years old, which of the (D)
12
following expresses Roger’s age?

(E) 12m – w
1
(A) b – 4
3 11. Circle A has a radius of v. The area
(B) b – 4 of Circle B is twice the area of
Circle A. If the radius of Circle B is
(C) b + 4 w, what is v in terms of w ?

(D) 3b – 4
w
(A)
(E) 3b + 4 2

w
10. If one-fourth of a number is 3 less (B)
than half of the number, what is the 2
number?
(C) w 2
(A) –12
(B) 6
(C) 12
(D) 16 (D) 2w
(E) 18

(E) 4w

© The Princeton Review, Inc. | 55

07 Math HW 55 11/15/05 3:29:50 PM


Refresher Manual for the SAT

13. If 20% of the science students at


10 a + 6 Central High study only physics,
11. If x = , and y = 5a, what is x
4 25% study only chemistry and the
in terms of y ? rest study only biology, what is the
smallest number of science students
who could be studying biology?
y+3
(A)
2 (A) 4
y+6 (B) 9
(B)
2 (C) 11
y+6 (D) 15
(C)
4 (E) 20
5y + 3
(D)
10 3
a2
(E) 20y – 6 13. If = 1 , what is the value of a ?
2a
1
12. If z =
2
, then x 2 = 1
x (A)
8
1
(A) 4 z 2 (B)
2

(B) z 2 (C) 2

1 (D) 2
(C)
z2
1 (E) 8
(D)
z4
1 1
14. If a = 4 b = 8c then, in terms of c,
(E) 2
2z4
what is the value of a + b ?
2 3
12. If (2m) = m , what is the value of m?
9c
(A) 1 (A)
2
(B) 2
(C) 3 (B) 6c
(D) 4
(E) 6 (C) 8c
25c
1 (D)
13. The weight of a barrel is of the 2
4
(E) 18c
weight of the water it contains when
full. If, when full, the barrel and
water weigh a total of 20 pounds,
what is the weight, in pounds, of the
empty barrel?
(A) 4
(B) 5
(C) 15
(D) 16
(E) 80

56 | © The Princeton Review, Inc.

07 Math HW 56 11/15/05 3:29:55 PM


Math Homework

14. Jenn saved some money that she 16. If 2x = 3y = 4w, what is 5x + 6w in
terms of y ?
earned working at a summer job.
(A) 8y
2 (B) 9y
She spent of the money on
5 (C) 10y
1 (D) 12y
clothes and of it on a DVD
4 (E) 15y
player. If she has $350 remaining,
16. If 0 < a < 1 and b < 0, which of the
how much money had she originally following must be true?
I. a × a−1 × b = b
saved?
II. a × b−1 = −ab
(A) $1,000 III. a × a −1 = −a2
(B) $900 (A) None of the above
(C) $800 (B) I only
(D) $600 (C) II only
(E) $500 (D) III only
(E) II and III only
15. If n is a positive multiple of five
and is less than 50, and a is an even 17. In a set of six consecutive integers,
number greater than or equal to 100, the sum of the three smallest
which of the following expressions integers is s. In terms of s, what is
CANNOT be an integer? the sum of the three greatest integers
in the set?
n
(A) (A) s+3
a
(B) s+6
a
(B) (C) s+9
n (D) 3s + 6
a (E) 3s + 9
(C)
2n
2a 17. The score for a certain exam is
(D) determined by awarding 3 points for
n
every correct answer and subtracting
n2
(E) 1 point for every incorrect answer.
a How many questions did a student
1
answer correctly if she answered all
3n 3 of the 93 questions on the exam and
15. If −
2 is an integer, which of the her final score was 247 ?
2n 3
(A) 71
following could be the value of n ?
(B) 77
I. 1 (C) 82
II. 2 (D) 85
III. 3 (E) 90
(A) None of the above
(B) I only
(C) II only
(D) I and II only
(E) I, II, and III

© The Princeton Review, Inc. | 57

07 Math HW 57 11/15/05 3:29:58 PM


Refresher Manual for the SAT

18. At a cost of t cents for 5 oranges, 19. If the local sales tax is 6%, and if t
how many oranges can be bought for is the price of an item after tax has
x dollars? been added, which of the following
expressions could be used to find the
price of the item before taxation?
500x
(A)
t
500t (A) 0.06t
(B)
x
(B) 0.94t
20x
(C) t
t (C)
0.94
20t
(D) (D) 1.06t
x
t
(E)
(E) 20tx 1.06

18. In sequence F, each term after the


first term is found by multiplying
the previous term by a. If the second
term is 10 and the fourth term is 2.5,
what is the value of a?

1
(A)
8
1
(B)
2

(C) 2

(D) 4

(E) 8

18. Andy had four times as many dollars


as Chris. When Andy gave Chris ten
dollars, he then had twice as many
dollars as Chris. How many dollars
did Andy have originally?
(A) 20
(B) 40
(C) 50
(D) 60
(E) 80

58 | © The Princeton Review, Inc.

07 Math HW 58 11/15/05 3:30:00 PM


Math Homework

19. If 0 < x < 1, which of the following 19. If the average (arithmetic mean) of
statements must be true? three numbers is 9, and two of the
numbers are greater than 11, which
of the following must be true?
(A) x2 < x < x
I. The greatest number is less than 27.
(B) x < x2 < x II. The least number is less than 5.
III. The second greatest number is 12.
(C) x < x < x2 (A) I only
(B) II only
(D) x < x < x2 (C) III only
(D) I and II only
(E) x2 < x <x (E) II and III only

© The Princeton Review, Inc. | 59

07 Math HW 59 11/15/05 3:30:01 PM


Refresher Manual for the SAT

GEOMETRY

1. What is the measure of a right angle?


_ ________________________________________________________________

2. What is the sum of the measures of the angles on one side of a straight line?
_ ________________________________________________________________

3. What is the sum of the measures of the angles in a triangle?


_ ________________________________________________________________

4. What is the sum of the measures of the angles in a quadrilateral?


_ ________________________________________________________________

5. What is the sum of the measures of the angles at the center of a circle?
_ ________________________________________________________________

6. What can you say about the measures of two angles across from each other
when two straight lines intersect?
_ ________________________________________________________________

7. What does it mean to bisect an angle or a line segment?


_ ________________________________________________________________

8. What does it mean for two lines to be perpendicular?


_ ________________________________________________________________

9. What does it mean for two lines to be parallel?


_ ________________________________________________________________

10. What can you say about the angle across from the longest side in a triangle?
_ ________________________________________________________________

11. What can you say about the angle across from the shortest side in a triangle?
_ ________________________________________________________________

60 | © The Princeton Review, Inc.

07 Math HW 60 11/15/05 3:30:01 PM


Math Homework

12. What can you say about the angles across from equal sides in a triangle?
_ ________________________________________________________________

13. Name two important facts about an isosceles triangle:


_ ________________________________________________________________

14. Name two important facts about an equilateral triangle:


_ ________________________________________________________________

15. Name three important facts about a parallelogram:


_ ________________________________________________________________

16. What makes a parallelogram a rectangle?


_ ________________________________________________________________

17. What makes a rectangle a square?


_ ________________________________________________________________

18. How does the diameter of a circle compare to the radius?


_ ________________________________________________________________

19. What is the formula for the area of a parallelogram (which also applies to a
rectangle or a square)?
_ ________________________________________________________________

20. What is the formula for the area of a triangle?


_ ________________________________________________________________

21. How do the base and height have to be related?


_ ________________________________________________________________

22. What is the formula for the area of a circle?


_ ________________________________________________________________

23. How can you find the perimeter of any polygon?


_ ________________________________________________________________

© The Princeton Review, Inc. | 61

07 Math HW 61 11/15/05 3:30:01 PM


Refresher Manual for the SAT

24. What is the formula for the circumference of a circle?


_ ________________________________________________________________

25. What is the formula for the Pythagorean theorem, and what do the variables
represent?
_ ________________________________________________________________

26. What are the ratios of the sides of ETS’s three favorite right triangles?
_ ________________________________________________________________

27. What is the formula for the volume of a rectangular box?


_ ________________________________________________________________

28. What is the formula for the slope of a line?


_ ________________________________________________________________

29. What is the ratio of the sides of a 45°-45°-90° triangle?


_ ________________________________________________________________

30. What is the ratio of the sides of a 30°-60°-90° triangle?


_ ________________________________________________________________

31. Where can you find a lot of this information during the test?
_ ________________________________________________________________

32. What should you do if a figure is not drawn to scale?


_ ________________________________________________________________

33. What should you do if no figure is provided for a geometry question?


_ ________________________________________________________________

34. What should you do if there are variables in the questions or answer choices?
_ ________________________________________________________________

35. What is Ballparking?


_ ________________________________________________________________

62 | © The Princeton Review, Inc.

07 Math HW 62 11/15/05 3:30:02 PM


Math Homework

Aƒ Bƒ
Xƒ Cƒ
Xƒ Xƒ Fƒ
Eƒ Aƒ

1. What is the value of x ? __________


a = 40, b = 30

4. What is the value of e + f ? __________

ƒ ƒ
Xƒ L

L

2. What is the value of x ? __________


l1 || l2

5. What is the value of x ? __________

ƒ

ƒ
 

3. What is the value of y ? __________ Yƒ

6. What is the value of y ? __________

© The Princeton Review, Inc. | 63

07 Math HW 63 11/15/05 3:30:03 PM


Refresher Manual for the SAT

 R
s

7. What is the area of the triangle? __________ 10. If the circumference is 12π, what is the area
of the circle? __________

  
s ƒ

11. What is the area of the shaded region?


8. What is the area of the triangle? __________
 __________

ƒ

ƒ
Xƒ Xƒ

9. What is the value of x ? __________

64 | © The Princeton Review, Inc.

07 Math HW 64 11/15/05 3:30:03 PM


Math Homework

Z 10. A line has a slope of 3 and contains


X  a point with coordinates (2, 1).
Z
 Which of the following coordinates
X also describes a point on that line?
))
)
(A) (0, –2)
Y Y (B) (1, 2)
 (C) (2, 3)
(D) (4, 1)
7. If the lengths of the sides of (E) (3, 4)
Triangles I and II above are as
shown, then the perimeter of
Triangle II is how many times the
perimeter of Triangle I ? 49˚
l

x˚ m
1
(A)
2
2 l % m
(B)
3
9
(C) 11. In the figure above, what is the value
8 of x + y ?
3
(D) (A) 119
2
(B) 121
9 (C) 129
(E)
2 (D) 131
(E) 139

Aƒ Cƒ

1
9. In the figure above, if a = b and
c = 30, then b = 2

(A) 30
(B) 45
(C) 50
(D) 60
(E) 100

© The Princeton Review, Inc. | 65

07 Math HW 65 11/15/05 3:30:05 PM


Refresher Manual for the SAT

" #


! % $

12. In the figure above, ABCD is a


rectangle. If BE = CE, what is the
value of y in terms of x ?
.OTE&IGURENOTDRAWNTOSCALE
x
(A)
2
11. The figure above represents a 180 − x
(B)
cross-section of a set of stacked 2
proportional cones. If the smallest
cone has a height of 12 inches and (C) 360 – 2x
a base diameter of 3 inches, and
the largest cone has a base radius of (D) x
3 inches, what is the height of the
largest cone, in inches?
(E) 2x
(A) 12
(B) 18 Y
(C) 24 " #
(D) 28 s s
n 
(E) 30
0
s 
12. What is the volume of a cube that s X
has a surface area of 96 ? /

(A) 24
(B) 32
(C) 48 !s s$
(D) 64 n n
(E) 72

12. In the figure above, if Q is a point


(not shown) on the perimeter of
ABCD such that segment PQ bisects
the square region ABCD, what are
the coordinates of Q ?
(A) (–8, –3)
(B) (–8, 3)
(C) (3, –8)
(D) (8, –3)
(E) (3, 8)

66 | © The Princeton Review, Inc.

07 Math HW 66 11/15/05 3:30:06 PM


Math Homework

13. Rectangle ABCD (not shown) has a 14. Points Q, R, S, and T lie on the same
perimeter of 20. If AB = 3, then what line, not necessarily in that order. If
is the area of ABCD ? QR = 5, RS = 7, and ST = 4, which
of the following could NOT be the
value of QT ?
(A) 2
(B) 6
(C) 8
(D) 10
(E) 16

14. Circle A has a radius of 2. If a square


is drawn with all four of its vertices
on Circle A, what is the area of the
square?
(A) 2
% & (B) 4
(C) 8
(D) 2π
(E) 4π

2
3

$ ' /

13. In the figure above, the smaller


square has its vertices at the 1
midpoints of the sides of square 0
DEFG. If EF = 6, what is the
perimeter of the smaller square?
14. In the figure above, points P, Q, R,
(A) 12 and S lie on the circumference of a
(B) 18 circle with center O. If the measure
(C) 24 of ∠ SOP = 100°, what is the
(D) 9 2 (approximately 12.73) measure of ∠ OPQ ?
(E) 12 2 (approximately 16.97) (A) 100°
(B) 80°
(C) 70°
(D) 60°
(E) 50°

© The Princeton Review, Inc. | 67

07 Math HW 67 11/15/05 3:30:08 PM


Refresher Manual for the SAT

! "

#
s/

ƒ
0 1
% $
Note: Figure not drawn to scale.

14. If the ratio of AC to CD is 1 to


3, then the area of  ABC is what 15. In the figure above, if O is the center
fraction of the area of CDE ? of the circle, then x =

1
(A)
9
1
(B)
4
1
(C)
3
1
(D)
2
4
(E)
3
14. A figure is formed by drawing
a rectangle and connecting the

midpoint of each side by line Xƒ
segments to the midpoint of every
other side. How many triangles are
formed in the resulting figure?
(A) 0 Yƒ Wƒ
(B) 4
(C) 8
(D) 12
(E) 16 15. In the figure above, (x – z) + (y – w) =

68 | © The Princeton Review, Inc.

07 Math HW 68 11/15/05 3:30:09 PM


Math Homework

L
s
L
ƒ Xƒ

s s ƒ

s
18. If l1 is parallel to l2 in the figure
above, what is the value of x ?
16. In the figure above, the four quarter-
circles share only the midpoints of
the sides of the square. If the length
of a side of the square is x, what
is the area of the shaded region, in
terms of x ?
π
(A) x 2 (1 − )
4
π
(B) x 2 (1 − )
2
(C) x 2 (π − 2 )

(D) x( 4 − π )

(E) 2 x(π − 2 ) 4

17. In the coordinate plane, rectangle


ABCD has vertices A(−2, 6),
B (f, 6), C (f, −2), and D (−2, −2). If 
the perimeter of ABCD is 38, which X
of the following could be the value
of f ? 5

(A) 8
(B) 9 
(C) 11 3

(D) 13 6
(E) 15

18. In the figure above, x =


(A) 7
(B) 6
(C) 4 2
(D) 30
(E) 5

© The Princeton Review, Inc. | 69

07 Math HW 69 11/15/05 3:30:12 PM


Refresher Manual for the SAT

Y 19. An isosceles triangle has a side of


length 9 and its perimeter is less
than 27. If the lengths of the sides of
the triangle are integers, what is the
"   greatest possible difference between
! the lengths of any two sides?
s X
  #

19. In the figure above, if right triangle


ABC is rotated about the origin
counter-clockwise until point B lies
on the positive y-axis, what will be
the new coordinates of point B ?
(A) (0, 1)

(B) ( 3 , 0)

(C) (0, 2)

(D) (0, 3)

(E) (0, 5)

3
19. If line m has a slope of – and
8
contains the points with coordinates
(–1, –2) and (x, –8), what is the
value of x ?

70 | © The Princeton Review, Inc.

07 Math HW 70 11/15/05 3:30:13 PM


Math Homework

OTHER APPROACHES

Average (Arithmetic Mean), Median & Mode

S = {14, 17, 12, 3, 4, 7, 20} 10. If 26 is the average (arithmetic


mean) of 33, 17, and x, what is the
5. What is the difference between the value of x ?
median of set S and the average (A) 50
(arithmetic mean) of set S ? (B) 28
(A) 1 (C) 26
(B) 2 (D) 16
(C) 3 (E) 8
(D) 4
(E) 5

10. If 13 is the average (arithmetic


mean) of 22, 12, y, y, y, and y, what
6. The average (arithmetic mean) is the value of y ?
of three numbers is 8. If the third (A) 11
number is 10, then the sum of the (B) 13
first two numbers is (C) 16
(A) 16 (D) 22
(B) 14 (E) 44
(C) 12
(D) 10
(E) 7

11. Nine members of a baseball team


have batting averages of .280, .310,
.200, .280, .270, .280, .240, .270, and
8. If the average (arithmetic mean) of .200, respectively. What is the mode
21, 12, 37 and y is 20, what is the of these batting averages?
value of y ? (A) .200
(A) 10 (B) .240
(B) 16 (C) .250
(C) 20 (D) .255
(D) 22.5 (E) .280
(E) 40

© The Princeton Review, Inc. | 71

07 Math HW 71 11/15/05 3:30:13 PM


Refresher Manual for the SAT

12. If the sum of 4, 7, 19, x, and y is


60, what is the average (arithmetic 1 1
16. If x = y and x = z , then the
mean) of x and y ? 2 3
(A) 12 average (arithmetic mean) of x, y,
(B) 15
(C) 20 and z in terms of z is
(D) 24
(E) 30
(A) 3z
13. If the sum of 5, 17, 18, x, and y is 60,
what is the average of x and y ? (B) 2z

2
(C) z
3
1
(D) z
3
1
(E) z
6

18. A group of 5 students whose ages


are 9, 7, 8, 13, and 17 are riding a
bus, which then stops to pick up a
sixth student. If the median age of
14. At the end of the season, the coach the new group is 10, what must be
of a basketball team analyzed the the age of the sixth student?
number of points the team scored
(A) 5
in each game. If the median of
(B) 6
the scores was 54 points, and the
(C) 8
mode was 48 points, which of the
(D) 10
following must be true? (E) 11
(A) There was at least one game in
which the team scored exactly 54
points.
(B) The average (arithmetic mean) of
the scores was less than 54 points.
(C) The score that occurred the most
frequently was 48 points.
(D) The number of games in which
the team scored less than 48
points was equal to the number of
games in which the team scored
more than 48 points.
(E) There were as many games in
which the team scored 60 points
as there were in which the team
scored 48 points.

72 | © The Princeton Review, Inc.

07 Math HW 72 11/15/05 3:30:15 PM


Math Homework

Percents 13. To graduate, Jill must take a certain


number of credits. If Jill took 25%
of the required credits her first year
6. What is 5% of 20% of 300 ? at school, and 40% of the remainder
the following year, what percent of
(A) 0.3
the required credits does she have
(B) 3
left to complete?
(C) 30
(D) 300 (A) 15%
(E) 3,000 (B) 35%
(C) 45%
10. The manager of a clothing store (D) 65%
decreases the price of a blouse 20%. (E) 75%
If the original price of the blouse
was $23.00, what is the new price? 14. During a special promotion, a shoe
store offers customers a 30 percent
(A) $27.60
discount on a pair of shoes when
(B) $18.40
they purchase a pair of boots at full
(C) $18.30
price. If Julia buys a pair of shoes
(D) $17.40
(E) $4.60 that originally cost $20 and a pair of
boots that has a full price of $40, by
what percent is the cost of her total
11. If 3 of a dealership’s 24 cars
purchase discounted?
were sold, what percent of the
dealership’s cars remained unsold? (A) 3%
(B) 10%
(A) 95% (C) 20%
(B) 87.5% (D) 30%
(C) 27% (E) 90%
(D) 21%
(E) 12.5%
4
15. If m is of n, then n is what
3
percent of m ?
(A) 133%
(B) 120%
(C) 90%
(D) 75%
(E) 33%

© The Princeton Review, Inc. | 73

07 Math HW 73 11/15/05 3:30:15 PM


Refresher Manual for the SAT

16. The price of a book is increased 14. Vikram lives in a state where the
by 10 percent and the new price sales tax is 7%. He purchases a piece
is then increased by an additional of exercise equipment and pays
10 percent. The two increases are $56.00 in sales tax. What was the
equivalent to a single increase of price of the equipment in dollars, not
including sales tax? (Disregard the
(A) 1%
dollar sign when gridding in your
(B) 11%
answer.)
(C) 20%
(D) 21%
(E) 100%

17. If a television set cost $350 in 1980


and $385 in 1990, by what percent
did the cost increase from 1980 to
1990 ?
(A) 3.5%
(B) 10%
(C) 35%
(D) 65%
(E) 85%

74 | © The Princeton Review, Inc.

07 Math HW 74 11/15/05 3:30:15 PM


Math Homework

Ratios 14. A line 120 meters long is divided


into two portions in a ratio of
7 1 : 5. The longer portion is how
1. If the ratio of a to b is , then the many meters longer than the shorter
ratio of 2a to b is 3
portion?
(A) 30
7 (B) 40
(A)
6 (C) 60
(D) 80
(B) 2 (E) 100

7
(C) Red White
3
Marbles Marbles
(D) 3 Jar 1 1 2

14 Jar 2 5 7
(E)
3 Jar 3 3 6
2. In a certain game, twelve players Jar 4 2 3
form a team. If a team must have at Jar 5 3 4
least one male player for every three
female players, what is the minimum
number of male players on any 16. According to the chart above, in
team? which jar is the ratio of red marbles
to white marbles the greatest?
(A) 1
(B) 2 (A) Jar 1
(C) 3 (B) Jar 2
(D) 4 (C) Jar 3
(E) 5 (D) Jar 4
(E) Jar 5

13. The recipe for Jim’s Punch calls for

6 cups of orange juice, 5 cups of


1
soda, and 3 cups of sherbet, and
2
serves 12 guests. If a bowl of punch
3
made in this proportion contains 8
4
cups of soda, how many guests will

it serve?
(A) 18
(B) 20
(C) 21
(D) 23
(E) 24

© The Princeton Review, Inc. | 75

07 Math HW 75 11/15/05 3:30:17 PM


Refresher Manual for the SAT

15. If a mixture requires 5 times as 18. A teacher spent $42 on books.


1 Hardcover books cost $4 each and
much charcoal as sulfur, and as
2 paperback books cost $2 each. If the
much saltpeter as sulfur, then what teacher purchased three hardcover
books for every paperback book,
fractional part of the mixture is how many books did the teacher
purchase all together?
sulfur?
(A) Three
(B) Seven
(C) Nine
(D) Twelve
(E) Fourteen

15. In a certain school, there are 4 boys


for every 5 girls, and 1 teacher for
every 4 boys. If there are 440 people
in the school, how many are boys?

76 | © The Princeton Review, Inc.

07 Math HW 76 11/15/05 3:30:17 PM


Math Homework

Proportions

3. If the ratio of 3 to 5 is the same as 13. A typist types at a constant rate of


the ratio of 12 to x, what is the value 155 words every 5 minutes. After 4
of x ? minutes, how many words has he
typed?
(A) 6
(B) 10
(C) 15
(D) 20
(E) 24

4. In a restaurant where the sales tax on


a $4.00 lunch is $0.24, what will be
the sales tax on a $15.00 dinner?
(A) $0.60
(B) $0.75
(C) $0.90
(D) $1.20
(E) $1.74
14. A race car completes a 450-mile
course in 3 hours. Another race car,
traveling at the same speed, would
take how many minutes to complete
a 90-mile course?

© The Princeton Review, Inc. | 77

07 Math HW 77 11/15/05 3:30:18 PM


Refresher Manual for the SAT

Graphs

9. In the xy-coordinate system, (q, 0) Y


is one of the points of intersection

of the graphs of y = x 2 − 16 , and 

1
 s
y = x 2 − 4 . If q is negative, what 

4  s
is the value of q ?


s


(A) –4


s
(B) –8 

s
(C) –16 X
           
(D) –32
(E) –64
13. The graph of y = g(x) is shown above.
If g(4) = d, which of the following
12. The graph of a linear function in
could be the value of g(d) ?
the xy coordinate plane is given by
the equation 2 y = 3 x + 7. Which of (A) 6
the following is the equation of the
reflection of this graph across the
y-axis? (B) 7

(A) 2 y = −3 x + 7 2
(C) 7
(B) 2 y = 3 x − 7 3
1
(C) 2 y = −3 x − 7 (D) 8
3
(D) 3 y = 2 x + 7
2
(E) 10
(E) 3 y = −2 x + 7 3

78 | © The Princeton Review, Inc.

07 Math HW 78 11/15/05 3:30:21 PM


Math Homework

Y Y

"n A # A

s /
  X
X

!n nA $ nA

14. The graph of y = f ( x ) is shown Note: Figure not drawn to scale.


in the figure above. Which of the
following could be the graph of
y = f ( x + 5) − 2 ? 18. In the figure above, ABCD is a
rectangle. Points A and D lie on
Y
the graph of y = –cx4, where c is a
constant. If the area of ABCD is 128,
what is the value of c ?
(A) /  
X

s
 

(B) /
X

(C) /
s
 
X

s
 

(D) /
X

s  

(E) /
X

© The Princeton Review, Inc. | 79

07 Math HW 79 11/15/05 3:30:22 PM


07 Math HW 80 11/15/05 3:30:22 PM
Critical Reading
Introduction

Easy 1 1
2 Easy 2
Medium
Sentence 3 3
4 Sentence 4
Completions Difficult
5 Completions Medium 5
6 6
Short 7 7
Difficult
Reading 8 8
9 9
10 Short 10
11 Reading 11
12 12 Easy 1
13 13 Sentence 2
3
Medium
14 14 Completions 4
5
15 15 Difficult 6
Long 16 16 7
8
Reading 17 Long 17 9
18 18 10
Reading 11
19 19 Long 12
20 20 Reading 13
14
21 21 15
22 22 16
23 23 17
18
24 24 19

24 Questions 24 Questions 19 Questions


25-Minute Section 25-Minute Section 20-Minute Section

As you know, the Critical Reading section of the SAT is made up of three types of
questions:

• Sentence Completions
• Short Passage-Based Reading
• Long Passage-Based Reading

© The Princeton Review, Inc. | 81

08 Crit Read Intro 81 11/17/05 2:47:19 PM


Refresher Manual for the SAT

Critical Reading passages can vary in length from 100-word short passages to
950-word long dual passages, and come in four formats: single short, dual short,
single long, and dual long. Short passages aren’t “easier”; they are simply shorter.

PROCESS OF ELIMINATION (POE)


Remember, reading questions don’t really have a “right” answer, they have a “best”
answer. To find the best answer, begin by eliminating the worst answers. Always
consider every answer choice! Remember: if you’re not sure of the meaning of a
word in an answer choice, don’t eliminate that choice.

Be aggressive; with our techniques, you’ll be able to eliminate a few answer


choices on almost every question. And as long as you can eliminate at least one an-
swer choice, guess from among the remaining choices—the odds are in your favor.

PACING
Most people think that if they are unable to finish a test, then they’ve really messed
up. This is not true on the SAT. It is much more important to go at a pace at which
you can answer questions correctly than it is to try to finish the test.

So attempt this many questions


To get: You need to 23–25 23–25 Total # of
19-question
(scaled earn: (raw question question questions to
section
score) points) section section attempt
300 5 6 6 3 15
350 9 8 8 4 20
400 14 11 11 8 30
450 21 15 15 12 42
500 29 20 20 14 54
550 38 23 23 18 64
600 46 all all all 67
650 53 all all all 67
700 59 all 24 all 67
750 63 all all all 67
800 67 all all all 67

After taking four or more diagnostic tests, you should have a good idea of what
your goal score should be. Look at the pacing chart and memorize the number of
questions you need to answer to get the score you want. You should use all of your
time to work on those questions rather than struggling to work all the questions
and making careless errors or worse, sitting around with ten minutes left at the
end of every section.

82 | © The Princeton Review, Inc.

08 Crit Read Intro 82 11/17/05 2:47:19 PM


Critical Reading Introduction

Take more time per question and get more questions right.

a Pood reMinder
Remember that order of difficulty pretty much goes out the window on the Critical
Reading section. You control which questions you answer, and the order in which
you answer them. Go through the section looking for questions that look easier for
you.

You’ll notice that the pacing chart tells you how many questions to do in each
section, but doesn’t tell you how many of each question type to do. That’s because
different people are better at different kinds of questions. If you have a strong
vocabulary, you may want to do more sentence completions. If you’ve developed
a knack for quickly fi guring out the main idea of a long passage, you may want to
concentrate on the long reading questions. By looking at your score report, you
and your teacher can fi gure out which kinds of questions you’re best at, and devise
a plan that suits you.

READING
Nothing will improve your score on the Critical Reading section of the SAT as much
as developing the ability to determine quickly what a piece of writing is trying to
say. The more practice you get, the better off you’ll be. Choose material whose
meaning isn’t evident at a glance and ask yourself:

• What did that sentence actually say?


• What is the author trying to get at in this paragraph?
• What is the main idea of this page?

VOCABULARY
The one thing that can make any Critical Reading question difficult, whether it be
a sentence completion or a reading question, is not knowing all the words in the
question or the answer choices. If you haven’t learned all of the words in the Hit
Parade, you’re putting yourself at a disadvantage. Even if you know all of our tech-
niques, you won’t have much success if you don’t know many of the words.

The SAT tests the same words year after year, so you don’t have to memorize
the entire dictionary. If you come across any unfamiliar words in your manual, di-
agnostic tests, 11 Practice Tests book, or other practice materials, learn them.
It’s very likely that you’ll see some of them on the real test. If you’ve learned all of
these words and want more, your teacher can give you a longer vocabulary list—
just ask.

© The Princeton Review, Inc. | 83

08 Crit Read Intro 83 11/17/05 2:47:20 PM


08 Crit Read Intro 84 11/17/05 2:47:20 PM
ReaDing
CoMpRehension

The ChoiCeS

The “best” answer on a reading question will not:

• offend anyone
• violate common sense
• use extreme language
• use deceptive language
• require outside knowledge of the topic
• suggest an extreme tone

11. According to the passage, the role played by Ameri-


can farmers in U.S. history has been
(A) generally insignificant with a few excep-
tions during the nineteenth century
(B) somewhat overlooked and deserving of
further recognition by historians
(C) difficult to discern from existing records
(D) more important than that of any other group
(E) different in several respects from the roles
played by European farmers

© The Princeton Review, Inc. | 85

09 Reading Comp 85 11/15/05 3:30:31 PM


Refresher Manual for the SAT

12. The author characterizes the great wave of European


immigrants in the late nineteenth century as
(A) resulting from various religious, political
and economic factors
(B) a burden on American resources from
which the United States has yet to recover
(C) an event of only short-term importance to
the nation
(D) identical in every respect to the immigra-
tions of the early twentieth century
(E) a boon to American industrial development

13. The author’s attitude toward Thomas Jefferson’s


presidency can best be described as
(A) jubilant
(B) admiring
(C) indifferent
(D) critical
(E) embittered

The questions
You always want to know what you’re looking for before you go to the passage. Are
you just fetching information, or are you reading between the lines? Translate
each of the questions below so you know exactly what you are being asked to do.

1. The author’s reaction to the photograph can best be


described as

2. The mention of the traffic light serves to

3. In the third paragraph (lines 12-16), the author sug-


gests that

86 | © The Princeton Review, Inc.

09 Reading Comp 86 11/15/05 3:30:31 PM


Reading Comprehension

4. On the morning described in the passage, Paul per-


forms all of the following actions EXCEPT

5. Which of the following, if true, would most weaken


the surveyor’s claim about the Albert Hall (lines 30-
31)?

Review : Question Types


Questions fall into two main types: fetch information questions and reasoning
questions that ask you to read between the lines. The former require are pretty
straightforward, while the latter require more work interpreting the author’s in-
tent. All require you to paraphrase your own answer before going to the answers.

Here’s how to approach the most common tasks in Critical Reading:

Specific Detail
Find the information the question is asking for. The answer will be nothing more
than a rewording of something in the passage.

Specific Purpose
The best answer choice will be the one that explains why a particular part of the
passage was included.

Primary Purpose
Choose the answer choice that best describes what the passage is doing.

Main Idea
Don’t get lost in details. Find the answer choice that summarizes the entire pas-
sage, not just a piece of it. Of course, don’t pick an answer choice that goes beyond
the passage, either.

Tone
Eliminate choices that are too extreme, then choose the best fit from the choices
that remain.

© The Princeton Review, Inc. | 87

09 Reading Comp 87 11/15/05 3:30:32 PM


Refresher Manual for the SAT

Vocab in Context
Treat these like sentence completions. Remember, if the word looks easy, the pas-
sage will be using it in an unusual way, so be on your guard.

Weaken/Strengthen
Figure out the main point of the argument, then find the answer choice that best
supports your side (with the author on strengthens, against the author on weak-
ens).

Inference
Pick the answer choice that sticks the closest to what must be true based on the
passage.

Analogous Reasoning
Paraphrase the relationship that the question is asking about, then eliminate any
answer choice that doesn’t share the same relationship.

Literary Terms
Make sure you’ve learned the definitions of these terms: simile, metaphor, personi-
fication, hyperbole, irony, and any others you’ve encountered in preparing for the
test. Use the context to determine what the author is trying to express through
the use of these devices.

Time Suck
For EXCEPT/NOT/LEAST questions, don’t look for what’s not there: if the problem
asks which one of the following five things isn’t in the passage, find the four that
are. On I/II/III questions, use the choices to help you avoid unnecessary work.

PARAPHRASING
Knowing that a particular line of the passage answers the question you’re work-
ing on doesn’t help if you don’t understand that line. Try to answer the question
in your own words before you go to the answer choices. For each of the excerpts
below, write a short paraphrase that shows what the passage is really saying.

1. The advent of chipboard, which is as cheap as par-


ticle board but equal in tensile strength and rigidity to
plywood, means that much of our home-construction
lumber will now come not from the giants of the for-
est but from replaceable saplings.

88 | © The Princeton Review, Inc.

09 Reading Comp 88 11/15/05 3:30:32 PM


Reading Comprehension

2. Architecture is doomed from the outset, as long as its


quest for modernity leads it toward globes, ellipses,
steep angles, and other “futuristic” shapes, and away
from the ideal shapes for human habitation: the rect-
angle and the cube.

3. The reversal of the trend toward mightier stonework,


thicker walls, and heavier armor was accomplished
by an agent far more subtle than an army of knights;
this change immediately followed the invention of
gunpowder.

4. The luminist school of American landscape painting


drenched the monumental vistas of the American
West in golden, surreal light, transforming already
striking scenes into glimpses of utopia.

5. Some argue that because Suppliants, in which the


chorus is the protagonist, was written later than Per-
sai and Septem, in which it is not, we can therefore
conclude that in early tragedy the chorus was passive,
its role limited to singing choral songs juxtaposed
with brief speeches; this notion may seem reasonable,
but in fact it is based on flawed logic.

© The Princeton Review, Inc. | 89

09 Reading Comp 89 11/15/05 3:30:32 PM


Refresher Manual for the SAT

The BASiC APPRoACh


1. Read the italicized blurb that comes before the passage (if there is one).

The GiST stands for the 2. Go straight to the questions. If you need to skim the passage to get your
General idea, Structure, bearings, do it quickly to get the GIST of the passage, but not for detail.
and Tone of the passage. 3. Read and translate the question. You can’t answer a question if you don’t
know what it’s really asking. Some ETS questions don’t even look like
questions, so make sure you know exactly what you’re looking for before
you move past this step.
4. Go to the passage and read what you need to fi nd the answer. There’s only
one way to be sure that the answer you’re choosing is the best answer:
You should be able to point to the place in the passage that proves it cor-
rect. If you can’t put your fi nger on the reason an answer is correct, it
probably isn’t.

Always go back to the passage to fi nd the information that answers the


question. It’s in there!

5. Answer the question in your own words. Jot down your answer so you
don’t waste effort reciting it to yourself as you read the answer choices.
6. Use POE to eliminate wrong answers.

Question 14 is based on the following passage.

Each flock of migratory birds flies at the altitude that gets


the birds the best speed while using the least amount of en-
ergy. The birds also take advantage of updrafts and tail winds.
Line Some take a different route south from their route north, in
5 order to take advantage of the prevailing winds. Birds’ sen-
sitivity to atmospheric pressure gives them an innate ability
to forecast the weather, and migratory birds do most of their
flying during weather favorable for flight. One leading orni-
thologist said that migratory birds “reach a level of expertise
10 flying that surpasses even the most skillful and experienced
in
human pilots.”

14. Which of the following statements can be inferred


from the above passage?
(A) Airplanes should fly at the same altitude as
birds do to in order to maximize efficiency.
(B) No mammal can predict weather as accu-
rately as can migrating birds.
(C) Some believe human pilots to be less adept
at flying than migratory birds.
(D) Migratory birds prefer to fly in the direction
from which the wind is blowing.
(E) Ornithologists study both migratory and
non-migratory birds.

90 | © The Princeton Review, Inc.

09 Reading Comp 90 11/15/05 3:30:32 PM


Reading Comprehension

Short Dual Passages


Short dual passages will appear at least once per test. There will be 1-2 questions
that refer to a single passage, and 2-3 that ask about the relationship between the
passages.

Treat these just as you would other reading passages—read the question first,
then go to the passages.

Questions 6-9 are based on the following passages.

Passage 1
Popular in the 1960’s, the Language Acquisition Device
theory stated that humans used a unique part of the brain that
was “hard-wired” to acquire the rules necessary to speak a
Line language. Evolutionary biologists were always skeptical, as
5 new brain structures typically take a longer time to evolve
than the time from which humans became a distinct species.
Furthermore, modern studies showed that, while certain parts
of the brain are used to learn languages, these areas of the
brain are not unique to humans. The use of American Sign
10 Language by chimpanzees was the final bombshell which de-
stroyed the old theory in the eyes of all but a few stalwarts.

Passage 2
The human vocal tract has evolved specifically for the
demands of speech. The arrangement of our larynxes, vo-
cal tracts, mouths, and pharynxes gives humans the ability to
15 produce a greater range of sounds than is possible for other
animals. This specialization for language, however, makes
other basic uses of these organs such breathing, swallowing,
and chewing difficult or even dangerous. Choking on food has
historically been a common cause of death in humans. The
20 evolutionary benefits of developing vocal language must have
been enormous in order to compensate for such a potentially
life-threatening drawback.

6. In Passage 1, the author’s main point about the Lan-


guage Acquisition Device theory is that
(A) it was a good model for studying the behav-
ior of chimpanzees
(B) nobody could have foreseen its quick de-
mise
(C) it proved the uniqueness of humans in the
evolutionary pyramid
(D) it was popular among evolutionary biolo-
gists
(E) it failed to hold up to rigorous scrutiny

© The Princeton Review, Inc. | 91

09 Reading Comp 91 11/15/05 3:30:33 PM


Refresher Manual for the SAT

7. Which of the following situations is most analogous


to the relationship of non-humans to vocal speech as
presented in the passages?
(A) An accomplished flute player attempts to
play a saxophone for the first time and
cannot produce a song she knows how to
play well on the flute.
(B) A Chinese girl who was born in Australia
and only spoken to in English by her par-
ents cannot speak Chinese when she visits
the country in adolescence.
(C) A Czech poet residing in the United States
faces difficulty in articulating his reflec-
tions in American English.
(D) A hobbyist who makes wooden models of
cars gathers all the parts needed to build
an engine and motor but cannot make the
car run in the configuration he has devised
for the equipment.
(E) With instruction, a human being can learn
many languages and dialects and therefore
communicate successfully in languages
other than his or her native tongue.

8. The author of Passage 2 mentions breathing, swal-


lowing and chewing in order to
(A) emphasize the danger of utilizing the vocal
tract for more than one process at a time
(B) imply that the ability to produce vocal commu-
nication is a basic necessity for humans
(C) highlight potential uses of the larynx and phar-
ynx that are more beneficial than is producing
verbal language
(D) demonstrate that unique parts of the body
developed to handle all the needs of humans
(E) refute the idea that it is impossible for animals
other than humans to produce vocal commu-
nication

9. Unlike the author of Passage 1, the author of Passage


2 does which of the following?
(A) Denies that human evolution could have led
to verbal speech
(B) Suggests one reason why verbal speech is
possible for humans, but not other animals
(C) States that the language-producing areas
of the brain are the same in humans and
animals
(D) Proves that humans are hard-wired to pro-
duce language
(E) Demonstrates that humans must choose to
utilize their vocal tract for only one activity
at a time

92 | © The Princeton Review, Inc.

09 Reading Comp 92 11/15/05 3:30:33 PM


Reading Comprehension

Long Passages
The chief difficulty posed by long passages is that it’s harder to find the informa-
tion you’re looking for. Resist the temptation to linger over the passage trying to
absorb every last detail; instead, use these techniques to find the relevant portion
of the passage quickly.

Line Reference
Go to the line the question indicates, then read from five lines before that line to
five lines after that line. The answer won’t be on the line the question directs you
to, but it’ll be nearby.

Lead Word
If there’s no line reference, hunt for a relevant word or phrase that will tell you that
you’re in the right area. Ask yourself what you’d type into a search engine if you
had one to help you.

Chronology
Don’t forget that the questions will appear in roughly the same order that their an-
swers appear in the passage. The exception is that general questions, such as main
idea and primary purpose, can appear anywhere; skip these when you encounter
them, and come back to them after you’ve finished the specific questions.

© The Princeton Review, Inc. | 93

09 Reading Comp 93 11/15/05 3:30:33 PM


Refresher Manual for the SAT

Questions 16-21 are based on the following passage.

The following passage discusses the poets Langston though each loved him jealously and possessively.
Hughes and Countee Cullen and their contributions But out of the trying experiences of his adolescence
to the Harlem Renaissance. 50 came the pensive interludes in which he conceived
“The Negro Speaks of Rivers.”
It would not be far-fetched to say that a poem by It might seem unlikely that two poets, so young
a black student, appearing in the De Witt Clinton and so profoundly different in temperament, could
High School (New York City) literary magazine in become the most striking voices of a cultural move-
Line January, 1921, was the first clear signal of the cultur- 55 ment, but Cullen and Hughes, already accomplished
5 al movement later known as the Harlem Renaissance. artists in their teens, were exceptional poets at the
“I Have A Rendezvous with Life (with Apologies center of a twentieth-century renaissance.
to Alan Seeger)” brought Countee Cullen to the at-
tention of the daily newspapers and became widely 16. The primary purpose of the passage is to
quoted in classrooms and even pulpits.
10 This poem was followed just six months later (A) describe two individuals whose
by the publication in The Crisis, the influential and poetry shared a common signifi-
widely read magazine of the NAACP, of a free-verse cance
poem, “The Negro Speaks of Rivers,” by another (B) contrast two viewpoints that
black youth, Langston Hughes. Hughes had graduat- attempt to evaluate a cultural
15 ed from high school in Cleveland a year earlier, and movement
“Rivers” had been written directly after that event. (C) link two apparently unrelated
“Rendezvous” struck New York like a lonesome events to a common cause
meteor, burned brightly for a short time, then faded. (D) discuss family life as an influence
“Rivers” touched down more like twilight itself. That on creativity
20 both were harbingers is now evident, and it is no (E) urge critical examination of
surprise that when the literary and cultural move- works by two young poets
ment of the Harlem Renaissance won attention three
years later, these two poets, though still unpublished 17. The author’s use of the words “meteor” (line
in book form, were the new stars that caused the eyes 18) and “twilight” (line 19) indicates the
25 of both black and white intellectuals to blink. If, as
Emily Dickinson wrote, “one clover and a bee/and (A) type of imagery used in each
reverie” is all it takes to make a prairie, an observer poem
might surmise that two such teenagers as Cullen and (B) loneliness felt by both Cullen and
Hughes could at least stir up a renaissance in the Hughes
30 right time and place. (C) imminent end of the Harlem
Even though the two had not known each other Renaissance
before they began to be noticed as part of the Harlem (D) author’s fascination with astrono-
Renaissance, only a few blocks separated them dur- my
ing Hughes’s freshman year at Columbia University. (E) different ways in which the two
35 In their personalities and backgrounds, as in their poems were received
attitudes toward life, there was little to suggest the
role in which they were about to be cast. An orphan,
Cullen was adopted by a childless couple and his
gratitude to his adoptive parents never ceased to
40 be a part of his adult personality, a fact reflected in
his poetry. Not even sad or tragic themes deprived
his lyrics of thankful overtones. In contrast to the
melancholy beginnings that brightened so abruptly
for Cullen, dilemmas clouded Hughes’s early years
45 and became more difficult as he matured. Blessed
with charisma and an instinct for tolerance, he was
thwarted by parents who could not bear each other,

94 | © The Princeton Review, Inc.

09 Reading Comp 94 11/15/05 3:30:33 PM


Reading Comprehension

18. The word “harbingers” is used by the author 20. The passage suggests that Cullen and
in line 20 to mean something that Hughes did not become acquainted with
each other until
(A) poses a threat
(B) defies authority (A) Hughes’s work was published in
(C) announces change The Crisis
(D) leads a rebellion (B) Hughes enrolled as a freshman at
(E) resists change Columbia University
(C) both were recognized as partici-
19. The author uses an image from pants in the Harlem Renaissance
Emily Dickinson’s poetry (lines 26- (D) Cullen’s poem was quoted in
27) primarily to suggest that newspapers and pulpits
(E) both published their work in book
(A) a capacity for daydreaming is es- form
sential to artists
(B) a small beginning may lead to a 21. The author describes the childhoods of
large result Cullen and Hughes in order to
(C) a cultural movement can be as
impressive as a natural wonder (A) trace the development of each
(D) an inspiration is useful only if it poet’s writing
has a practical outcome (B) emphasize the similar experiences
(E) an artist is inspired by nature of the two poets
(C) prove that adversity cannot stifle
artistic genius
(D) account for the differing outlooks
of the two poets
(E) show that the two poets were
destined to work together

© The Princeton Review, Inc. | 95

09 Reading Comp 95 11/15/05 3:30:34 PM


09 Reading Comp 96 11/15/05 3:30:34 PM
Sentence
Completions

Look Before You Leap


Fill in your own word or phrase for the each blank before looking at the available
answer choices.

1. Only since the nineteenth century have we come to


regard the wilderness as valuable and beautiful in
itself, not to be ------- but preserved.
(A) revered
(B) exploited
(C) depicted
(D) nurtured
(E) extolled

2. A moderate degree of insight is so ------- in human


interaction that we tend to take it for granted.
(A) dramatic
(B) common
(C) significant
(D) exaggerated
(E) hypothetical

© The Princeton Review, Inc. | 97

10 Sent Completions 97 11/15/05 3:30:37 PM


refresher Manual for the SaT

3. The architects agree that it would be ------- to recon-


struct the mansion exactly as it had been because the
few existing records provide only ------- information
about the structure.
(A) difficult . . unavailable
(B) efficient . . historical
(C) impossible . . sketchy
(D) important . . reclassified
(E) pointless . . detailed

THe CLue

The clue is a word or phrase that ETS gives you to help you anticipate the
word that will best fit in the blank. To help identify the clue, ask yourself:

• Who or what is the sentence talking about?


• What information does the sentence give you about that person or
thing?

1. Synthetic fuels and conventional fuels are chemically


alike and thus virtually -------.
(A) expendable
(B) dependable
(C) indefinable
(D) unobtainable
(E) interchangeable

2. Because Andy had always adhered to his principles in


the past, we were confident he would remain ------- in
supporting what he thought was right.
(A) isolated
(B) cautious
(C) steadfast
(D) agreeable
(E) perplexed

3. Sometimes, when one cannot make a decision by


logical reasoning and careful deliberation, one must
rely on ------- and ------- instead.
(A) analysis . . scrutiny
(B) gullibility . . intelligence
(C) intuition . . impulse
(D) compliance . . argument
(E) dissent . . curiosity

98 | © The Princeton Review, Inc.

10 Sent Completions 98 11/15/05 3:30:37 PM


Sentence Completions

TrIGGerS

Triggers help you decide whether the blank will be a word similar in mean-
ing or opposite in meaning to the clue.

5. Although jubilant following their team’s victory, the


fans were ------- the next day when word of the star’s
serious injury was made public.
(A) enlightened
(B) amused
(C) dejected
(D) elated
(E) provoked

5. Contrary to the impression of slave traders that


African religions were -------, the peoples from which
American slaves were drawn possessed intricate
systems of religious beliefs.
(A) simple
(B) blasphemous
(C) sophisticated
(D) universal
(E) discernible

6. Unlike her colleagues, who tended to be introverted


and relatively uncommunicative, Dr. Phillips was
------- and quite -------.
(A) modest . . reticent
(B) dismayed . . nervous
(C) humble . . egotistical
(D) affable . . articulate
(E) steadfast . . loyal

7. Lynn, like her studious sisters, approached her school


work with great ------- and was rewarded with excel-
lent grades.
(A) trepidation
(B) diligence
(C) urgency
(D) renown
(E) skepticism

© The Princeton Review, Inc. | 99

10 Sent Completions 99 11/15/05 3:30:38 PM


refresher Manual for the SaT

Remember, sentences often direct you to the clue with punctuation trig-
gers such as colons (:) and semicolons (;). These are signals that the au-
thor is going to restate something or give an example, which will usually
help you pinpoint the clue.

5. The partners of that successful advertising agency


rarely grant interviews; in fact, they are so ------- that
only a few of their clients ever meet them.
(A) unpopular
(B) irresolute
(C) compliant
(D) democratic
(E) reclusive

TWo BLaNkS = TWICe aS eaSY


Two-blank sentence completions are easier if you work on one blank at a time.

5. It became obvious that if its opponents were to -------


the proposed legislation, their strategy would have to
be -------.
(A) justify . . implemented
(B) promote . . concealed
(C) amend . . impaired
(D) defeat . . revised
(E) repeal . . foiled

5. Some industries appear to ------- new techniques time


and time again, while others seem almost to shun
-------.
(A) forge . . decadence
(B) stymie . . creativity
(C) mobilize . . stagnation
(D) stifle . . modification
(E) generate . . innovation

7. The idea that most women stayed at home until the


feminist movement of the 1960’s and 1970’s is alto-
gether -------; in fact, throughout history women have
------- outside as well as inside the home.
(A) sacred . . contributed
(B) unethical . . investigated
(C) untrue . . attributed
(D) erroneous . . labored
(E) proven . . deplored

100 | © The Princeton Review, Inc.

10 Sent Completions 100 11/15/05 3:30:38 PM


Sentence Completions

RELATIONSHIP BETWEEN THE BLANKS


If the clue in a two-blank question has been blanked out, determine whether the
words in the blanks should be similar or opposite in meaning.

4. Since the judge has a reputation for being -------,


everyone was astonished when she imposed a -------
sentence on the convicted man.
(A) clement . . moderate
(B) lenient . . severe
(C) hostile . . harsh
(D) authoritative . . stringent
(E) austere . . strict

DRILL

1. Although the painter’s high-paying advertising work


brought her a great deal of -------, it did little to
increase her ------- as an artist.
(A) wealth . . reputation
(B) satisfaction . . generosity
(C) money . . obscurity
(D) uncertainty . . respect
(E) doubt . . experience

2. In the past, Hispanic novelists were rarely given


the recognition they deserved; today, however, the
literary world has come to regard these writers with
increased -------.
(A) intelligence
(B) appreciation
(C) complexity
(D) uniformity
(E) suspicion

3. Homelessness in America is no longer solely


------- issue; indeed, there are increasing numbers of
homeless people in our small towns and rural areas.
(A) an urban
(B) a peripheral
(C) a multifaceted
(D) an imperative
(E) an inconsequential

© The Princeton Review, Inc. | 101

10 Sent Completions 101 11/15/05 3:30:39 PM


Refresher Manual for the SAT

4. The sculptor was ------- at the lack of illumination in


the gallery, so she brought in her own lighting equip-
ment to ------- the situation.
(A) unconcerned . . amend
(B) disconcerted . . worsen
(C) dismayed . . ameliorate
(D) enchanted . . alter
(E) affronted . . contaminate

5. Before documenting their peaceful behavior, Dian


Fossey had subscribed to the common belief that
mountain gorillas were ------- animals.
(A) tranquil
(B) malevolent
(C) adaptable
(D) unresponsive
(E) aggressive

6. The dread of being labeled ------- if we do not unhesi-


tatingly embrace new technologies prevents us from
disregarding new computer hardware that has no
advantages, but is simply -------.
(A) progressive . . attractive
(B) antediluvian . . superior
(C) enlightened . . unique
(D) reactionary . . novel
(E) defensive . . archaic

7. Stubborn to the last, the maverick physicist continued


to ------- her theory long after the rest of the scien-
tific community had rejected it as inconsistent with
experimental evidence.
(A) revise
(B) expand
(C) espouse
(D) moderate
(E) discard

8. The emotional depth that so impressed readers of


Susannah Kaysen’s first book is missing from her
second novel, which exhibits ------- that approaches
superficiality.
(A) a melancholy
(B) a glibness
(C) an intensity
(D) an optimism
(E) an inventiveness

102 | © The Princeton Review, Inc.

10 Sent Completions 102 11/15/05 3:30:39 PM


Critical Reading
Homework

Passage-Based Reading

ETS Comprehension
Use what you know about what makes for a “best” ETS answer to eliminate choices
that would almost never be correct.

21. The author is primarily concerned with which


of the following?
(A) Belittling the UN philosophy of
limited internationalism
(B) Condemning the United States for
its ungrateful response to the UN
(C) Implying that the hypocrisy of UN
officials will be responsible for the
downfall of the organization
(D) Arguing for reduced United States
commitment in the UN and a con-
comitant reduction of the nation’s
influence in the UN
(E) Deploring problems facing the UN
and urging renewal of the organi-
zation

© The Princeton Review, Inc. | 103

11 Crit Read 103 11/15/05 3:30:42 PM


Refresher Manual for the SAT

26. The author is mainly concerned with the


(A) eliminating of all standards in the
teaching of English
(B) distortion of language and what
might be done about it
(C) determined effort some people
make to destroy their own lan-
guage
(D) difference between everyday
speech and expository writing
(E) emerging need for government
regulation of writing

104 | © The Princeton Review, Inc.

11 Crit Read 104 11/15/05 3:30:42 PM


Critical Reading Homework

Text Comprehension
Translate each of the following excerpts into your
own words:

1. While the Africanized bees are certainly


more defensive and more prone to follow
intruders a great distance than are their
European counterparts, in only very rare
instances is anyone seriously injured in an
attack by these so-called “killer bees.”

2. The story of the Confederacy is filled with


dramatic moments, but to the thoughtful
observer few are more dramatic than the con-
junction of these three men in the inaugura-
tion of the Confederate president. Beneath a
surface of apparent unanimity they carried,
like concealed weapons, points of view that
were in deadly antagonism. This antagonism
had not revealed itself hitherto. But it was
destined to reveal itself almost immediately.

3. But Edinburgh pays cruelly for her high seat


in one of the vilest climates under heaven.
She is liable to be beaten upon by all the
winds that blow, to be drenched with rain, to
be buried in cold sea fogs out of the east, and
powdered with the snow as it comes flying
southward from the Highland hills.

4. When a bird is in motion its wings (except when


flapping) are extended in a straight line at right
angles to its body. This brings a sharp, thin
edge against the air, offering the least possible
surface for resistance, while at the same time
a broad surface for support is afforded by the
flat underside of the wings. The same thing is
identically done in the construction of the flying
machine.

5. Values have shrunken to fantastic levels: taxes


have risen, our ability to pay has fallen, govern-
ment of all kinds is faced by serious curtail-
ment of income, the means of exchange are
frozen in the currents of trade, the withered
leaves of industrial enterprise lie on every side,
farmers find no markets for their produce, the
savings of many years in thousands of families
are gone.
© The Princeton Review, Inc. | 105

11 Crit Read 105 11/15/05 3:30:42 PM


Refresher Manual for the SAT

Short Passage-BASED READING


Questions 12-13 are based on the following Questions 14-15 are based on the following
passage. passage.

Located off the coast of Papua New Guinea, the While we tend to think of pizza as the ultimate
Trobriand Islands have intrigued anthropologists for Italian food, it is in fact a multicultural and, indeed,
nearly a century. Anthropologists are especially inter- multi-continental collaboration that has evolved over
Line ested in the Kula exchange system in which virtually Line the centuries since its invention. The earliest versions
5 every male Trobriander participates. Kula involves 5 of pizza were probably eaten in ancient Greece where
a very intricate process whereby the men exchange a flat piece of bread served as an “edible plate” for
necklaces made of the most exquisite shells with various toppings and relishes. While the Greeks con-
friends on other islands for armbands made of compa- tributed the idea of toppings, the base was improved
rably valued shells. Finding shells requires a lifetime by the nearby Etruscans’ method of baking oil-basted
10 of dedication and patience; as a man establishes more 10 dough on the hearth of the fireplace. A final ingredi-
friendships on distant islands within the island chain, ent, tomato sauce, required quite a wait; the tomato is
he receives and redistributes shells of great variety, indigenous to America and didn’t even reach Europe
thus enhancing his reputation. until the 1500’s.

12. Which of the following can be inferred from 14. The passage seeks to prove that pizza
the passage?
(A) has been falsely claimed as a
(A) The Trobriand Islanders have been national dish by the Italians
exchanging shells for nearly a (B) was invented in the 16th century
century. after tomatoes had been discov-
(B) Shell exchange forms the founda- ered
tion of Trobriand culture. (C) was actually invented by the Etrus-
(C) The value of shells is a function of cans
how difficult they are to find. (D) is derived from a number of differ-
(D) A male Trobriander who did not ent backgrounds and places
participate in Kula would be an (E) was a favorite dish of the early
outcast. Greeks
(E) Papua New Guinea is not land-
locked. 15. Which of the following would most support
the author’s conclusion?
13. The author would be most likely to agree
(A) A previously unknown civilization
with all of the following EXCEPT
with no ties to the outside world
(A) Male Trobrianders can develop eats a dish similar to pizza.
their reputations based upon (B) Pizza is the most popular dish
participation in the Kula system among half of the world’s popu-
(B) The Kula system is found on more lation.
than one of the Trobriand Islands (C) There were many differences
(C) Women will never be able to between the “edible plate” of the
participate in the Kula system Greeks and today’s pizza.
(D) The shells received through the (D) The “deep-dish” style of pizza
Kula system can be fashioned originated in restaurants in Chi-
into more than one form of cago.
decorative jewelry (E) Pizza’s popularity has remained
(E) It is acceptable within the Kula high even though it is high in fat
system to give shells that you and carbohydrates.
have received to someone else

106 | © The Princeton Review, Inc.

11 Crit Read 106 11/15/05 3:30:43 PM


Critical Reading Homework

Questions 16-17 are based on the following Questions 18-19 are based on the following
passage. passage.

It is commonly acknowledged that human habita- Imagine yourself as a backyard astronomer gaz-
tion necessarily has a negative effect on the surround- ing up into the sky. Mars, an angry red light in the
ing biome. Take, for instance, the San Joaquin kit fox. darkness, returns your stare. What would you make
Line As their territory has increasingly been populated by Line of this strange crimson object? To early civilizations,
5 strip malls and housing developments, kit fox popula- 5 especially the Greeks and Romans, this dusky red
tions have seen their numbers drop. But at the same point of light betokened the tides of war. It seems
time, it appears that the foxes may actually benefit only natural that some cultures would associate the
somewhat from human relations. Kit foxes find aban- hue of the planet with the passion and bloodshed of
doned construction pipes and storm-water storage battle. Of course, scientists now know that the planet’s
10 sumps to be suitable replacements for their lost ter- 10 ruddy complexion results not from any ill will toward
ritory, and the young have even been spotted playing man but from the large quantity of iron oxide on the
with abandoned golf balls and paintbrushes. Thus it planet’s surface.
appears that humans and animals may be able to cre-
ate new, mutually beneficial, economies. 18. The author uses personification in describing
the appearance of Mars (line 3 and line 10) in
16. Which of the following is the best description order to
of how the passage is structured?
(A) explain why the surface of Mars is
(A) A theory is outlined, a counterex- red
ample is presented, and the theory (B) provide additional contrast
is disproved. between the mythological and
(B) A principle is presented, a specific scientific images of Mars
example is evaluated, and a dif- (C) justify why the Greeks and Ro-
fering interpretation is proposed. mans considered Mars the god of
(C) A predicament is described, com- war
mon solutions are discussed, and (D) enable to the reader to imagine the
new measures are suggested. exact hue of the planet
(D) A stereotype is attacked, its param- (E) support the notion that Mars’ pres-
eters are defined, and a rebuttal is ence in the sky is threatening
articulated.
(E) A common belief is introduced, 19. In line 6, “betokened” most nearly means
then quantified, then miscon-
strued. (A) controlled
(B) resisted
17. In line 14, “economies” most nearly means (C) foretold
(D) reflected
(A) preserved resources (E) bemoaned
(B) functional arrangements
(C) efficient uses
(D) ecological concerns
(E) negotiated transactions

© The Princeton Review, Inc. | 107

11 Crit Read 107 11/15/05 3:30:43 PM


Refresher Manual for the SAT

Questions 6-9 are based on the following 7. The author of Passage 2 would most likely
passages. respond to Passage 1 by

Passage 1 (A) agreeing with the author of Pas-


sage 1 regarding the main theme
Of the five main themes in the ancient Epic of of Gilgamesh
Gilgamesh, Man’s quest for immortality is the most (B) expressing skepticism that the
prominent, as is evident from the primary thrust meaning of Gilgamesh can be
Line of the narrative. After his friend Enkidu is killed, reduced to a single plot device
5 Gilgamesh, fearing his own death, leaves his kingdom (C) criticizing the author of Passage 1
to search for the secret of immortality. The culmina- for misinterpreting the meaning
tion of the epic deals with this journey. A seemingly of Gilgamesh’s journey
unrelated episode in Tablet XII of the epic reinforces (D) arguing that there are more than
the point that even if there is no eternal life, a sort of the five themes mentioned in Pas-
10 permanence can be achieved through the memory of sage 1 present in Gilgamesh
one’s descendants. (E) claiming that the historical sig-
nificance of Gilgamesh is more
Passage 2 important than its significance as
literature
To see for ourselves the meaning of a story, we
need to look carefully at what happens in the story, at 8. Both passages imply that the Epic of
a character’s actions, and at the implications of their Gilgamesh
15 consequences. But we need to consider, too, how a
story is put together—how it uses the conventions of (A) is very old
language, of events with beginnings and endings, of (B) was originally an oral history
description, and of character. We read Gilgamesh, before it was written down
four thousand years after it was written, in part be- (C) is based on a real person
20 cause we wish to learn something about human his- (D) is primarily a story about avoiding
tory. We read it as well because we want to know the death
meaning of life. (E) cannot be truly understood by the
modern reader
6. Which of the following, if true, would most
weaken the author’s main point in Passage 1? 9. Which best expresses the relationship
between Passage 1 and Passage 2?
(A) Many ancient epics other than Gil-
gamesh were heavily concerned (A) Passage 1 is a factual description
with the quest for eternal life. of the plot of Gilgamesh, while
(B) Scholars doubt the authenticity Passage 2 offers a more subjec-
of Tablet XII, which appears to tive summary of the story.
date from a later century than do (B) Passage 1 was written for histori-
the other tablets which constitute ans, while Passage 2 was written
Gilgamesh. more for specialists in literature.
(C) A recently discovered missing (C) Passage 1 discusses thematic
tablet describes Gilgamesh’s elements of Gilgamesh, while
journey as motivated primarily by Passage 2 explicitly denies the
his feelings of loss after Enkidu’s presence of any concrete theme
death. in the story.
(D) In the story, the Bull of Heaven (D) Passage 1 is specifically about Gil-
attacks Gilgamesh and Enkidu gamesh, while Passage 2 uses the
because Gilgamesh has rejected example of Gilgamesh to make a
the goddess Ishtar. broader point about narratives.
(E) Gilgamesh is written in cuneiform, (E) Passage 1 and Passage 2 both refer
which only a handful experts can to the importance of Gilgamesh
read. in world literature.

108 | © The Princeton Review, Inc.

11 Crit Read 108 11/15/05 3:30:43 PM


Critical Reading Homework

Long passage-based Reading


Questions 10-18 are based on the following ers of the century after the death of Giotto and Dante
passage. with expanded opportunities to emphasize realism
and to bring the viewer into the picture. Again, the
The fourteenth century saw the beginning of the Sienese resisted, refusing to employ perspective for a
renaissance of Italian art. The following passage 50 century.
discusses the changes in art that occurred during this Let us be sure that we understand the meaning of
period. perspective. In such a painting straight lines (often
imaginary) converge in what is called a vanishing
At the turn of the fourteenth century the influence point, located somewhere in the background (often
of Gothic sculpture descended into Italy from north- 55 at the center of the horizon). This gives the impres-
ern Europe and revivified all the arts. Gothic sculptors sion of a real scene that is visible to the viewer. This
Line emphasized realism in their carvings of religious sub- approach had never been used before in any art. The
5 jects, and this new realistic bent soon overcame the new art of perspective said something radically dif-
abstract, symbolic Byzantine style that had previously ferent and new about the position and role of human
been dominant in most of Italy. Pisan and Florentine 60 beings in the cosmos, in the world picture. In pre-
sculptors began to imitate the Gothic style. Giotto, a Renaissance art, the scene depicted is seen not from
Florentine, painted frescoes that had a new realism the viewpoint of the beholder, an ordinary human, but
10 and vitality. Dante, a fellow Florentine and poet, ex- from the viewpoint of God, from a point at infinity.
celled in the dolce stil nuovo, the “sweet new style” of
writing verse that focused on the experiences of real, * The Annunciation, the Crucifixion, the Deposition, and the Marriage at Cana
are episodes from the Bible.
even ordinary, people.
Realistic portrayals of the lives and acts of or-
15 dinary people are not the only things that art can 10. The first paragraph (lines 1-13) implies that
produce, and it is not what art traditionally had ac- in pre-fourteenth-century Italy, realistic
complished throughout the centuries leading up to this portrayals of life were
time. And even during the fourteenth century, there
(A) the exclusive domain of artists
were artists who held out against the new style. The
(B) valued more in writing than in
20 painters of the Sienese school, in particular, continued
painting
to produce works that were notably Byzantine in style
(C) not considered the purpose of art
with their quiet, stylized faces and forms and their ob-
(D) rare but highly valued
vious religious symbolism. For this reason we usually
(E) imbued with religious symbolism
do not think of the fourteenth-century Sienese paint-
25 ers, great as they were, as being part of the Italian
Renaissance. They were great painters, but they were 11. The author mentions the Sienese school of
not Renaissance artists. painting (line 20) in order to
As the Renaissance spread throughout Europe, it (A) illustrate the accomplishments of
everywhere produced a new style in art that em- painters of the new style
30 phasized realism, naturalness, and verisimilitude. (B) describe the pre–fourteenth cen-
The subjects often remained the same as in the old tury ideal of painting
Byzantine symbolic style: the Annunciation, the (C) criticize the resistance of this
Crucifixion, the Deposition, the Marriage at Cana, school to the new developments
and the like*. But now the people depicted reflected in art
35 the viewer’s world, expressed feelings like his own, (D) show that the new style in painting
and moved him, as a consequence, in an entirely new was not universally adopted
way. (E) describe the variety of styles of
Giotto, though a master, was not wholly a painting that are considered part
Renaissance painter, in that he did not experiment of the Renaissance
40 with perspective as the Florentine artists of the
fifteenth century did (the quattrocento, as Italians
say). The discovery of the possibilities of perspec-
tive helped to produce works of art that are decidedly
more familiar to us than those of Giotto, and more
45 “Renaissance-looking.” Perspective provided the paint-

© The Princeton Review, Inc. | 109

11 Crit Read 109 11/15/05 3:30:44 PM


Refresher Manual for the SAT

12. The word “verisimilitude” (line 30) most 16. The author’s use of quotation marks in line
nearly means 45 serves to indicate
(A) the appearance of truthfulness (A) his disapproval of the use of labels
(B) the reflection of light (B) his confusion about the characteris-
(C) the making of comparisons tics of Renaissance painting
(D) the portrayal of crowds (C) the difficulty of establishing precisely
(E) the skill of artistry what makes a painting Renaissance
(D) his criticism of Giotto’s work
13. It can be inferred from the passage that which (E) his adoption of a conversational tone
of the following would most likely be an
example of an early Renaissance painting? 17. According to the final paragraph (lines
51-63), the most important result of the use of
(A) A stiff and subdued scene of a family perspective was
having dinner
(B) A symbolic portrait of a man in a (A) a more accessible style of poetry
position of power as seen in the works of Dante
(C) An abstract rendering of a street scene (B) the ability to create a scene that
(D) A natural depiction of a scene from more closely resembled images a
the Bible person would actually see
(E) A stylized image of a peasant woman (C) a depiction of people in their ordi-
nary lives
14. Which of the following best describes the (D) the departure from the overly calm
relationship between the new sense of style of Byzantine art
realism that appeared during the fourteenth (E) the depiction of scenes that caused
century and the use of perspective? viewers to feel as though they
were actually in them rather than
(A) The emphasis on realism and the viewing them
use of perspective were essen-
tially the same thing. 18. It can be inferred from the passage that prior
(B) The use of perspective followed to the Renaissance
the introduction of a more realis-
tic style of art. (A) art did not succeed in moving people
(C) The use of more realistic repre- (B) man’s perspective on the universe
sentation was superseded by the was given less value than that of
adoption of perspective. God
(D) The use of perspective was re- (C) art had at times explored Biblical
sponsible for the new importance themes
given to realism. (D) the Sienese school had been con-
(E) The depiction of realistic scenes sidered the foremost in Italy
and the use of perspective were in (E) Dante had already written in the
opposition to each other. dolce stil nuovo of verse

15. Which of the following statements could be


accurately applied to Giotto?
(A) His use of perspective separated him
from the Sienese school of painting.
(B) His frescoes were close imitations of
Gothic styles of sculpture.
(C) He moved to Florence from Pisa.
(D) His lack of use of perspective
distinguished him from later
Renaissance painters.
(E) His mastery of painting faltered only
when he attempted to use perspective.

110 | © The Princeton Review, Inc.

11 Crit Read 110 11/15/05 3:30:44 PM


Critical Reading Homework

Questions 7-18 are based on the following playoffs. But an individual club had to face the fact
passage. that an extended season would do it no good, that its
dwindling audience would be spread too thin over too
The following passage describes some of the factors 50 long a period of time.
that brought about the decline of the All-American Still, the clubs wanted to believe that solutions
League, a women’s professional baseball league that were possible. They sought a scapegoat and decided
formed in the 1940s. that poor financial management coupled with poor
direction were the major culprits. The clubs’ response
No one realized at the end of 1948 that the 55 was to tighten the League’s belt. They cut expenses
All-American League had reached its high point. for administration, publicity and scouting. As of 1949,
Women’s baseball seemed be on a roll, riding an ever- collective spring training was abolished. That deci-
Line upward curve of popularity. Attendance had risen sion had far-reaching consequences. Spring training—
5 steadily since 1943; surely it would continue to do so. along with preseason exhibition games—had provided
The League made decisions based on this optimistic 60 the game with a surefire kickoff each season, attracted
outlook, but Fred Leo acknowledges that they were the national press, and unearthed at least some worth-
whistling in the dark; churning out public relations while talent.
hype that a close look at hard figures failed to warrant. As the League pursued its relentless cutbacks, sala-
10 But the figures were hard to read; the decline wasn’t ries failed to keep pace with inflation. To a woman
absolute. Some cities continued to rack up phenom- 65 in her twenties, her playing days numbered, a steady
enal attendance, given the size of the population base. job of any kind began to look attractive. No one could
The league’s head office and the club directors kept make long-term plans. For the majority of players,
telling each other that all their woes were simply staying in the All-American meant putting the rest
15 growing pains, that a tighter control on the finances of their lives on hold. This sacrifice would eventually
would put them in the black. But they were wrong. 70 become too great.
The summer of 1948 was the last time the League All in all, 1949 was not a good year for the All-
moved forward. Soon it would slip, slowly at first, and American. Only two clubs showed a profit, and five
then with ever-gathering momentum. registered losses ranging between $15,000 and $27,000.
20 None of the problems that had plagued the League One broke even. Rockford had, predictably, won the
in 1948 were to be resolved. Training and recruitment 75 championship, and cutbacks continued unabated. The
procedures failed to provide an adequate number of League board members argued over whether or not
promising players. Teams were aging in place, relying players should take a salary cut. The 1950 season was a
on longtime veterans. Even winning teams believed debacle. The Muskegon Lassies folded halfway through,
25 they’d done well despite, not because of, the system. and the team was relocated to Kalamazoo. Racine—a
Expansion hadn’t worked in 1944, and it didn’t work 80 founding city—called it quits after the 1950 season,
in 1948, either. and the Belles moved to Battle Creek, Michigan. The
In 1948, the League would claim that overall at- trickle of departing players became a flood. Over the
tendance had reached one million, but even this was course of two seasons, the All-American lost some-
30 an exaggeration—the actual figure was 910,000. This thing like two dozen experienced veterans, and many
was about 120,000 more than in 1947, but two-thirds 85 who replaced them weren’t of the same quality.
of that increase was accounted for by home games in And so the League straggled on, locked in a los-
Springfield and Chicago. In other words, both expan- ing battle. Attendance and revenues continued their
sion teams together added a mere 80,000 people all downward slide. But somehow the clubs endured,
35 season long—a pitiful showing, especially consider- sustained by never-say-die fans and players for whom
ing the potential audience in a city the size of Chicago. 90 the League continued to exert appeal. A new crop
Only half the other teams showed any increase in of rookies kept appearing, year after year. The post-
attendance at all. In Muskegon, where unemployment 1948 period had its own stars, who were as beloved
continued to rise because of postwar layoffs, 60,000 by fans as those of the glory days. The League was
40 fewer people came out to the ballpark to see the still something to aim for: the top of the pole, and the
Lassies play in 1949. Rockford, the playoff champi- 95 only game in town. But those who dreamed of playing
ons, ran an $11,000 deficit. The League grossed more had realized the fragility of their ambition. Marilyn
money than ever before but several teams had peaked, Jenkins, the Grand Rapids bat girl, remembers hoping
for any number of reasons, and had nowhere to go that the League would last long enough for her to play.
45 but down. The League could make money by adding It did, and she became the Grand Rapids catcher until
cities, or by extending both the regular season and the 100 the League’s demise.

© The Princeton Review, Inc. | 111

11 Crit Read 111 11/15/05 3:30:44 PM


Refresher Manual for the SAT

7. In line 9, the word “warrant” most nearly 11. According to the third paragraph (lines
means 28-50), expansion teams are described as
having
(A) arrest
(B) justify (A) made an insignificant contribution
(C) guarantee to overall attendance at League
(D) combat games
(E) question (B) severely drained the League’s
already strained finances
8. According to the author, optimism about (C) spread a limited audience too thin
the future of the League in 1948 was strong (D) brought less-skilled talent into the
because League
(E) been a primary reason for the
(A) profits had remained consistently League’s decline
high
(B) after a disturbing drop, attendance 12. The author’s attitude toward the players who
at games had stabilized left the League for financial reasons is most
(C) publicity had effectively convinced
likely one of
the public of the League’s stabil-
ity (A) concern
(D) attendance at games had consis- (B) bemusement
tently risen (C) disapproval
(E) the attendance-to-profit ratio had (D) understanding
steadily improved (E) disappointment

9. The author most likely uses the phrase 13. The “two clubs” that registered a profit in
“Teams were aging in place” (line 23) in order 1949 (line 72) are presented as
to
(A) typical of the success of previous
(A) demonstrate the toll that professional years
baseball took on the players (B) exceptions to the otherwise bleak
(B) establish that the average age of a financial performance of League
new player had increased teams
(C) comment on the insufficient number (C) representative of teams that suc-
of talented new players in the League ceed financially but not on the
(D) prove that women’s teams were playing field
failing to make progress (D) examples of what could be accom-
(E) delineate the difference between plished with proper management
successful and unsuccessful (E) proof of the impossible task faced
teams by League teams

10. The author would probably view the 14. The passage suggests that the decline of the
League’s claim that “attendance had reached All-American League can be attributed to all
one million” (lines 28-29) as of the following EXCEPT
(A) a regrettable but understandable (A) the effect of postwar unemployment
error on attendance figures
(B) an example of the League’s inef- (B) a lack of new talent to replace retiring
fective management veteran players
(C) the result of an optimistic over- (C) inadequate restriction of expenditures
sight (D) the failure of expansion teams to bring
(D) a malicious attempt to deceive the public in large numbers of spectators
(E) an example of publicity hype (E) the large number of individual
teams that registered significant
losses

112 | © The Princeton Review, Inc.

11 Crit Read 112 11/15/05 3:30:45 PM


Critical Reading Homework

15. The author implies that the League’s 17. Which of the following would most justify
abolition of “collective spring training” (line the author’s description of the League as “the
57) only game in town” (lines 94-95)?
(A) discouraged players from putting (A) Playing for the League had more
their full concentration into their prestige than many other occupa-
playing tions.
(B) caused the eventual collapse of the (B) Playing professional baseball
Muskegon Lassies allowed women to leave rural
(C) allowed the League to minimize areas and to move to more urban
pay cuts to the players settings.
(D) lowered morale among the (C) Participation in baseball was more
younger, less experienced players acceptable than participation in
(E) deprived the League of an impor- sports such as boxing or golf.
tant source of publicity (D) Playing baseball was the only way
for women to be publicly ac-
16. The reason that the author describes claimed.
the ambition of women wanting to play (E) Women had few opportunities to
professional baseball after 1948 as “fragile” participate in professional sports
(line 97) is that in the 1940’s and 1950’s.

(A) society at that time viewed women 18. It can be inferred from the passage that
as physically less strong than Marilyn Jenkins
men
(B) it was uncertain how long the All- (A) was one of the founding team
American League would continue members
to exist (B) was one of the stars of the post-
(C) a high level of skill was required of 1948 period
players in order to be accepted into (C) was optimistic about the future of
the League the League
(D) women were unwilling to sacrifice (D) was not among the first women to
financial comfort for the chance to play for the League
play professional baseball (E) contributed to the demise of the
(E) many women quit baseball after All-American League
enduring its rigors for several years

© The Princeton Review, Inc. | 113

11 Crit Read 113 11/15/05 3:30:45 PM


Refresher Manual for the SAT

Questions 1-13 are based on the following


passages.

The following two passages presents views of the such extraordinarily high standards from the indus-
public’s perception of air pollution and the validity of trial system? Who are the people who are willing to
certain methods used to curtail that pollution. make such sacrifices in material production in order
to achieve such marginal gains in public health and
Passage 1 50 safety? And why, it might be added, is their crusad-
ing always aimed so directly at the industrial system?
We may legitimately ask: Who are environmental- There are other environmental factors, after all, that
ists? Are they generous enough that they are ready contribute to such health problems. One of the best
to give up the quest for material goods and distrib- ways to improve the air for asthmatics, it might be
Line ute their wealth among the poor? Are they virtuous 55 argued, would be to eliminate all the pollen-produc-
5 enough to be ready to renounce their possessions ing weeds and flowers with an aggressive herbicidal
and pursue only spiritual values? At a time when the spraying program. These natural sources cause much
average income in the world, if distributed perfectly more suffering among asthmatics than does industrial
equally, would put us all at a rather low standard of air pollution.
living, who are these people that feel that we are so
10 wealthy that we can now let slip the long human quest Passage 2
for material improvement?
Let me approach the problem from another angle. 60 Everyone knows about air pollution, and no one
In recent years, the Environmental Protection Agency likes it. Even without dramatic incidents such as
has been trying to set minimum standards for clean, the tragic deaths of seventeen persons in Donora,
15 healthy air. Most people assume that this means that Pennsylvania, within a single four-day period of
the average person would be able to breathe this air intense smog, everyone who lives in or near a big
without being adversely affected. Congress was per- 65 city—and many who live in rural areas—is aware of
suaded, however, to protect small subgroups as well. air pollution. There is nothing obscure about it. Daily
As a result, the Environmental Protection Agency has they see it, feel it, and breathe it. And they do not like
20 set the standard so that the air must be clean enough it.
not to affect asthmatics. These people have special They may be people living in New York City,
breathing problems that are not caused by air pollu- 70 where the normal cost of air pollution in lost working
tion but are aggravated by a variety of environmen- time because of pollution-related illness and in extra
tal insults, the most notable being the pollen that is painting and cleaning averages several thousand dol-
25 produced each year by plants. Asthmatics constitute lars per family, according to a study done by the state.
about 4 percent of the population. Nevertheless, the They may be the residents near Garrison, Montana
EPA has decided that the rest of the society must pay 75 who complained about the fluoride gases released into
the price for redesigning the industrial system to pro- the air from a phosphate plant. They may be farm-
tect a few individuals. ers whose crops have been ruined by air pollution. Or
30 This might seem like a significant sacrifice for a they may be persons living in the northern portion of
society to make, since the costs are not small, and Staten Island, New York, where a study made by the
the resources devoted to this task must be taken away 80 U.S. Public Health Service indicated a close relation-
from producing other goods. But environmental lob- ship between air pollution and a high incidence of
bying groups in Washington are still not satisfied. For death from lung cancer.
35 some time, the Environmental Defense Fund, one Setting aside economic and aesthetic consider-
of the three or four major environmental lobbying ations, our health alone should give us great concern
groups, has been pressuring the EPA to tighten the 85 about air pollution. Much remains to be learned about
clean-air standards to make the air safe for people the relationship between various diseases and the
suffering from cystic fibrosis. This is an inherited dis- contaminants in the air, and the experts often dis-
40 ease that affects .005 percent of the population, about agree on what causes what and to how great a degree.
eleven thousand people. It is not caused by air pollu- Nevertheless, the Senate’s Committee on Public
tion but is only aggravated by it. The costs of achiev- 90 Works minced no words when it reported on the
ing this kind of cleanliness would be rather high. problem. “From the standpoint of public health,” the
So, once again, we can ask the question: Who committee stated, “the information available concern-
45 are the citizens of a country who are able to demand ing the acute air pollution episodes that have occurred

114 | © The Princeton Review, Inc.

11 Crit Read 114 11/15/05 3:30:45 PM


Critical Reading Homework

and the laboratory evidence of the effects of exposure 3. In Passage 1, the reference to EPA standards
95 to various pollutants that are in the air puts an excla- that maintain that air must be “clean enough
mation mark by the word ‘urgency’ in relation to this not to affect asthmatics” (lines 20-21) serves
problem. to
“In any given instance, there may be several rea-
sons why a particular situation may result in chronic (A) suggest that certain environmental
100 disease. But there is no question that the pollutants in legislation is not extreme enough
the air are contributing factors to the chronic respi- (B) applaud the Environmental Protec-
ratory diseases, lung cancer, emphysema, bronchi- tion Agency’s efforts at cleaning
tis, and asthma. When considering mortality in this up the nation’s air
country, it is this group that is showing a rapid rise (C) concede that some environmental
105 as a cause of death and disability. These subtler, less legislation is good and necessary
dramatic long-range effects of air pollution are of (D) question the practice of allowing
much more serious consequence to the population as a special interests to shape legisla-
whole than the occasional major tragedy.” tion
(E) imply that greater care should be
taken to protect citizens with
1. In line 11, the word “material” most nearly
certain respiratory ailments
means
(A) fabricated 4. The word “aggravated” in line 42 most nearly
(B) substantive means
(C) monetary
(D) textile (A) angered
(E) synthetic (B) annoyed
(C) worsened
(D) degraded
2. The author of Passage 1 poses the questions
(E) polluted
in lines 1-11 in order to
(A) denounce the environmentalists for 5. For which of the following reasons does
their reckless disregard of ethical the author of Passage 1 fault the “lobbying
realities groups” mentioned in lines 33-34?
(B) praise the environmentalists for
their charitable natures (A) Their narrow range of experience
(C) suggest that the views of environ- (B) Their limited resources
mentalists are hypocritical and (C) Their ineffective practices
impractical (D) Their unethical motives
(D) point out to the world’s population (E) Their lack of concern for other
that there are not limitless funds interests
with which they may achieve
their goals
(E) imply that environmentalists have
a hidden agenda and are only
interested in their own material
gain

© The Princeton Review, Inc. | 115

11 Crit Read 115 11/15/05 3:30:45 PM


Refresher Manual for the SAT

6. The reference in line 49 to “marginal gains” 8. Which of the following does NOT support
serves to the contention expressed in Passage 2
that there are non-health-related costs to
(A) prove that measures taken by envi-
pollution?
ronmentalists are inefficient and
unnecessarily extreme (A) Citizens of New York City spend a
(B) suggest that even a small amount great deal of money on air filter-
of improvement is desirable in ing systems.
the face of deplorable environ- (B) The destruction of agricultural
mental conditions crops causes farmers to purchase
(C) imply that some standards are not goods that they would normally
worth the resources required to grow for themselves.
enforce them (C) A Montana phosphate plant loses
(D) highlight the disparity between a great deal of costly fluoride gas
the attention paid to certain small every year.
subgroups and that paid to the (D) Pollution causes the U.S. Public
industrial system Health Service to conduct many
(E) establish that eliminating certain expensive studies.
pollen producing weeds and (E) New York City has the highest
flowers would be a more effective number of workdays lost due
means of improving the air for to pollen-related respiratory ill-
asthmatics nesses.

7. According to the author of Passage 2, the 9. By putting an “exclamation mark by the


“intense smog” mentioned in line 64 was word ‘urgency’ ”, (Passage 2, lines 95-96) the
responsible for which of the following? Senate’s Committee on Public Works means
to
(A) A greater awareness of the prob-
lems of air pollution (A) use needless hyperbole in report-
(B) A growing dissatisfaction with big ing the effects of air pollution
city life (B) show the disparity between actual
(C) A considerable number of fatali- air pollution episodes and certain
ties laboratory evidence
(D) Increased diagnoses of asthma (C) strongly question the findings of
(E) Begrudging acceptance of poor air the Senate committee
quality (D) emphasize the dangerous effects
that certain pollutants have on
public health
(E) question the accuracy of laboratory
evidence when dealing with pol-
lution and issues of public health

116 | © The Princeton Review, Inc.

11 Crit Read 116 11/15/05 3:30:46 PM


Critical Reading Homework

10. According to the quotation at the end of 12. Which of the following represents the
Passage 2, what is the difference between fundamental difference in the arguments of
the “long-range effects of air pollution” (line the two authors?
106) and “the occasional major tragedy” (line
(A) While the author of Passage 1 is
108)?
concerned with monetary issues,
(A) The second is more psychological- the author of Passage 2 concen-
ly threatening to the population trates on health issues.
because of its dramatic impact. (B) While both of the authors con-
(B) The first is more threatening to cern themselves with issues of
the population than the second expense, the author of Passage
because the long-term effect on 1 believes the money is worth
chronic disease could lead to a spending.
greater number of fatalities. (C) While the author of Passage 1
(C) The first is more threatening be- recognizes air pollution as a nec-
cause the major tragedies usually essary evil, the author of Passage
involve lesser forms of pollution. 2 believes in the eradication of all
(D) The second is more severe because air pollution.
major environmental tragedies (D) While the author of Passage 2 has
usually involve tens of thousands little respect for environmen-
of people. talists, the author of Passage 1
(E) The second is more life-threaten- reveres them.
ing to a greater percentage of (E) While neither of the authors
the population because it affects believes the problem of air pol-
people without chronic respira- lution is as bad as is commonly
tory diseases. believed, the author of Passage 2
recognizes it as a greater threat
11. One distinction between the attitudes of the than does the author of Passage 1.
two authors is that
13. Which of the following does the author of
(A) the author of Passage 1 believes Passage 2 use in the construction of his
that only a small percentage of argument that the author of Passage 1 does
the public is affected by pollution, not?
while the author of Passage 2
believes that pollution has a more (A) A direct quotation
universal impact (B) A hypothetical question
(B) the author of Passage 2 fails to (C) A reference to a medical condition
take into account as many crucial (D) A discussion of social issue
statistics supplied by the govern- (E) A specific example
ment that affect the public as
does the author of Passage 1
(C) the author of Passage 1 doesn’t
care about the public’s interest
while the author of Passage 2
does
(D) the author of Passage 1 regards the
public with more suspicion than
the author of Passage 2, believing
that the small minority of pollu-
tion sufferers demand unreason-
able and questionable changes
(E) the author of Passage 1 thinks that
not enough citizens consider the
environmental dangers they cause,
while the author of Passage 2
feels that people knowingly jeop-
ardize the health of the planet
© The Princeton Review, Inc. | 117

11 Crit Read 117 11/15/05 3:30:46 PM


Refresher Manual for the SAT

Sentence completions 9. Despite the appearance of prosperity, at his


death Colin Farnsworth left his family -------,
ruined by secret gambling debts.
Fill In the Blank 10. Having shown up without the ------- number
of players, the team was forced to forfeit the
On each of the following questions, substitute your game.
own word or phrase to complete the sentence. Pay
close attention to clues and triggers. 10. Her questions regarding her son’s where-
abouts the night before were -------; she sim-
1. Few teams had endured such a ------- season, ply did not give up until he told her every-
losing every game by a landslide. thing.
1. Shirley’s giggling was entirely inappropriate
given the ------- of the situation.
One Blank at a Time
2. Despite his higher salary, Owen felt that his
new job lacked the ------- of his previous po- Approach two-blank sentence completions one
sition; he could find absolutely no excitement blank at a time. We’ve made it easier for you by
in the work that the job entailed. eliminating answer choices for one of the blanks.
Eliminate as many wrong answers as possible.
3. The school did not have a ------- grading pol-
icy; each teacher was free to mark students
according to any system he or she thought
appropriate. 4. While most environmental activists ------- the
ravaging of the South American rain forests,
5. The declining neighborhood underwent a governments continue to insist that reshaping
------- when a group of investors bought sev- the land is ------- to economic growth.
eral crumbling tenements and turned them
into attractive apartment buildings. (A) encourage . . ????
5. Although she usually insisted upon having (B) deny . . ????
things her own way, Lori would sometimes (C) deplore . . ????
------- her position in order to promote har- (D) epitomize . . ????
mony among her friends. (E) support . . ????

6. Although he tried to never raise his voice,


6. The elderly woman had enjoyed -------
James frequently came close to ------- in
most of her adult life, but memories of
response to his older sister’s teasing.
her impoverished childhood had made her
7. The ------- of new immigrants around the excessively -------.
turn of the century produced a culture of
many different customs and languages. (A) ???? . . thrifty
(B) ???? . . sociable
7. When shopping, Marlene was one of the most
(C) ???? . . afraid
------- people I knew; most of her purchases
(D) ???? . . knowledgeable
were never even used.
(E) ???? . . provincial
7. Environmentalists argued that the chemical
wastes being dumped in the river were so 7. The ------- of foreign pronunciations makes
lethal as to threaten the fish with -------. one overlook the common heritage of all
7. The clever forgery fooled the museum cura- languages, whose bond becomes obvious when
tor but did not withstand the ------- of the one discovers the countless spelling -------.
experts; after studying it for several weeks,
the panel of art historians pronounced the (A) ???? . . irregularities
painting a fake. (B) ???? . . similarities
(C) ???? . . corrections
(D) ???? . . intricacies
(E) ???? . . variations

118 | © The Princeton Review, Inc.

11 Crit Read 118 11/15/05 3:30:46 PM


Critical Reading Homework

8. Although this book about Brazilian poets 3. For Gretchen, rock climbing was -------; her
is, in general, accurate and consistent, it brother, though, found the thought of hanging
presents some ------- arguments that it never hundreds of feet from the ground a -------
really -------. notion.
(A) similar . . ???? (A) daunting . . hopeless
(B) precise . . ???? (B) thrilling . . chaotic
(C) conflicting . . ???? (C) invigorating . . petrifying
(D) paradoxical . . ???? (D) perilous . . discouraging
(E) erroneous . . ???? (E) exhausting . . fatiguing

10. The phenomenon of jazz is remarkable; 4. To the officer, who was accustomed to
somehow an otherwise ------- cascade of outbursts of ------- from apprehended
sounds becomes organized into a ------- suspects, the alleged culprit was unusually
whole. -------.
(A) disparate . . ???? (A) hostility . . composed
(B) consistent . . ???? (B) violence . . humorous
(C) discordant . . ???? (C) indifference . . reserved
(D) fractional . . ???? (D) belligerence . . disorderly
(E) significant . . ???? (E) thievery . . underhanded

5. Dr. Higgins was reported to be a strict


Putting It All Together professor, but he always tempered his critical
Combine all the techniques you have learned to words with a ------- smile.
answer each of the following sentence comple-
(A) detached
tions.
(B) venerable
(C) disarming
(D) scholarly
1. Not surprisingly, the ruling government (E) perfunctory
crushed the latest rebellion just as it had
------- each previous coup.
6. Even though his research is greatly -------, it is
(A) suppressed by no means flawless, and his results will not
(B) assimilated be accepted without some degree of -------.
(C) satisfied
(A) esteemed . . ovation
(D) distracted
(B) confidential . . adulation
(E) established
(C) admired . . skepticism
(D) understood . . support
2. William’s temporary paralysis, the result of a (E) mistaken . . befuddlement
violent automobile accident, left him helpless
and ------- him to live off the generosity of
7. Novelist Gabriel García Márquez is anything
his friends.
but -------; much of the power of his work
(A) transposed derives from his willingness to abandon
(B) inclined realistic notions of time and space.
(C) repressed
(A) an individualist
(D) compelled
(B) a theorist
(E) enabled
(C) a misanthrope
(D) a literalist
(E) a fanatic

© The Princeton Review, Inc. | 119

11 Crit Read 119 11/15/05 3:30:46 PM


Refresher Manual for the SAT

8. At its best, scientific investigation is a


------- activity; researchers know that their
work will be open to general scrutiny, and so
they ------- their arguments in defense of their
positions.
(A) careful . . neglect
(B) secret . . strengthen
(C) tedious . . invent
(D) public . . marshal
(E) scholarly . . denounce

9. Playwright David Henry Hwang is devoted


to ------- the theory that Chinese culture is
irrelevant to young Chinese-Americans, a
theory that he contends has only alienated
them from their ethnic heritage.
(A) disseminating
(B) exploding
(C) fostering
(D) endorsing
(E) dignifying

120 | © The Princeton Review, Inc.

11 Crit Read 120 11/15/05 3:30:47 PM


Writing Introduction

1
Easy .
Improving .
.
Sentences Medium .
.
Difficult 11
1
12 .
Easy . Easy .
. .
. .
Error . .
Medium . .
ID . .
. . Improving
. Medium . Sentences
. .
Difficult . .
29 .
30 .
. .
Improving . .
Paragraphs . Difficult .
. .
35 14
35 Questions 14 Questions
25-Minute Section 10-Minute Section

What is on the Test?


• 18 Error IDs
• 25 Improving Sentences questions
• 6 Improving Paragraphs questions
• One 25-Minute Essay

© The Princeton Review, Inc. | 121

12 writing 121 11/16/05 4:09:32 PM


Refresher Manual for the SAT

HOW IS THE WRITING SECTION SCORED?


The scaled score (200–800) you receive on the Writing section is derived from two
components: the essay score and the multiple-choice grammar score. Your essay
will be graded by two different people and each will give it a score on a scale of 1–6.
The sum of these two grades is weighted so that the essay score is worth approxi-
mately 30% of the total score; then the result is added to your grammar raw score.

Your essay score: × (2) =


<2–12> <4–24>

Grammar questions correct:


minus

Grammar questions incorrect: 4 =


equals

Grammar raw score:

1
Your grammar raw score is the number of questions you get correct minus of
4
the questions you get wrong. Questions left blank neither contribute to nor detract
from your score.

Now compare this to the chart on the next page to determine your scaled score.

122 | © The Princeton Review, Inc.

12 writing 122 11/16/05 4:09:33 PM


Writing Introduction

Estimated Writing Scores


If your scaled Depending on your essay score,
grammar score is your writing score will range from
40 360-500
45 400-540
50 440-580
55 480-620
60 520-660
65 560-710
70 610-750
75 640-780
80 700-800

Grammar Pacing Chart


So attempt this many questions
To get: You need to Total # of
35-question 14-question
(scaled earn: (raw questions to
section section
score) points) attempt
35 5 10 5 15
40 11 13 7 20
45 17 16 8 26
50 22 22 9 31
55 27 26 10 36
60 31 27 11 38
65 36 31 all 45
70 40 all all 49
75 44 all all 49
80 49 all all 49

Your POOD and You


ETS doesn’t make an order of difficulty on the Writing section. For the multiple-
choice questions on the Writing section, you have to make your own Personal Or-
der of Difficulty (POOD). First do the ones that you do best on, then save the ones
you find the most difficult for last.

© The Princeton Review, Inc. | 123

12 writing 123 11/16/05 4:09:34 PM


Refresher Manual for the SAT

Joe Bloggs and grammar questions


Improvement in the Grammar questions starts with knowing how Joe approaches
them.

What Joe Does What Should You Do?


• Focuses on what “sounds” right
• Reads and re-reads endlessly
• Eliminates answers just because he
doesn’t like them

the essay
Essay writing under timed conditions is never easy. The best way to improve is to
write as many practice essays as you can. Trust us: it does get easier over time!

As you work, always keep in mind a few important things:

• Know what you are going to say and how you are going to say it before
your pencil hits the page.
• There are no right answers to the questions posed: Don’t be afraid to take
an “unpopular” position.
• Organization is a good thing: Always have an introduction and conclusion.
• Watch your language! Use language appropriate to the task, and don’t for-
get to apply all that grammar you worked so hard to learn!
• Don’t be afraid to use expressive language if and when it helps support
your essay.

124 | © The Princeton Review, Inc.

12 writing 124 11/16/05 4:09:34 PM


The Essay, Revisited

How the other half lives


As you will undoubtedly recall, SAT essays are graded quickly and holistically.
This means that graders are not carefully correcting each essay, red pencil in
hand; they are looking it over for 45 to 60 seconds and then slapping a number on
it based on their overall impression of it. The first question they ask themselves
when deciding on a score tends to be, “Top half or bottom half?” Is the essay in the
4-6 range, or the 1-3 range?

Your first goal, therefore, is to crank out an essay that screams “TOP HALF!”
The three aspects of your essay that will most help in this effort are:

Length. Come as close to the last line provided as you can, without
going over.
Structure. The organization of your essay must be clear even to
graders who have read 200 of these things in a row and are praying
for a power outage to give themselves a break.
Examples. This is where a lot of people run into trouble even when
they’ve mastered the first two elements of the SAT essay, so we’ll talk
about examples in more depth.

BODY LANGUAGE
In each body paragraph, you have three tasks to accomplish:

• Make a smooth transition to introduce your example


• Present your example
• Tie your example clearly back to the prompt

© The Princeton Review, Inc. | 125

13 essayrevis 125 11/15/05 3:30:55 PM


Refresher Manual for the SAT

Transitions
Compare the following three excerpts:

…even today, no period of American history is as unrelentingly tragic as


the Civil War.

Romeo and Juliet by William Shakespeare is a play in which two young


lovers take their own lives. Their reasons can be traced back to…

vs.

…even today, no period of American history is as unrelentingly tragic as


the Civil War.

Another example of the greatest griefs being those we cause ourselves


can be seen in William Shakespeare’s Romeo and Juliet. In this play, two
young lovers…

Why is the second excerpt better than the first?

…even today, no period of American history is as unrelentingly tragic as


the Civil War.

Not only does history furnish us with an endless array of examples


illustrating that the greatest griefs are those we cause ourselves, but lit-
erature does as well; a prime example is William Shakespeare’s Romeo and
Juliet. In this play, two young lovers…

What does this third version to do improve on the previous two?

Introducing Your Examples : Assume Nothing


What does the second excerpt do that the first doesn’t?

One example that shows that the greatest griefs are those we cause
ourselves occurs in Adventures of Huckleberry Finn by Mark Twain. At
one point in this book, Huck pulls a trick on Jim, making him think he only
dreamed that they had been separated in the fog when in reality they actu-
ally had been.

vs.

One example that shows that the greatest griefs are those we cause
ourselves occurs in the Adventures of Huckleberry Finn by Mark Twain. In
this novel, the orphan Huck Finn and his friend, an escaped slave named Jim,
are rafting down the Mississippi River when Huck, who has paddled ahead of

126 | © The Princeton Review, Inc.

13 essayrevis 126 11/15/05 3:30:55 PM


the essay, revisited

the raft in a canoe, is separated from Jim in a dense bank of fog. By sheer
luck, Huck spots the raft again when the fog clears, and finding Jim asleep,
tricks him into thinking he’d dreamed the entire incident.

Never assume your reader is familiar with the characters, political fi gures,
historical events, or other examples you refer to. Be sure to provide con-
text for your examples.

speciFic examples Rule


In introducing your examples, it is important to be as specific as you can. Which of
these is more likely to impress a grader?

In the weeks leading up to the war, many government officials took to the
airwaves claiming that Iraq had weapons of mass destruction.

vs.

In the weeks leading up to the war, government officials such as Secretary


of State Colin Powell and National Security Advisor Condoleezza Rice took
to the airwaves claiming that Iraq had weapons of mass destruction.

How are you supposed to remember all these details? Hey, that’s why you’re pre-
paring your examples ahead of time!

Make sure you can rattle off names, dates, titles, locations, and other
such specifics as needed.

Specificity goes beyond names and dates. Compare these two sentences, from a
paragraph relating a personal experience:

I vividly recall my grandmother talking about how amazed she was by


things that we take for granted but which she had never even imagined be-
fore she emigrated from Russia to California.

vs.

I vividly recall my grandmother talking about how amazed she was by the
sight of mountains of fresh fruit in supermarkets and by the fact that she
could walk around in short sleeves on a February morning; she had never
even imagined such things before she emigrated from Russia to California.

What makes the second more effective than the fi rst?

© The Princeton Review, Inc. | 127

13 essayrevis 127 11/15/05 3:30:56 PM


refresher Manual for the sat

Tying examples BacK To The pRompT


Let’s fi nish one of the paragraphs we started earlier, written for the prompt, “The
greatest griefs are those we cause ourselves.” What would a grader think of a body
paragraph like this?

One example that shows that the greatest griefs are those we cause
ourselves occurs in Adventures of Huckleberry Finn by Mark Twain. In this
novel, the orphan Huck Finn and his friend, an escaped slave named Jim, are
rafting down the Mississippi River when Huck, who has paddled ahead of the
raft in a canoe, is separated from Jim in a dense bank of fog. By sheer luck,
Huck spots the raft again when the fog clears, and finding Jim asleep, tricks
him into thinking he’d dreamed the entire incident.

vs.

One example that shows that the greatest griefs are those we cause
ourselves occurs in Adventures of Huckleberry Finn by Mark Twain. In this
novel, the orphan Huck Finn and his friend, an escaped slave named Jim, are
rafting down the Mississippi River when Huck, who has paddled ahead of the
raft in a canoe, is separated from Jim in a dense bank of fog. By sheer luck,
Huck spots the raft again when the fog clears, and finding Jim asleep, tricks
him into thinking he’d dreamed the entire incident. When Jim finally learns
that Huck has pulled a prank on him, he is so hurt by this betrayal that
Huck agonizes over what he’s done; Huck undergoes many hardships over
the course of the novel, but his deepest suffering comes with the realiza-
tion that his own actions have been so cruel.

The last task of each body paragraph—tying the example back into the prompt
— is really the payoff of the entire paragraph.

After introducing each example, be sure you spend time clearly


explaining why it’s a good example.

peRsonal examples : BeWaRe The piTFalls


You may have heard that personal examples are not as good as other kinds of ex-
amples. Not true! The problem is that personal examples can be hard to use well.
Here are a few pointers:

• Treat them like any other kind of example. Explain exactly why your ex-
ample supports your thesis and to tie it back to the prompt.
• Don’t get so caught up in relating the story that you forget to come back
to earth. Avoid irrelevant details.

128 | © The Princeton Review, Inc.

13 essayrevis 128 11/15/05 3:30:57 PM


The Essay, Revisited

Staying out of Trouble


Graders of SAT essays encounter an astonishing variety of errors. Listing them all
individually could take indefinitely long and wouldn’t be very useful. But there are
a few general tips that can help you avoid many potential problems at once.

• Think first, then write. Remember that you only have 25 minutes, and
you’re writing longhand. You have to get it right the first time.
• Stick to your active vocabulary. The SAT is not the time to try out fancy
new words you aren’t 100% sure how to use correctly.
• Don’t be overly casual. You’re not chatting with your friends—you’re writ-
ing an essay for a grade on an important test. Avoid slang, and never use
abbreviations such as “+”, “b/c”, or “w/o”.
• Avoid the passive voice—active voice is always the better choice!

Sentence Variety
ETS graders are evaluating, among other things, how well you can vary your sen-
tences in both length and structure. Therefore, you want to deliver a blend of short
and long sentences.

Short, declaratory sentences are perfectly acceptable, but be careful not to Don’t use a pronoun that
overuse them. Too many short sentences will make your writing sound simplistic doesn’t stand for anything
and repetitive. Here’s an example: in particular. You won’t get
a high score if your grader
“Sometimes censorship is justified.” I disagree with this quote. I believe has to ask, “Which it?” or
that censorship is never justified. Censorship interferes with our freedom of “This what?”
expression. It may endanger our democracy. Censorship cannot solve any-
thing. It only takes away our individual freedoms .

Add punctuation and conjunctions to connect related ideas and create smoother
renditions of the sentences above.

A little complexity goes a long way, however. Long, complex sentences can be
difficult for you to control and may be difficult for a reader to easily understand.
How can this sentence be made clearer?

Many like to think that individual freedom supersedes all else, but in real-
ity, there is and always will be some sort of authority existing to keep one
within the rules, because no matter how important we believe the exercise
of individual freedom to be, there is always a need to maintain a clear dis-
tinction between right and wrong, a distinction that is not always easy for
an individual to see.

© The Princeton Review, Inc. | 129

13 essayrevis 129 11/15/05 3:30:57 PM


refresher Manual for the sat

Adjust sentence length where necessary when you proofread.

GooD, Better, Best


Here are some examples of actual student essays representing various scoring lev-
els. Describe what is wrong with each, and then rewrite or add to the text to im-
prove it.

Assignment: What is your opinion of the claim that sometimes honesty is not
the best policy?

essay a
I agree that honesty is sometimes not the best policy. At times exces-
sive honesty is unnecessary and can prevent desired outcomes. I will show
this to be true using the examples of Shakespeare’s Othello and the spy
Nathan Hale during the American Revolution.

In Othello, the main character confides in the villain, Iago. Othello reveals
his deepest secrets, holding nothing back. Iago, who is secretly bent on
Othello’s demise, uses this information to twist Othello from a successful
military leader to a paranoid man who murders the woman he loves.

During the American Revolution, Nathan Hale acts the part of a loyal-
ist while secretly feeding information to the colonial militia. He did this out
of love for his and his country’s independence from the tyrannical rule of
Britain.

130 | © The Princeton Review, Inc.

13 essayrevis 130 11/15/05 3:30:57 PM


The Essay, Revisited

Using the examples of Shakespeare’s Othello and Nathan Hale, I have il-
lustrated that sometimes honesty is not the best policy.

Essay B
Although it is commonly thought of as the correct and proper thing to
do, telling the truth is not always the best option. Although honesty nor-
mally creates fewer problems than lies, the truth should not be told when
and if one’s feelings, heart, or emotions are on the line.

Honesty, when used in the wrong situation, can cause much unneeded
hurt, anger, and resentment. For example, if one does not appreciate or
like a gift that they have received from a friend, it would not be wise to
state that they are unappreciative or not in favor of the present. Instead,
it would be a choice that would be beneficial to the giver for the recipient to
smile, state their thanks, and make comments regarding what they do like
about the gift. This method doesn’t even necessarily require stating any-
thing untrue; rather it allows the recipient to guard the giver’s feelings by
acknowledging what they do appreciate about the gift, instead of stating
their dislike.

Also, the truth should not be used when and if it will most likely cause
conflicts or arguments between two separate people. For instance, if one
had had an affair with a friend’s girl/boyfriend, it would not necessarily be a
wise decision to be completely upfront and direct with the friend regarding
the situation. If this were to be done, it would most likely cause tremendous
amounts of resentment between the couple as well as the friends. These
feelings would erupt because the girl/boyfriend that had been cheated on
would feel betrayed by two people: his/her partner and his/her friend.

In many situations, honesty allows people to move on and forget a situa-


tion, but many times, it can cause many more negative emotions than that
are necessary. In these instances, one should either tell the partial truth
or lie in the most minimal way. This will allow for the least amount of anger,
showing that honesty is not always the best policy.

Why did this essay get a 4? How could it be made better?

© The Princeton Review, Inc. | 131

13 essayrevis 131 11/15/05 3:30:57 PM


Refresher Manual for the SAT

Assignment: What is your view of the claim that the greatest griefs are those
we cause ourselves?

Essay C
I agree with the statement, “The greatest griefs are those we cause our-
selves.” Everyone is responsible for this or her own actions. Individuals have
a great ability to cause problems for themselves. My procrastination prob-
lems and the Monica Lewinsky scandals are examples of individuals causing
major problems for themselves.

In the past I have been known to cause problems for myself by procrasti-
nating. When I recieved an assignment, I would toss it aside and say I would
complete it later. This behavior led to me doing a sloppy, last minute job or
forgetting about the assignment alltogether. My grade fell in easy classes
because of my laziness. It was my biggest problem, and I had caused it on
my own for myself.

In the Monica Lewinsky scandal, President Bill Clinton was accused of


adultery. He didn’t admit to having been involved with her, but he was caught
in a lie. He was a married man and president and so should have stayed
away from a possibly hazardous situation. President Clinton brought the
problem upon himself by making a bad decision.

Some of the worst situations that a person can get into are caused by
the person themself. Knowing that each individual is responsible for his or
her own life and decisions is important. People can easily ruin their own lives
because of their poor judgment.

Why did this essay get a 4? How could it be made better?

132 | © The Princeton Review, Inc.

13 essayrevis 132 11/15/05 3:30:58 PM


The Essay, Revisited

Essay D
One may say that we are destined for a future and have no control over
the inevitable, but throughout history it is shown that the hardships we
face are often caused by our own actions. The troubles that we encounter
are ones of our own fault. The Great Depression, invention of nuclear weap-
ons and the civil rights movement all prove such a statement.

The rise of the economy during the early part of the 1900s, led by the
industry of the United States, caused a rise in the stock market which led
to an increase in stockholders investing in booming companys. However, at
the end of the 1920s fear of a crashing economy sent investers panicing
and selling of stocks. This panic caused the economy to crash and soon
led to the Great Depression. Had the panic not struck investers with such
strength, the events may never have happened. It is our own actions that
cause consequences.

The same is also true during World War II where the invention of the first
nuclear weapons used to defeat the Axis Powers were created. Once the
power of the weapons were demonstrated, nations seeked their own weap-
ons of mass destruction. To this day, the United States’ invention has
caused a constant threat of nuclear war with the Middle East and North
Korea.

On a social level, the oppression of minority groups in the United States


has caused turbulence in society throughout much of the past century.
First, women gained power in the 1920s, followed by African Americans dur-
ing the 1960s and 1970s and then other groups such as Latin Americans,
and men and women of homosexual orientation. The oppression suffered by
these minority groups led to the eventual liberation of civil rights through
tough campaigning, social equality was reached. Though many sought peace-
ful demonstrations, even turned violent such as race riots in Northeastern
cities.

History does not lie, and it is evident that much of the troubles in
the United States were caused by none other than ourselves. The Great
Depression of the 1930s were caused by events of the 1920s, the threat
of nuclear was, caused by its creation during World War II, and social turbu-
lence caused by oppression because of a misunderstanding in social values.

Why did this essay get a 4? How could it be made better?

© The Princeton Review, Inc. | 133

13 essayrevis 133 11/15/05 3:30:58 PM


Refresher Manual for the SAT

Essay E
“The greatest griefs are those we cause ourselves.” This statement was
clearly made by a wise, noble individual. Quite often, people either make
hurtful statements or commit heinous actions which hurt another person,
and eventually, hinder their relationship and elicit animosity. If more beings
were to simply think before their spoke, or contemplate the consequences
of their actions, they would rarely be forced to endure grief.

Reminiscing upon my grandfather, I find that statement to manifest


even more truth. About fifty years ago my paternal grandfather and grand-
mother adopted a young boy named Neil. Neil was raised unaware of his
adoption. However, when he turned sixteen, my grandparents believed that
it was finally time to divulge the truth to him; thus, they proceeded to re-
veal that they were not Neil’s biological parents. To their astonishment, Neil
handled the news maturely, and even decided that this issue was never to
be discussed again, and that they were to exist together as if he was their
real biological offspring.

Several years passed, and the fragile issue was never once brought up.
When Neil returned home from a European medical school at the age of
twenty-seven, he told his parents what he assumed to be wonderful news.
He revealed that he had met and fallen in love with a charming, success-
ful woman with an amiable nature and remarkable aspirations. Initially, my
grandparents were overjoyed, and extremely eager to meet their future
daughter-in-law. Therefore, a “meeting-date” was set for the following week.
When my grandparents opened the door to Neil’s girlfriend, their faces be-
came radiant with acrimony, for the woman standing at the door was not of
the same descent. My grandparents were appalled at Neil’s actions and my
grandfather insolently said, “Your actions simply provide proof that you are
not my blood. You were never my real son, and you never will be.”

Since the day my grandfather stated those words, our family has not
heard from Neil. My grandfather now completely regrets letting his enmity
get in the way of a beautiful relationship. With one utterance, he lost a re-
spectful, intelligent, and loving son.

In conclusion, the greatest grief we experience is instigated by our own


actions. If my grandfather had merely been open-minded, he would have nev-
er lost a son. Now, for the rest of his life, he’ll experience a perpetual feeling
of emptiness, which was only caused by himself.

Why did this essay get a 5? How could it be made better?

134 | © The Princeton Review, Inc.

13 essayrevis 134 11/15/05 3:30:58 PM


IMPrOVInG senTenCes

BASIC APPROACH
The key to success on Improving Sentences questions is to identify errors and com-
pare answer choices with each other. Follow this plan:

Read the sentence. If you spot an error right away, do the following:

1. Eliminate (A) and any other answer choices that make the same error.
2. Compare the remaining answer choices and note how they differ.
Don’t read the sentence
Eliminate any answer choices that contain new errors.
over and over, hoping the
3. Repeat Step 2 until you have one answer choice left. right answer will leap out
If you don’t spot an error when you read the initial sentence, do the follow- at you. Be methodical.
ing Compare answers, spot
the differences, and elimi-
1. Compare the answer choices and note how they differ. Eliminate any nate systematically.
answer choices that contain errors.
2. Repeat Step 1 until you have one answer choice left.

AGREEMENT
Agreement is one of the most heavily tested rules on the SAT. Let’s take a look at
some specific situations.

© The Princeton Review, Inc. | 135

14 Improving Sent 135 11/15/05 3:31:02 PM


Refresher Manual for the SAT

Verbs
If you have a singular subject, you need to use a singular verb. If you have a plural
subject, you need to use a plural verb.

Trim the fat

Ignore what’s between the subject and verb—it’s just there to distract you.
TRIM THE FAT.

1. While the sneakers that bear the logo of America’s


largest footwear manufacturer costs several hundred
dollars per pair, the workers who actually make them
are paid only pennies a day.
(A) that bear the logo of America’s largest foot-
wear manufacturer costs
(B) which bear the logo of America’s largest
footwear manufacturer costs
(C) who bear the logo of America’s largest
manufacturer of footwear costs
(D) that bear the logo of America’s largest foot-
wear manufacturer cost
(E) which bear America’s largest footwear
manufacturer’s logo cost
Collective Nouns

Collective nouns are singular. They need to be paired with singular verbs.

Ignore prepositional 2. The peasants fl ed when they heard that a band of


phrases when you check armed men were headed toward their village.
for agreement.
(A) a band of armed men were
(B) an armed band of men were
(C) a band composed of armed men were
(D) a band of armed men was
(E) armed men in a band was

136 | © The Princeton Review, Inc.

14 Improving Sent 136 11/15/05 3:31:03 PM


Improving Sentences

Compound Subjects

Two singular nouns joined by “and” make a plural subject. Two singular
nouns joined by “or” or “nor” make a singular subject. If a singular noun
and a plural noun are joined by “or” or “nor,” the verb agrees with the
closer noun.

3. In Hinduism, the gods Shiva and Parvati is the par-


ents of Ganesha, the elephant-headed god of wisdom.
(A) the gods Shiva and Parvati is the parents
(B) the gods Shiva and Parvati is the parent
(C) the gods Shiva and Parvati are the parents
(D) Shiva and Parvati are the gods who are the
parents
(E) Shiva is one and Parvati the other parent

4. Either my parents or my brother is going to pick me


up at the airport tomorrow afternoon.
(A) parents or my brother is
(B) brother or my parents is
(C) parents or my brother are
(D) brother or otherwise my parents are
(E) parents or my brother will

PrOnOUns

When a pronoun is underlined, determine which noun it’s replacing, and


make sure they agree: Singular with singular, and plural with plural.

5. Stage actors judge how the audience feel about a


show by how loudly they applaud their performances.
(A) feel about a show by how loudly they
applaud their
(B) feels about a show by how loudly it
applauds its
(C) feels about a show by how loudly it
applauds their
(D) feels about a show by how loudly they
applaud its
(E) feel about a show by how loudly they
applaud their own

© The Princeton Review, Inc. | 137

14 Improving Sent 137 11/15/05 3:31:04 PM


Refresher Manual for the SAT

The following pronouns are SINGULAR.

anybody everybody somebody nobody anyone

everyone someone no one anything everything

something nothing either neither each

much

The following pronouns are PLURAL.

few many both several

The following pronouns can be singular OR plural, depending on the context.

all most some none more any less

6. The safety instructions on the airplane indicated that


in the event of an evacuation, everyone should leave
their briefcases and backpacks behind.
(A) everyone should leave their briefcases and
backpacks
(B) everybody should leave their briefcases and
backpacks
(C) everyone should leave their briefcase or back-
pack
(D) everybody should leave their briefcase or back-
pack
(E) everyone should leave his briefcase or backpack

138 | © The Princeton Review, Inc.

14 Improving Sent 138 11/15/05 3:31:05 PM


Improving Sentences

MIsPLACeD MODIFIers
To avoid misplaced modifiers, make sure that modifying phrases are indeed modi-
fying what they’re supposed to modify.

If the modifier isn’t underlined, choose the subject that the modifier is ac-
tually describing. If the modifier is underlined, make it describe the sub-
ject or else turn it into a clause.

7. Barking loudly from the break of dawn until sunset,


Jim could not enjoy his retirement because of the
dog next door.
(A) Jim could not enjoy his retirement because
of the dog next door
(B) Jim could not enjoy his retirement due to
the dog next door
(C) the dog next door kept Jim from enjoying
his retirement
(D) the dog next door could not allow Jim to
enjoy being in retirement
(E) Jim’s retirement could not be enjoyed be-
cause of the dog next door

8. Having ravaged the beaches, caused billions of dol-


lars in damage, and taken dozens of lives, the gover-
nor announced that he was assembling a committee
to develop a plan to protect the state’s populace from
hurricanes.
(A) Having
(B) After having
(C) Despite having
(D) Seeing that it had
(E) Because they had

© The Princeton Review, Inc. | 139

14 Improving Sent 139 11/15/05 3:31:05 PM


Refresher Manual for the SAT

PARALLELISM
ETS will give you sentences that mix and match different parts of speech, differ-
ent tenses, different comparisons. You need to make sure everything is consistent
and parallel.

LIsTs

Items in a list should be parallel in form.

9. Jennifer’s cats liked to climb her bookshelves, shred


her furniture, and to bite her toes.
(A) shred her furniture, and to bite
(B) shred her furniture, and bite
(C) to shred her furniture, and bite
(D) to shred her furniture, and also bite
(E) to shred and to bite her furniture and

10. The soldiers who fought in the revolution had a vari-


ety of motivations: some patriotic, some mercenary,
and vengeance brought some, too.
(A) mercenary, and vengeance brought some,
too
(B) mercenary, and some brought by vengeance
(C) mercenary, and some vengeful
(D) mercenary, some because of revenge
(E) mercenary; brought by vengeance

140 | © The Princeton Review, Inc.

14 Improving Sent 140 11/15/05 3:31:06 PM


Improving Sentences

Verb Tense
Another issue with parallelism is the use of parallel verb tenses. Generally speak-
ing, the tense of verbs in a sentence should be consistent.

11. The state’s new educational priorities marginally


increased math and reading scores among this year’s
high school graduates, but critics argued that the
students would be better citizens had they learned
history and been exposed to the arts.
(A) had they learned history and been exposed
(B) if they had learned history and exposed
(C) if they have learned history and had been
exposed
(D) had they learned history and have been
exposed
(E) if they would have learned history and been
exposed

Of course, if there is a reason for verbs to be in different tenses, putting each


verb into the correct tense is more important than keeping them parallel:

12. Each of the astronauts in the Apollo program under-


went an exhaustive battery of tests by the time he was
cleared for an actual mission.
(A) underwent
(B) undergoes
(C) has undergone
(D) had undergone
(E) will have undergone

© The Princeton Review, Inc. | 141

14 Improving Sent 141 11/15/05 3:31:06 PM


Refresher Manual for the SAT

COMPArIsOns

When you spot a comparison in a sentence, make sure the items being
compared are properly comparable: Apples to apples, and oranges to or-
anges.

13. The commentators agreed that Springfi eld’s hockey


team would surely score less goals than Shelbyville
in Wednesday’s playoff game.
(A) score less goals than Shelbyville
(B) score fewer goals than Shelbyville
(C) score less goals than Shelbyville’s
(D) score fewer goals than Shelbyville’s
(E) score the least goals compared to
Shelbyville

nOUn PArALLeLIsM
Even when they aren’t explicitly part of a list or a comparison, nouns should agree
in number unless there is a compelling reason for them to disagree.

14. In ancient Rome, it was common for emperors to


adopt their chosen successor as a son.
(A) their chosen successor as a son
(B) his chosen successor as a son
(C) as a son one’s chosen successor
(D) their chosen successors as sons
(E) his chosen successors as a son

142 | © The Princeton Review, Inc.

14 Improving Sent 142 11/15/05 3:31:07 PM


Error IDs and
Improving Paragraphs

Error ID questions
Eighteen of the grammar questions on the SAT will ask you not to fix the sentence,
but merely to identify where the error is. Each Error ID question presents a sen-
tence with four underlined portions, labeled (A), (B), (C), and (D). You also have
the option “No error,” which is labeled (E). Your job is to find the error in the sen-
tence (if there is one). Here’s an example:

1. Four score and seven years ago our fathers


A
bringed forth on this continent a new nation,
B
conceived in liberty and dedicated to the propo-
C
sition that all men are created equal. No error
D E Error IDs are the quickest
type of question to tackle
Here are a few key points about Error IDs. because you don’t need to
read any answer choices.
• There is never more than one error in a sentence.
• If there is an error in the sentence, it will always be underlined. The non-
underlined portions of the sentence are always correct. Thus, you should
use the non-underlined portions to help you decide whether the under-
lined portions are correct.
• Any underlined portion of the sentence that you know is correct can be
eliminated. You will nearly always be able to determine that a few of them
are correct, so Error IDs are excellent questions for POE if you’re having
trouble.

© The Princeton Review, Inc. | 143

15 Error ID and Imp Paragraphs 143 11/15/05 3:31:10 PM


refresher manual for the sat

• Approximately one-fi fth of the sentences will have no error. Thus, you
should expect to see about 3 or 4 (E)’s out of your 18 Error IDs.

BasIc aPProach
The key to Error ID questions is to know the rules of grammar that appear on the
SAT, and recognize whether any of the underlined portions of the sentence violate
them. Follow this plan.

Read the sentence. If you spot an error right away, do the following:

1. Try to articulate what the error is, so that you avoid picking an
answer solely by ear.
2. Check the other underlined portions, just to be safe.
3. Select the answer you originally identified as an error if you’re
confident it truly is an error.
If you don’t spot an error when you fi rst read the sentence, do the
following:

1. Attack one underlined portion at a time by checking the grammar


rules that apply to each.
2. Eliminate answers that you know are grammatically correct.
3. Select the answer that violates a rule, or (E) if there is no error.

agrEEmEnt anD ParaLLELIsm


Error ID questions are just as likely to test agreement and parallelism as sentence
improvements are. Be on the lookout for these sorts of errors.

2. The documents released by the Executive

Secretariat of the State Department indicates


A
that the diplomat in question was well regarded
B C
by her superiors. No error
D E

3. Niccolo Machiavelli advised Renaissance

princes not to shy away from treachery,


A B
subterfuge, and being cruel in accomplishing
C
their goals. No error
D E

144 | © The Princeton Review, Inc.

15 Error ID and Imp Paragraphs 144 11/15/05 3:31:11 PM


Error IDs and Improving Paragraphs

4. The rules of cricket baffle many Americans but


A
are really no more complicated than baseball.
B C D
No error
E

morE aBout Pronouns


As we have seen, the fi rst thing you should check about underlined pronouns is
agreement. But there are a few other pronoun errors that ETS will occasionally
throw at you.

pronoUn amBIgUITY

There should never be any confusion or ambiguity about which noun a


pronoun is referring to. Even if it seems obvious from context, it must be
grammatically unambiguous which noun a pronoun stands for.

5. The farmers were horrified when they walked


A
through the fields of corn plants and discovered
B
that crows had been eating them. No error
C D E

6. Although they were once commonplace along


A B
our highways and in our cities, the government

has now banned billboards advertising tobacco.


C D
No error
E

© The Princeton Review, Inc. | 145

15 Error ID and Imp Paragraphs 145 11/15/05 3:31:12 PM


refresher manual for the sat

pronoUn CasE
The case of a pronoun indicates whether it stands for the subject of the sentence
or the object.

Subject Object
Watch out for it and
I me.
they—these pronouns
are most commonly the
He him.
culprits in sentences with
ambiguity errors. She went to the mall with her.

We us.

They them.

Who whom?

7. I appreciate that Tracy is only trying to help her


A
brother, but she is going to have to accept that
B
this dispute is between he and I and that her
C
interference is only making the problem worse.
D
When choosing between No error
who and whom, ask your- E
self whether you would
use he or him in place of 8. The message on my desk could hardly have
the current pronoun. If A
you’ d say he, you want been less useful: it indicated that the telephone
who; if you’ d say him, you B
want whom. had rung, but not who had called. No error
C D E

146 | © The Princeton Review, Inc.

15 Error ID and Imp Paragraphs 146 11/15/05 3:31:13 PM


Error IDs and Improving Paragraphs

idiomS and Diction


Idioms are not rules like “Verbs must agree with their subjects.” Idioms are conven-
tions of language. In English, certain combinations of words are just correct, and
others are incorrect. There’s no reason; that’s just the way it is. Some idioms you’ll
know by ear. Others, you’ll have to learn and memorize.

Many idiom questions require you to know which preposition follows a particu-
lar word, so it’s good to know the most common combinations.

...of ...to ...from ...with

responsibility
capable conform argue
composed intend comply
different
in search plan consistent
prohibit
jealous try compare
distinguish
resentful superior contrast
consist compare credit
attribute

...over ...as ...for ...about

define provide
dispute worry
regard responsible
debate concern
see qualify

...to be (no preposition)

estimate named
appears off
believe advocate

© The Princeton Review, Inc. | 147

15 Error ID and Imp Paragraphs 147 11/15/05 3:31:13 PM


Refresher Manual for the SAT

9. From the moment she stepped off the plane,


A B
Serena found that Uruguay was much different
C
as she had expected. No error
D E

10. Try and understand what I am telling you: the


A B C
company has been liquidated and your stock
D
options are worthless. No error
E

Related to idiom errors are diction errors, in which the wrong word has been
substituted for a similar-sounding word.

11. The destruction caused by the sudden storm was


A
the most incredulous thing the anchorwoman
B C
had ever seen. No error
D E

12. As he lay on the ground in agony, the injured


A
player thought less about his broken leg than
B C
about the fact that his team would now surely

loose the game. No error


D E

Improving paragraphs
Paragraph improvement questions are the least common question type, account-
ing for only six questions on the entire test. Here are a few key points about Im-
proving Paragraph questions.

• Be sure to leave time for these at the end of your POOD, since they are
only of easy or medium difficulty.
• The “20% are correct as written” rule doesn’t apply here. You often don’t
even have the option to leave the paragraph alone, and when you do, it is
almost never the correct answer.
• These passages aren’t just poorly written—they’re disaster areas. There
are many, many more flaws within each passage than you will be asked
about. Therefore, do not edit as you read.

148 | © The Princeton Review, Inc.

15 Error ID and Imp Paragraphs 148 11/15/05 3:31:13 PM


Error IDs and Improving Paragraphs

ThE BasIC approaCh

1. Skim the passage and identify the following things:


• Main Idea
• Structure
2. Read the question and go back to the passage for context.
3. POE

Keep the following ideas in mind when tackling these questions.

• Think about what the author is trying to convey. Since your job here
is to improve the paragraphs, you want to help the author get his or her
main idea across as effectively as possible. In order to do that, you need to
know what the author is trying to do.
• Pay attention to the logical flow of ideas. Many of the questions
relate to the order in which ideas are presented. You want to make sure
that the ideas flow in a logical progression, and that each part leads to the
next.
• Avoid ambiguity and wordiness. The most effective revisions will be
marked by precision of language and conciseness of expression.

ThE QUEsTIons
You’ll see three basic types of questions:

• Revision Questions: You’ll be asked to revise sentences or parts of sen-


tences. These are extremely similar to sentence improvements.
• Combination Questions: You’ll be asked to combine two sentences.
• Weird Questions: You’ll be asked to split paragraphs, insert sentences,
swap sentences, describe the relationship between sentences, or fi gure
out what topics could hypothetically precede or follow the passage.
Handling a weird question depends on what you’re specifically asked to do. Here
are some guidelines.

• If you’re asked to split a paragraph in two, look for where a new idea is
introduced. Where is there a shift to something new?
• If you’re asked to insert a sentence, focus on the connection between the
surrounding sentences. How could you connect the ideas more logically?
• If you’re asked to swap sentences, focus on the order of ideas. Where are
there ideas coming out of order?
• If you’re asked to describe the relationship between two sentences, focus
on whether they agree or disagree. Then ask how the sentences relate to
each other.
• If you’re asked to identify the best topic to hypothetically precede or fol-
low the passage, stick as closely to the passage as you can. Something
preceding the passage should tie directly into the fi rst sentence.
Something following the passage should flow directly from the last sen-
tence.
Always remember to focus on the main point of the passage.
© The Princeton Review, Inc. | 149

15 Error ID and Imp Paragraphs 149 11/15/05 3:31:14 PM


Refresher Manual for the SAT

(1) Many people think that instant messaging has 2. Which of the following is the best revision of
only been around for a few years but they are mistak- sentence 1 (reproduced below)?
en and it is not. (2) My uncle who is an engineer says
Many people think that instant messaging has
he used instant messaging when he was in college
only been around for a few years but they are
way back in the 1980’s. (3) He used a program called
mistaken and it is not.
“talk” that allowed people to send messages back and
forth to computers thousands of miles away instantly. (A) Mistakenly, many people think
(4) But only computer experts used it. (5) Also you that instant messaging has been
couldn’t send pictures. around for a few years, but it has
(6) Today almost all young people in the United not.
States use instant messaging. (7) Even if they don’t (B) Many people mistakenly think that
use computers for anything else they use it. (8) You instant messaging has only been
type messages on your computer, and they appear on around for a few years but it is
the person with whom you are conversing. (9) There not.
is a sound that tells you when a new message has ar- (C) Many people are mistaken to think
rived. (10) You can talk to many people at the same that instant messaging has only
time. (11) Many parents are surprised at how popu- been around for a few years, for it
lar instant messaging is. (12) They say why would is not.
you want to send cold text back and forth when you (D) Many people think that instant
could talk on the phone and hear the tone of the other messaging has been around only
person’s voice. for a few years, but mistakenly, it
(13) But I bet that decades ago parents couldn’t un- has not.
derstand why kids would want to talk on the phone to (E) Many people think that instant
their friends when they went to school with them and messaging has only been around
could visit them just by walking a few blocks. (14) for a few years, but they are mis-
Walking is good exercise, but not as good as biking taken.
or swimming. (15) But when you talk on the phone
you don’t have to worry about whether you look good
and you can do other things. (16) Just like with instant
messaging you don’t have to answer back right away,
you can go get a snack and write back when you feel
like it and no one thinks it’s weird. (17) And it is easi-
er to ask someone on a date with instant messaging.

1. The passage would be most improved


if which of the following sentences were
eliminated?
(A) Sentence 1
(B) Sentence 6
(C) Sentence 8
(D) Sentence 13
(E) Sentence 14

150 | © The Princeton Review, Inc.

15 Error ID and Imp Paragraphs 150 11/15/05 3:31:14 PM


Error IDs and Improving Paragraphs

3. Sentences 6 and 7 (reproduced below) could 5. Sentence 8 could best be revised by


best be combined in which of the following
(A) changing the word “with” to “to”
ways?
(B) changing the phrase “they” to “the
Today almost all young people in the United same messages”
State use instant messaging. Even if they (C) changing the phrase “and they ap-
don’t use computers for anything else they pear” to “which appears”
use it. (D) inserting the phrase “the screen of”
before the phrase “the person”
(A) Today almost all young people (E) taking it out of the second person
in the United States use instant and putting in the first person
messaging, even if they don’t use
computers for anything else. 6. This essay would most logically go on to
(B) Today, even if they don’t use them discuss
for anything else, almost all
young people in the United States (A) the author’s own dating experience
use computers for instant messag- (B) the technology involved in instant
ing. messaging
(C) Today almost all young people (C) the reasons teenagers feel more
in the United States use instant comfortable arranging dates over
messaging; even if they don’t use an instant messaging system than
computers for anything else, they in person
use them for that. (D) other sources of misunderstand-
(D) Today almost all young people in ing between teenagers and their
the United States, if they don’t parents
use computers for anything else, (E) the future of the Internet
they use instant messaging.
(E) Today almost all young people in
the United States use computers
for instant messaging, not for
anything else.

4. The best way for the author of this essay to


rearrange the paragraph breaks would be to
(A) insert a paragraph break between
sentences 3 and 4, and eliminate
the paragraph break between
sentences 5 and 6
(B) eliminate the paragraph break
between sentences 5 and 6, and
insert a paragraph break between
sentences 8 and 9
(C) insert a paragraph break between
sentences 9 and 10, and eliminate
the paragraph break between
sentences 12 and 13
(D) insert a paragraph break between
sentences 10 and 11, and elimi-
nate the paragraph break between
sentences 12 and 13
(E) eliminate the paragraph break
between sentences 12 and 13, and
insert a paragraph break between
sentence 15 and 16

© The Princeton Review, Inc. | 151

15 Error ID and Imp Paragraphs 151 11/15/05 3:31:14 PM


15 Error ID and Imp Paragraphs 152 11/15/05 3:31:14 PM
Grammar Homework

© The Princeton Review, Inc. | 153

16 Grammar Homework 153 11/15/05 3:31:18 PM


Refresher Manual for the SAT

Error IDs Drill One 5. People which are considered healthy


A B
by most doctors may still find it
1. After studying diligently for two C
A difficult to obtain affordable health
years, Alex finally realized that he D
B C insurance. No error
had scarcely no interest in biology. E
D
No error
E 6. Public buses maintain stringent
A B
environmental standards including
2. Bianca left Javier a note asking him C
A B increased fuel efficiency and cleaner
to go with Liz and she to
C emissions. No error
the upcoming symphony concert. D E
D
No error
E

7. Deb soon realized why her roommate


3. To understand fully William
A had become resentful of she and her
Shakespeare’s body of work, we A B C
friends; their parties kept him awake
must analyze not only his most
B late at night when he wanted to sleep.
famous plays such as Hamlet, but also D
C No error
his lesser-known plays, poems, E
D
and sonnets. No error
E 8. Though the memory of the recent oil
A
shortage remains, most Americans
4. Every morning Adeline has to show
A view the present abundance that they
identification at the front desk, B
currently enjoy as a sign they
ride up to the eighth floor, and then C
may again waste fuel. No error
swipes her pass through a reader D E
B
before she can enter her office.
C D 9. Emily goes to clubs frequent and
No error A
E returns home late, but is always
B C
awake and alert the next morning.
D
No error
E

154 | © The Princeton Review, Inc.

16 Grammar Homework 154 11/15/05 3:31:19 PM


Grammar Homework

10. The 1,800 islands on the St. Lawrence 15. When Chris unplugged the monitor
A A
River are counted as part of the from his computer, he discovered it to be
B B C
Thousand Islands if they stay above seriously damaged. No error
C D E
water all year and grew at least two
D
trees. No error 16. As more art historians investigate the
E A
heritage of the Chinese, he becomes
B
11. The debate over whether inoculations increasingly aware of the religious
A C
for very young children are themes that inspired ancient artists.
B D
beneficial or are they dangerous No error
C E
continues to rage. No error
D E
17. The students agreed that
A
12. Barnaby surprises most people among the candidates for governor
B
he meets because he not only of California in the recent election,
A C
loves listening to heavy metal and also Gary Coleman was the more
B C D
he enjoys watching ballet. No error qualified. No error
D E E

13. John Bubbles is usually credited


A
with the invention of rhythm
B C
tap-dancing, an art form in which
D
both heels and toes are used to

produce a syncopated sound. No error


E

14. Peter had lay on the couch all afternoon


A B
watching television before Don
C
called him for dinner. No error
D E

© The Princeton Review, Inc. | 155

16 Grammar Homework 155 11/15/05 3:31:19 PM


Refresher Manual for the SAT

Error IDs Drill Two 5. Mr. Marsh warned his strong-willed


A
daughter that unless she remembered
1. Achieving heights of twelve feet, B
A to drive slower, he would take away
the African elephant is the taller of C
B C her learner’s permit for the remainder
the two elephant species that exists D
D of the year. No error
today. No error E
E

6. As a result of the defendant’s


2. That painter has had the honor of A
A continuous refusal to demonstrate
having his work included in more B
B C even the slightest semblance of
international exhibits than other painters. C
D appropriate behavior in the court room,
No error
E the judge held him in contempt. No error
D E

3. Every day when Vinnee gets to work,


A 7. More than 80,000 animal and 45,000
she checks her voice mail messages, A
B plant species, approximately 8% of
meets with her editors, and read the local B C
C D the world’s total, they have recorded
newspapers. No error D
E in India. No error
E
4. In the eyes of many readers, The
A
New York Times, one of the nation’s 8. Few employees understood the
A
most long-standing newspapers, concept of niche marketing, and so
B B
exemplifies the ideal traits of a daily they could not account for the shop’s
C C D
newspaper: insightful reporting and slow sales. No error
D E
exceptional writing. No error
E

156 | © The Princeton Review, Inc.

16 Grammar Homework 156 11/15/05 3:31:19 PM


Grammar Homework

9. Many foods associated with 14. Jodie should of known that wearing
A A B C
European cuisines, like potatoes a tiara to work on Halloween was a
B
and tomatoes, actually originated bad idea; now everyone refers to her
D
in the Americas and were unknown as “The Queen.” No error
C E
in Europe prior to the voyages of
D 15. Carly was a skilled driver, so even
Christopher Columbus. No error A
E after having been without a car for
B C
10. Andy was hardworking and a a year, she was still comfortable
A D
prolific writer until a mysterious behind the wheel. No error
B E
illness caused him to start falling
C
asleep, sometimes in the middle of a 16. Rising domestic gas prices have
D A
conversation. No error increased dramatically in the
E B
past several years, causing many

11. “Remind me never to sign up for businesses to spend a greater


A C
another psychology class,” Linus percentage of their budgets on
D
said petulantly, “because the teacher transportation. No error
B E
expects far too much reading of us.”
C D
No error 17. Although the work of both artists
E A
has been displayed at the gallery,
B
12. The range of electric cars has always been only one has managed to sell
A B C D
smaller than gasoline-powered cars. anything. No error
C D E
No error
E 18. I was so shaked up after the tractor-trailer
A
crashed into my car on the highway
13. The best students in my class always B
that I avoided driving for several
considers producing high quality C D
A weeks. No error
work important, regardless of E
B
whether it affects their grades. No error
C D E

© The Princeton Review, Inc. | 157

16 Grammar Homework 157 11/15/05 3:31:20 PM


Refresher Manual for the SAT

19. The data entry work was given to Julie 24. The class president stressed that it is
A A
because her typing speed is nearly imperative that we maintain a
B C B
double that of Martha. strong reserve balance in our
D
No error school improvement fund since we are
E C
responsible to plant additional
D
20. Vladimir Nabokov, the popular Russian- trees in the common area. No error
A E
American writer, was born in
B
St. Petersburg, but had moved 25. While it is often expensive to attend
C A
to England to attend college at Cambridge. a live soccer match, it is much more
D B
No error exciting and allows you to meet
E C D
other fans. No error
E
21. The extraordinary attention to detail needed
A
by pilots in flight are essential to 26. The team should have asked us for
B C A B
their survival. No error clarification since no one
D E C
understands the rules better than we.
D
22. Everyone in the junior class are going to be No error
A E
eligible for a chance to win a
B C
scholarship toward a summer travel

program offered by the language


D
department. No error
E

23. Although the venom’s antidote had been


A B C
tested only in the laboratory, it

received a unique human trial when


D
the rare spider bit one of the guests.

No error
E

158 | © The Princeton Review, Inc.

16 Grammar Homework 158 11/15/05 3:31:20 PM


Grammar Homework

Improving Sentences Drill 4. Since one should not count out the Yankees
early in the season, as they always get
stronger by the middle of August.
1. None of his old clothes fit him, so much
(A) Since one should not count
weight having been lost on the new diet.
(B) One ought not count
(A) so much weight having been lost (C) Since one ought not count
on the new diet (D) One had ought not to count
(B) on the new diet having lost so (E) One should not be counting
much weight
(C) having the new diet losing so 5. Martin Luther King Jr., who endured
much weight countless humiliations and hardships, forcing
(D) on the new diet and losing so even his detractors to acknowledge the
much weight sincerity and strength of his convictions.
(E) since he lost so much weight on
the new diet (A) forcing
(B) he forces
2. The Surgeon General reports that while (C) forced
cigarette smoking is less popular than at any (D) and forcing
(E) to force
time since 1957, it is still the cause of over
100,000 cancer deaths a year.
6. The admissions officer put an acceptance
(A) it is still the cause of letter in one of the envelopes and in the other
(B) it is still the result of was put a rejection letter.
(C) causing
(D) being the cause of (A) and in the other was put a rejection
(E) while being the cause of letter
(B) with the other put in a rejection
3. Although he didn’t consider himself lucky, letter
(C) and the other was put in a rejection
he won a teddy bear for his brother at the
letter
carnival.
(D) and the thing he put in the other
(A) he won a teddy bear for his brother was a rejection letter
at the carnival (E) and a rejection letter in the other
(B) his brother had won by him a
teddy bear at the carnival 7. Never again would Jerry attempt to jump out
(C) because he won his brother a teddy of an airplane without wearing a parachute.
bear at the carnival
(D) still he will be able to win a teddy (A) to jump out of an airplane without
bear for his brother at the carnival wearing a parachute
(E) he won, for his brother at the car- (B) to jump out of an airplane without
nival, a teddy bear wearing parachutes
(C) to jump out of an airplane, with
Jerry not wearing a parachute
(D) to jump out of an airplane having
no parachute
(E) jumping out of airplanes without
parachutes

© The Princeton Review, Inc. | 159

16 Grammar Homework 159 11/15/05 3:31:20 PM


Refresher Manual for the SAT

8. The primary resource used to power industry 11. When she picked up the phone and a voice
was initially water; eventually it progressed asked, “May I speak to Ms. Gardner?” Rose
to steam, oil, and finally electricity. answered, “This is she.”
(A) oil, and finally electricity (A) asked, “May I speak to Ms. Gard-
(B) oil burning, and finally the use of ner?” Rose answered, “This is
electricity she.”
(C) oil, and finally electric power (B) asked, “May I speak to Ms. Gard-
(D) oil, and finally the use of electric- ner?” Rose answered, “This is
ity her.”
(E) oil used for power, and finally (C) is asking, “May I speak to Ms.
electric power Gardner?” Rose answered, “This
is her.”
9. Daniel, while being an honor roll student, has (D) is asking, “May I speak to Ms.
nearly irritated every teacher in the school. Gardner?” Rose is answering,
“This is she.”
(A) Daniel, while being an honor roll (E) asking, “May I speak to Ms. Gard-
student, has nearly irritated every ner?” Rose answering, “This is
teacher in the school. her.”
(B) Daniel, although an honor roll
student, has irritated nearly every 12. People are free to make their own decisions,
teacher in the school. but there will always be rules in their life that
(C) Daniel, although an honor roll
must be followed.
student, had been nearly irritating
to every teacher in the school. (A) will always be rules in their life
(D) While being an honor roll student, that must be
Daniel has nearly irritated every (B) were always rules in their lives,
teacher in the school. which must have been
(E) Even when an honor roll student, (C) will always be rules in one’s life
Daniel, in school, is irritating that must be
nearly to every teacher. (D) will always be rules in their lives
that must be
10. Frank Lloyd Wright will be considered as (E) would always be rules in their
one of America’s greatest architects because lives, which must be
of works such as “Fallingwater,” a house
built over a waterfall. 13. The Beat poets of the 1950’s broke new
literary ground with their innovative use
(A) will be considered as one of Amer- of language, choosing of subjects, and they
ica’s greatest architects because
expressed taboo emotions.
of
(B) has been considered by America as (A) choosing of subjects, and they
a great architect due to expressed taboo emotions
(C) is considered one of America’s (B) choosing of subjects, and express-
greatest architects because of ing of taboo emotions
(D) considered as one of America’s (C) choice of subjects, and they ex-
greatest architects because of pressed taboo emotions
(E) is considered great by America’s (D) choice of subjects, and expression
architects due to of taboo emotions
(E) subject choice, and the taboo of
expressed emotions

160 | © The Princeton Review, Inc.

16 Grammar Homework 160 11/15/05 3:31:21 PM


Answers &
Explanations

MATH Homework

Plugging In
page 54

6. B Plug in 4 for p. This gives you (4 + 5) = 9 and (4 – 7) = –3. The question asks
for the difference, so 9 – (–3) = 12.

6. B Plug in the answers, starting with (C). As you work, just remember that the
total number of puppets sold must equal 25, and the total sold for $30.

sml × $1 each lg × $2 each total


(A) 25
(B) 20 $20 5 $10 $30 ¸
(C) 15 $15 10 $20 $35 too big
(D) 10
(E) 5

© The Princeton Review, Inc. | 161

17 Answers Exp 161 11/16/05 4:13:21 PM


Refresher Manual for the SAT

6. C Plug in the answers, starting with (C). The numbers in the answers are the
possible weights of the block at the beginning of the day on Tuesday.

orig lbs. 2/3 melted 1/2 melted


(A) 540

(B) 480
2 1
(C) 360 360 × = 120 120 × = 60 ¸
3 2
(D) 180

(E) 20

7. A Cross out the “in terms of” phrase and plug in! Let’s say that x = 12, which

is the number of pages printed by Printer A in one hour. Printer B prints


12
at one-third this rate, so it prints at a rate of = 4 pages per hour. Work-
3
ing together, in one hour they will print 12 + 4 = 16 pages per hour, and

16 × 3 = 48 pages in three hours. This is your target. Only (A) works:

4x = 4 × 12 = 48. Watch out! If you picked (D), you chose a partial answer.

8. C Plug in the answers, starting with (C). The answers are possible values of c,
the number of clients the company started with. Multiply by three for each
year. Did you also notice that (E) is way too big? If so, cross it off.

c yr 1 yr 2 yr 3 yr 4 yr 5
(A) 5
(B) 15
(C) 45 × 3 = 135 × 3 = 405 × 3 = 1,215 × 3 = 3,645 × 3 = 10,935 ¸
(D) 729
(E) 3,645 too big!

9. A Cross out the “in terms of” phrase and plug in! If j = 20, then Alicia is 5 years
younger, so a = 15. In 8 years, Alicia will be 15 + 8 = 23. This is your target.
Only (A) works: j + 3 = 20 + 3 = 23.

9. C Cross out the “in terms of” phrase and plug in! Plug in for a and the other
two angles in that triangle—be sure that your numbers add up to 180°.
If a = 50, make angle CAG = 110 and angle CGA = 20. That makes
b = 160 and x = 70, which is your target. Only (C) works:
180 + a – b = 180 + 50 – 160 = 70.

162 | © The Princeton Review, Inc.

17 Answers Exp 162 11/16/05 4:13:23 PM


Answers & Explanations

page 55

9. C Plug in a number such as 5, which is neither a factor nor a multiple of the


numbers in the answer choices—this will minimize the chances that more
than one answer will work. If a = 5, only (C) produces an even integer:
2a = 2 × 5 = 10.

10. E Plug in! If Bert is 10, and Ernie is three times as old as Bert, then Ernie is
30. Ernie is 4 years younger than Roger, so 30 + 4 = 34, which is your target.
Only (E) works: Roger’s age is (3)(10) + 4 = 34.

10. C Plug in the answers, starting with (C).

the (1/4) the (1/2) the


number number number
(A) –12
(B) 6
(C) 12 3 6 3 is 3 less than 6 ¸
(D) 16
(E) 18

11. D Plug in 5 feet for m, and choose a number for w that can be easily converted

to feet, such as 36 inches, which equals 3 feet. So if m = 5 (feet) and w = 36

(inches), then Marlene is 2 feet taller than Albert. That makes 2 your target
12 m − w (12) ( 5 ) − ( 36 ) 60 − 36 24
answer. Only (D) works: = = = = 2.
12 12 12 12

11. A Cross out “in terms of” and plug in! Start with Circle A and its radius, v. If

v = 4, then the area of Circle A = π(42) or 16π. If the area of Circle B is twice

that of Circle A, then the area of B is 16π × 2 = 32π. 32π = r2π, and the radius

of B is w, so w2 = 32, and w = 32. Reread the question: We are looking for


32
the value of v, which is 4. Only (A) works: = 16 = 4.
2

© The Princeton Review, Inc. | 163

17 Answers Exp 163 11/16/05 4:13:24 PM


Refresher Manual for the SAT

page 56

11. A Cross out the “in terms of” phrase and plug in! If you plug in a = 2, then

y = 5 × 2 = 10, and x =
(10) ( 2) + 6 = 26 = 13 , which is your target. Only (A)
4 4 2
works: 10 + 3 = 13 . Notice that sometimes the numbers don’t work out to be
2 2
integers on harder questions. Don’t restart the question unless the numbers

are really messy—a fraction isn’t too hard to work with here.

1 1
12. D Plug in for z to begin. If z = 3, z2 = 9 = . Solve for x = . Therefore,
x 9
1
x2 = . Only (D) works.
81
12. D Plug in the answers, starting with (C).

m (2m)2 m3 Does (2m)2 = m3 ?


(A) 1
(B) 2
(C) 3 (2 × 3)2 = 62 = 36 33 = 27 36 ≠ 27 X
(D) 4 (2 × 4) = 8 = 64
2 2
4 = 64
3
64 = 64 ¸
(E) 6

13. A Plug in the answers, starting with (C). Remember: the weight of the barrel

plus the weight of the water must equal 20, and the weight of the barrel must
1
be the weight of the water.
4
barrel + water = 20 Does (1/4)weight
of water = weight of barrel?
1
(A) 4 16 × 16 = 4 ¸
4
1
(B) 5 15 × 15 ≠ 5 X
4

1 × 5 ≠ 15
(C) 15 5 X
4

(D) 16

(E) 80

164 | © The Princeton Review, Inc.

17 Answers Exp 164 11/16/05 4:13:29 PM


Answers & Explanations

13. C Plug in the answers, starting with (C). What do we know from the question?
That 20% of students study only physics and 25% study only chemistry.
Therefore, the remaining 55% study biology, so the number of students
studying biology must equal 55% of some integer. Plug in and look for the
answer that produces an integer value for t. Eliminate any answer that gives
you a non-integer total.

biology t (total)
(A) 4
(B) 9
55
(C) 11 11 = t , so t = 20 ¸
100
(D) 15
(E) 20

13. A You can solve for the variable here, but it’s safer to plug in the answers, start-
ing with (C).

a
2
3
 1 1 1
 8  3
1 64 = 4 = 1
(A) = ¸
8  1 1 1
2 
 8 4 4

2
3
 1 1
1  2  3

(B) = 4 ≠1 X
2  1 1
2 
 2

( 2)
2
3 3
(C) 2 2 X
= ≠1
2 2 2 2

3
22 3
4
(D) 2 = ≠1 X
2× 2 4

3
82 3
64 4
(E) 8 = = ≠1 X
2×8 16 16

14. E Cross out “in terms of” and start plugging in for c. If c = 3, then (8)(3) = 24.
1
24 = 4b, so b = 6. Also, 24 = a, so a = 48. Reread the question: It asks for
2
the value of a + b, so 48 + 6 = 54, your target. Only (E) works: You plugged

in c = 3, so (18)(3) = 54.

© The Princeton Review, Inc. | 165

17 Answers Exp 165 11/16/05 4:13:34 PM


Refresher Manual for the SAT

page 57

14. A Plug in the answers, starting with (C). Remember that the final result should
be $350.

minus clothes (2/5 minus DVD


savings = $350?
of savings) (1/4 of savings)
(A) $1000 – 400 – 250 = $350 ¸
(B) $900
(C) $800 – 320 – 200 = $280 X
(D) $600
(E) $500

15. A Plug in the answers, starting with (C). Use POE and plug in until you have
only one answer remaining. Remember that you are looking for answers that
produce integers so you can eliminate them.

n=5 n = 20
a = 200 a = 200
n 5 20
(A) ¸
a 200 200

a 200
(B) = 40 X
n 5

a 200
(C) = 20 X
2n 10

2a 400
(D) = 80 X
n 5

n2 25 400
(E) =2 X
a 200 200

15. C Using your calculator, plug in the answers in I/II/III and see which ones make
1

the expression equal an integer. Only II does:


( 3) ( 2) 3
= 3.
2
( 2) ( 2)−
3

16. D Cross out the “in terms of” phrase and plug in! Let’s say w = 3, so y = 4 and
x = 6. Using these values, 5x + 6w = (5 × 6) + (6 × 3) = 30 + 18 = 48, your
target. Only (D) works: 12y = 12 × 4 = 48.

166 | © The Princeton Review, Inc.

17 Answers Exp 166 11/16/05 4:13:40 PM


Answers & Explanations

1
16. B Plug in! Let’s say a = and b = –2. II and II can be eliminated, but I must
2
1
stay: Plugging our values into a × a–1 × b = b gives us × 2 × –2 = –2. If you
a 2
think about it, a × a–1 will always produce = 1, so I must always be true.
a
17. C Plug in for the consecutive integers: 2, 3, 4, 5, 6, and 7. s = 2 + 3 + 4 = 9, and
5 + 6 + 7 = 18, which is your target. Only (C) works: s + 9 = 9 + 9 = 18.

17. D Plug in the answers, starting with (C). Remember that the number correct
and incorrect must add up to 93.

number number
score = 247?
correct incorrect
(A) 71
(B) 77

(C) 82 93 – 82 = 11 82(3) – 11(1) = 235 X

(D) 85 93 – 85 = 8 85(3) – 8(1) = 247 ¸

(E) 90

© The Princeton Review, Inc. | 167

17 Answers Exp 167 11/16/05 4:13:42 PM


Refresher Manual for the SAT

page 58

18. A Plug in! If t = 100 and x = 6, then set up the proportion and solve. Let’s use

the variable z to represent the number of oranges we are solving for. Remem-

ber that x is 6 dollars, so you’ll need to convert this value to 600 cents to

solve, but use the value 6 when you plug into the answers to find your target.
t = 100 cents 600 cents
= . Cross multiply and solve for z to find that for
5 oranges z oranges
500 × 6
x = $6, you can buy 30 oranges, your target. Only (A) works: = 30 .
100

18. B Plug in the answers, starting with (C). You can see right away that the

numbers get smaller the more times you multiply by a. Therefore, a must

be smaller than 1, so you can eliminate (C), (D), and (E). Fill in the terms
1
that you are given, and fill in the missing numbers using as the multiplier.
2
Only (B) follows the correct pattern.

a 1st term 2nd term 3rd term 4th term


1
(A)
8

1 1 1 1
(B) 20 × = 10 × = 5× = 2.5 ¸
2 2 2 2

(C) 2 too big X

(D) 4 too big X

(E) 8 too big X

168 | © The Princeton Review, Inc.

17 Answers Exp 168 11/16/05 4:13:46 PM


Answers & Explanations

18. D Plug in the answers, starting with (C). Divide Andy’s amount by 4 to get Chris’
amount.

Is Andy’s amount
Andy Chris Andy – 10 Chris + 10
twice Chris’?

(A) 20

(B) 40

1
(C) 50 12.5 40 22.5 22.5 ≠ × 40 X
2

1
(D) 60 15 50 25 25 = × 50 ¸
2

(E) 80

19. E Plug in numbers that are easy to work with. Let’s say that an item originally

costs $10. 6% tax on $10 is 60 cents, so after tax, an item that originally costs

$10 will cost t = $10.60. Reread the question to know what to look for: The

target is the price before tax was added, which was $10. Plug in $10.60 for t
10.60
in the answers to find the one that works. Only (E) does: = 10.
1.06

© The Princeton Review, Inc. | 169

17 Answers Exp 169 11/16/05 4:13:47 PM


Refresher Manual for the SAT

page 59

19. A Plug in a simple value for x that is between 0 and 1, and which you can
1 1
easily find the square root of: works well. If x = , then x2 = 1 and
4 4 16
1
x = . Put the values in order from smallest to largest, as they are listed
2 1 1 1
in the answer choices: < < , so x2 < x < x , or (A).
16 4 2
19. B What makes this question tough is that you need to consider numbers that
you may not think of at first glance. Using the average pie, we know that the
total of our three numbers must equal 9 × 3 = 27. However, nowhere does it
say that these numbers have to be positive. Therefore, try plugging in some
“weird” numbers, such as –30, 27, and 30. Their sum is 27, and two are greater
than 11, so these are acceptable. We can clearly eliminate I and III. And since
one of the statements has to be true, it must be II, or (B).

170 | © The Princeton Review, Inc.

17 Answers Exp 170 11/16/05 4:13:51 PM


Answers & Explanations

Geometry
page 60

1. 90°

2. 180°

3. 180°

4. 360°

5. 360°

6. The measures of two angles across from each other when two lines intersect are
equal.

7. To bisect an angle or line segment means to divide it in half.

8. Two lines that are perpendicular form two right angles.

9. Two lines that are parallel never intersect.

10. The angle across from the longest side in a triangle is the largest angle in the
triangle.

11. The angle across from the smallest side in a triangle is the smallest angle in the
triangle.

page 61

12. Angles across from equal sides in a triangle have equal measures.

13. Isosceles triangles: (1) have two equal sides and (2) have two equal angels.

14. Equilateral triangles: (1) have three equal sides and (2) three equal angles that
each measure 60°.

15. In parallelograms: (1) opposite sides are equal and parallel, and (2) opposite
angles are equal.

16. A parallelogram is a rectangle when it has four right angles (a rectangle is a special
kind of parallelogram).

17. A rectangle is a square when it has four equal sides (a square is a special kind of
rectangle).

18. The diameter is twice as long as the radius.

19. Area of a parallelogram = base × height

1
20. Area of a triangle = base × height
2
21. Base and height must be perpendicular.

© The Princeton Review, Inc. | 171

17 Answers Exp 171 11/16/05 4:13:52 PM


Refresher Manual for the SAT

22. Area of a circle = πr2

23. Find the perimeter of any polygon by adding up all its sides.

page 62

24. Circumference of a circle = 2πr OR πd

25. a2 + b2 = c2, where a and b are the sides and c the hypotenuse of a right tri-
angle.

26. ETS’s favorite Pythagorean triples are: 3:4:5, 6:8:10, and 5:12:13.

27. Volume = length × width × height, or V = lwh

y2 − y1 rise
28. Slope = =
x2 − x1 run
29. Ratio of sides of a 45°-45°-90° triangle = 1 : 1 : 2

30. Ratio of sides of a 30°-60°-90° triangle = 1 : 3 : 2

31. This information is in the box at the beginning of each math section.

32. Don’t trust figures not drawn to scale; try to redraw the figure more accurately.

33. If there is no figure, try to draw one.

34. If there are variables in the answer choices of any question, PLUG IN!

35. Ballparking is estimating and eliminating unlikely answers.

172 | © The Princeton Review, Inc.

17 Answers Exp 172 11/16/05 4:13:54 PM


Answers & Explanations

page 63

180
1. x = = 60
30
2. x = 360 – 40 = 320

3. y = 180 – 110 = 70

4. e + f = 110 + 30 = 140

5. x = 150

6. y = 115

page 64

7. area = 6

8. area = 9 3

9. x = 20

10. area of circle with circumference 12π = 36π (d = 12, r = 6)

1
11. area of shaded region = 8π (r = 8, so area = 64π. 45° = of 360°, so area of shaded
8
1 1
region is of total area: 64π × = 8π.)
8 8

© The Princeton Review, Inc. | 173

17 Answers Exp 173 11/16/05 4:13:56 PM


Refresher Manual for the SAT

page 65

7. D If you recognize that these triangles are similar, then (D) is the easy choice.
If not, plug in for x, y, and z. Let’s say x = 4, y = 6, and z = 8. If so, then the
perimeter is 4 + 6 + 8 = 18. Using these numbers, you find that the sides
of Triangle II are 6, 9, and 12, and that the perimeter of Triangle II is 27.
27 3
= .
18 2

9. E Plug in the answers, starting with (C). Remember that the angle measures
must add up to 180.

b a c = 180?

(A) 30

(B) 45
1
(C) 50 × = 25 30 50 + 25 + 30 =105 X
2
1
(D) 60 × = 30 30 60 + 30 + 30 = 120 X
2
1
(E) 100 × = 50 30 100 + 50 + 30 = 180 ¸
2

4 −1
10. E Plug in the answer choices, starting with (C). Only (E) works: = 3.
3−2
11. D Fred’s theorem tells you that the angle adjacent to y is 49°. Using the rule of
180, we also know that x + y + 49 = 180. So, x + y = 131.

page 66

11. C The triangles created by the cross-section of the cones are all similar. There-

fore, we can set up a proportion to find what we need. Just read carefully:

the smallest cone is 12 inches high and has a base diameter of 3, while the

largest has an unknown height and a base radius of 3, so the base diameter
12 x
of the largest cone is 6. = , so x = 24.
3 6
12. D Surface area = 6s2 where s is the length of a side. If the surface area here is
96, then 96 = 6s2, so 16 = s2. The value of s is therefore 4. Volume is simply
s3, so the volume of this figure is 43 or 64.

174 | © The Princeton Review, Inc.

17 Answers Exp 174 11/16/05 4:14:00 PM


Answers & Explanations

12. E Cross out the “in terms of” phrase and plug in! Remember to follow the rules
of geometry as you work. Let’s say x = 40, so the angle adjacent to x must be
50°. Now focus on triangle BEC. Given that BE = CE, angle EBC is also 50°,
so y must be 80°. Reread the question: You want to find y, so your target is
80. Plug in x = 40. Only (E) works: 2(40) = 80.

12. A We know that point Q must fall somewhere on segment AB, so eliminate any
answer that doesn’t have an x coordinate equal to –8. That leaves us with
only (A) and (B). If PQ is to bisect the square, it must pass through the
origin and hit at AB below the x-axis, so the y value must also be negative.
Eliminate (B). Drawing point Q is very helpful.

© The Princeton Review, Inc. | 175

17 Answers Exp 175 11/16/05 4:14:01 PM


Refresher Manual for the SAT

page 67

13. 21 Start with what you know. Perimeter = 2l + 2w. In this problem, l = 3, so
solve for w: 20 = (2)(3) + 2w, so 20 = 6 + 2w, 14 = 2w, and w = 7. Reread
the question: We need the area of the figure, so 3 × 7 = 21.

13. E If EF is 6, then the distance from E to the point at which EF intersects a ver-
tex of the smaller square is 3. Focus just on the distance from E to midpoints
of EF and DE and the isosceles right triangle that is formed there. The
hypotenuse of that right triangle is 3 2 , which is also the length of a side of
the smaller square inside DEFG. Therefore, the perimeter is 4 × 3 2 = 12
2.

14. D Draw, then draw again!

0 5 12 16
Q R S T

0 5 8 12
Q R T S

–2 0 2 5
S Q T R

–6 –2 0 5
T S Q R

14. C Draw the figure as described. It should look something like this:

2 2

Given the information you have, you can find the area of one of the little tri-

angles inside, then figure out the area of the square from there. If the radius
1
is 2, then the area of each little triangle is × 2 × 2 = 2. Since the square
2
consists of 4 such triangles, the area of the square is 4 × 2 = 8.

14. E If angle SOP measures 100°, then the adjacent angle POQ must measure 80°.
Since segments OP and OQ are both radii, they are equal in length. Equal
sides of a triangle are opposite equal angles, so angles OPQ and OQP are
equal, and the sum of their angle measures is 100°. Reread the question: What
is the measure of OPQ? This angle measure is 100 ÷ 2 = 50°.

176 | © The Princeton Review, Inc.

17 Answers Exp 176 11/16/05 4:14:05 PM


Answers & Explanations

page 68

14. A Notice that the triangles in the figure given are similar. Just make sure that
the numbers you plug in adhere to the ratio given in the problem. Let’s say
AC = 5 and CD = 15. Using the Pythagorean triples ETS loves to use, plug in
BC = 3, AB = 4, CE = 9, and ED = 12. Find the areas, then divide to get the
fraction:

1
area ABC ( 4 ) ( 3) 6 1
= 2 = =
area DEC 1 54 9 .
(12) ( 9 )
2

14. D Draw the figure and count up the triangles. Your figure should look something
like the one below. Notice both the smallest triangles and those formed by
combining two small triangles together.

• •

15. 76 Since both OP and OQ are radii, they are equal in length, making this triangle
isosceles. Equal sides are opposite equal angles, so both angles OPQ and OQP
equal 52°. Reread the question: What is the value of x? 52 + 52 = 104, so
180 – 104 = 76.

15. 0 Plug in for the variables, making sure to follow the rules of geometry. Let’s
say x = 40, y = 50, z = 60, and w = 30. Therefore, (x – z) + (y – w) =
(40 – 60) + (50 – 30) = –20 + 20 = 0.

© The Princeton Review, Inc. | 177

17 Answers Exp 177 11/16/05 4:14:06 PM


Refresher Manual for the SAT

page 69

16. A Plug in! Let’s say x = 10, so the radii of the partial circles are all equal to 5.

The area of the square is 102 = 100. By adding together the four unshaded

regions, you get a whole circle with r = 5, so find that area and subtract

it from 100 to get the answer you need: Area of the circle = π52 = 25π, so

the unshaded area is 100 – 25π. This is your target. Only (A) works: 102
π 100 π
(1 – ) = 100 – = 100 – 25π.
4 4
17. B Draw the figure and write down the coordinates. The distance between points
A and D is 8, so the length of the two sides added together is 16. The total
perimeter is 38, so the other two sides must sum to 38 – 16 = 22. Therefore,
the distance between points A and B is 11, and  f = 9.

18. 150 Focus on the small triangle formed where line l1 meets the other two
lines. The angle adjacent to the 140° must measure 40°. The angle at the bot-
tom measures 110°, since its adjacent angle measures 70° (Fred’s theorem
tells you that). The third, smallest angle must therefore measure 30°. Reread
the question: What is the value of x? Since it is adjacent to the 30° angle, it
must be 180 – 30 = 150.

18. D Sometimes you have to use the Pythagorean theorem! Draw a line from T to
V. Now you have two right triangles that share a hypotenuse. First calculate
TV: 32 + 52 = TV2 = 34. Now do the same with the other triangle: x2 + 22 = TV2,
which we now know is equal to 34. Therefore, x2 + 22 = 34, so x2 = 30, and
x = 30 .

page 70

19. E Use Pythagorean theorem: Since we know that AC = 2 and CB = 1,


22 + 12 = AB2 = 5. Therefore, AB = 5 . Notice you could have eliminated (B)
immediately, because the x value must be 0. You could have also ballparked
to eliminate (A) and (C), since the distance from A to B must be greater than
the lengths of the other two sides of the triangle.

−3 −2 − ( −8 )
19. 15 Use the points provided in the slope formula: = . Simplify to get
8 −1 − x
−3 6 , then cross-multiply and solve. –3(–1 – x) = 48, so 3 + 3x = 48,
=
8 −1 − x
3x = 45, and x = 15.

19. 8 Here we are looking for the greatest difference between the lengths of sides,
so use the smallest number that you can for the shortest side. Since the
lengths must be integers, that makes the shortest side equal 1. A triangle
with sides of 9, 9, and 1 fits our restrictions, so the greatest difference is
9 – 1 = 8. If you thought that the triangle could have sides of 1-1-9, remember
that the third side of any triangle must have a length between the sum and
the difference of the other two sides.

178 | © The Princeton Review, Inc.

17 Answers Exp 178 11/16/05 4:14:10 PM


Answers & Explanations

Other Approaches
page 71

5. A Put the numbers is order: 3, 4, 7, 12, 14, 17, 20. The median is 12. The aver-
3 + 4 + 7 + 12 + 14 + 17 + 20 77
age is 11, since = = 11. 12 – 11 = 1.
7 7
6. B Use the average pie. If the average of 3 numbers is 8, then their sum is
8 × 3 = 24. If the third number is 10, then the sum of the first two is
24 – 10 = 14.

8. A Use the average pie. If the average of these 4 numbers is 20, then their sum
is 4 × 20 = 80. Therefore, 80 = 21 + 12 + 37 + y, so y = 10.

10. B Use the average pie. If the average of 3 numbers is 26, their sum is
3 × 26 = 78. Therefore, 33 + 17 + x = 78, so x = 28.

10. A Use the average pie. If the average of 6 numbers is 13, then their sum is
6 × 13 = 78. Therefore, 4y + 34 = 78, so 4y = 44, and y = 11. Did you notice
that all the other answers were also wrong because y must be less than 13
for the average to be 13?

11. E Which one shows up the most time? .280.

page 72

12. B Use the average pie. If 4 + 7 + 19 + x + y = 60, then x + y = 30. Reread the
question: What is the average of x and y? 30 ÷ 2 = 15. Watch out! (E) is a Joe
Bloggs answer.

13. 10 Use the average pie. If 5 + 17 + 18 + x + y = 60, then x + y = 20. Reread the
question: What is the average of x and y? 20 ÷ 2 = 10.

14. C (C) is the definition of mode.

16. C Cross out the “in terms of” phrase and plug in! Let’s say that x = 4, so y = 8
4 + 8 + 12
and z = 12. The average of these three numbers is = 8, which is
2 3
your target. Only (C) works: z =2 × 12 = 8.
3 3
18. E List the numbers: x, 7, 8, 9, 13, and 17 where x is the age of the sixth student.
There are an even number of members in this list, and the median is 10, so
the median must be the average of 9 and x, so x = 11.

© The Princeton Review, Inc. | 179

17 Answers Exp 179 11/16/05 4:14:12 PM


Refresher Manual for the SAT

page 73

5 20
6. B Translate: x = × × 300 . So, x = 3.
100 100
80
10. B The new price must be 80% of the original price: × 23 = x. So,
x = 18.40. 100

11. B 24 – 3 = 21 cars are unsold. What percent of the cars are unsold? Translate:
x
• 24 = 21. So, x = 87.5.
100

13. C Plug in! Let’s say that she must take 100 credits. Her first year she takes

25%, or 25, credits, and she has 75 remaining to take. In her second year, she
40
takes 40% of the remaining credits, or × 75 = 30. Now she has taken
100
25 + 30 = 55 credits, and she still needs to take 45 more. Watch out! (D) and

(E) are the Joe Bloggs answers.

14. B Use bite-sized pieces. Remember that the 30% discount is for shoes only.
30
× $20 = $6 discount. $6 is what percent of the total amount she spent?
100
x
Translate: $6 = • $60. So, x = 10.
100
x
15. D Plug in! Let’s say m = 8 and n = 6, then translate: 6 = • 8. So, x = 75.
100
page 74

16. D Plug in! Let’s say the book is $100. An increase of 10% would add $10 to the
price, making the new price $110. An additional 10% of this new number
would add $11 to the price, making the final price $121. This is the same as
adding 21% to the original price. Watch out! (C) is the Joe Bloggs answer.

17. B To find percent increase or decrease, remember the formula:

difference
× 100. Using the numbers in the problem, we get:
original

385 − 350 × 100 = 10%. Watch out! (C) is the Joe Bloggs answer.
350

14. 800 The problem says that $56 is 7% of the price, so translate:
7
56 = • x. So, x = 800.
100

180 | © The Princeton Review, Inc.

17 Answers Exp 180 11/16/05 4:14:12 PM


Answers & Explanations

page 75

= 2 × 14 =
2a 28 14
1. E Plug in a = 14 and b = 6. = .
b 6 6 3
2. C Use the ratio box. Plug in what you know, and add across top. How do you
get from 4 to 12? Multiply by 3, so you need 1 × 3 = 3 males at a minimum.

male female total


1 3 4
×3 ×3
3 12

3
8 cups soda
5 cups soda 4
13. C Set up your proportion and solve: = . Cross-mul-
tiply: 5x = 105, so x = 21. 12 guests x guests

14. D Use the ratio box. Plug in what you know, and add across top. How do you
get from 6 to 120? Multiply by 20, so the longer portion is 100 and the shorter
portion is 20, and the difference is 100 – 20 = 80. Watch out! (E) is a partial
answer.

short long total


1 5 6
× 20 × 20 × 20
20 100 120

16. E Just write each ratio as a fraction of red over white, then calculate the decimal
3
value. (E) is largest: = .75.
4
page 76

2
15. or .153 Use the ratio box and plug in! Let’s say that the mixture is 2 parts
13
sulfur, so there are 10 parts charcoal and 1 part saltpeter. Add across the
2
top: the total is 13. Sulfur is 2 parts out of 13, or .
13
sulfur charcoal saltpeter total
2 10 1 13

15. 176 Use the ratio box. Plug in what you know, and add across top. How
do you get from 10 to 440? Multiply by 44, so there are 176 boys.

teacher boy girl total


1 4 5 10
×44 ×44 ×44 ×44
176 440

© The Princeton Review, Inc. | 181

17 Answers Exp 181 11/16/05 4:14:13 PM


Refresher Manual for the SAT

18. D Use the ratio box. Plug in what you know, and add across top, only this time
include the information about the cost of the books, too. For every paperback
purchased, three hardcovers are also purchased, for a total of four books. How
much do these four books cost? (3)($4) + (1)($2) = $14. How do you get
from $14 to $42? Multiply by 4, so the teacher purchased 12 books total.

paperback hardcover total


3 × $4 1 × $2 4 books costing $14
×3 ×3 ×3
12 books costing $42

page 77

3 12
3. D Set up the proportion and solve: = . 3x = 60, so x = 20.
5 x
$4.00 $15.00
4. C Set up the proportion and solve: = . 4x = 3.60, so x = $0.90.
$0.24 x
155 words x
13. 124 Set up the proportion and solve: = . 5x = 620, so
x = 124. 5 mins 4 mins

450 miles 90 miles


14. 36 Set up the proportion and solve: = . 450x = 16200, so
x = 36. 180 mins x

page 78

9. A Plug in the answers. Look for an answer that, when plugged into the equa-
tions provided as the x value, makes y = 0. Only (A) works: y = x2 – 16 when
q (the x coordinate) is –4 and y = 0.

12. A Re-write the equation in the question as well as those in the answer choices

so that they are in y = mx + b form. The equation in the question is now

y = 3 x + 7 . When a line is reflected across the y-axis, the slope of the new
2 2
line is the negative of the original line’s slope, so we want an answer with
3
a slope of – . Eliminate (B), (D), and (E). We also know that when a line
2
is reflected across the y-axis, its y-intercept remains the same, so pick the
7
answer with a y-intercept value of . Only (A) works.
2
13. C Use the graph to find the values you need. If g (4) = d, that means that
x = 4. Look at the graph and find the y value when x = 4. Clearly, when x = 4,
y = 6, so d = 6. Now find g(6) the same way on the graph: when x = 6,
y equals a number between 7 and 8.

182 | © The Princeton Review, Inc.

17 Answers Exp 182 11/16/05 4:14:17 PM


Answers & Explanations

page 79

14. A Use transformation rules. When you add in the parentheses, the graph shifts
that number of units to the left. When you subtract outside of the parentheses,
the graph shifts down that number of units. Only (A) works.

1
18. Start with what you know: The distance from point A to D is 8, and the area
32
of the figure is 128, so the other side of the rectangle must be 128 = 16.
8
Therefore, the value of a is 8, and point D is (4, –8) on the graph. You now

have the coordinates of point D on the graph, so plug in the coordinates for

D into the equation and solve for c. y = –cx4, so –8 = –c(4)4 = –c(256). So, c
1
= .
32

© The Princeton Review, Inc. | 183

17 Answers Exp 183 11/16/05 4:14:19 PM


Refresher Manual for the SAT

CRITICAL READING Homework


pages 103–104

21. Eliminate (A), (B), and (C) because they are probably all too extreme to be best
answers on the SAT.

26. Eliminate (A) and (C) because they are also probably too extreme to be best
answers on the SAT.

page 105

1. “Killer bees” is not an accurate name for Africanized bees.

2. Three men who seemed to agree actually disagree strongly.

3. The weather in Scotland is not pretty.

4. Flying machines’ wings were modeled after birds’ wings.

5. The economy is not good.

pages 106–107

12. E Papua New Guinea has a coast (line 1) and is an island (line 2).

13. C Kula is defined as a system in which male Trobrianders participate (lines


4–5), but we can’t know if women will eventually take part as well.

14. D Lines 1–3.

15. D The author’s conclusion is that pizza is a “multicultural and ... multi-conti-
nental collaboration that has evolved over the centuries” (lines 2–4). If pizza
originates in Greece (line 4), but is further developed in Chicago, these facts
would support his conclusion.

16. B The principle: human habitation has a negative effect on the environment. The
example: the San Joaquin fox’s population drop. The different interpretation:
foxes benefiting from living near humans (line 7).

17. B Put in your own word or phrase: something like “living together situation.”
Only (B) carries this meaning.

18. B This use of language helps the reader “imagine” the perspective of early
peoples.

19. C Put in your own word: something like “predicted.” Only (C) carries this
meaning.

184 | © The Princeton Review, Inc.

17 Answers Exp 184 11/16/05 4:14:19 PM


Answers & Explanations

page 108

6. C You need an answer that suggests the quest for immortality is NOT the most
important theme in Gilgamesh. (C) is correct, because it suggests that the
most important theme is “loss.”

7. B The author of Passage 2 states that we “see for ourselves the meaning of a
story,” so he would probably not agree with the author of Passage 1 that there
is only one important theme.

8. A Both passage mention the age of Gilgamesh: it is referred to as “ancient”


(Passage 1, line 1) and as having been written “four thousand years” ago in
Passage 2, line 19.

9. D This is a great paraphrase of the primary purpose of each passage.

pages 109–110

10. C Lines 1–7.

11. D Lines 16–23.

12. A We need a word that means “realism.” (A) is closest to this meaning.

13. D The footnote and lines 28–32 support this answer.

14. B Lines 38–42.

15. D Lines 38–45.

16. C The quote refers to the paintings, not Giotto, and is neither critical nor con-
fused.

17. B Lines 55–60.

18. C Lines 28–34 and the footnote support this answer.

pages 111–113

7. B We need a word that means “uphold.” (B) is closest to this meaning.

8. D Lines 1–5.

9. C Lines 21–24.

10. E Lines 8–9 and 28–30.

11. A Lines 33–36.

12. D Lines 63–70.

13. B Lines 71–73.

14. C This is not cited as a reason anywhere in the passage.

15. E Lines 56–62.

© The Princeton Review, Inc. | 185

17 Answers Exp 185 11/16/05 4:14:19 PM


Refresher Manual for the SAT

16. B Lines 86–96.

17. E This is the best paraphrase of the meaning of lines 94–95, that there was no
alternative for women at this time.

18. D Lines 96–100.

pages 114–117

1. C We need a word that means “wealth.” (C) is closest to this meaning.

2. C The author is wondering whether environmentalists are truly devoted to the


cause. Only (C) works.

3. D Lines 25–29.

4. C We need a word that means “made worse.” (C) is closest to this meaning.

5. E Lines 30–34.

6. C Lines 47–50.

7. C Lines 61–64.

8. E Pollen is not pollution. The other four answers do suggest that there are costs
connected to pollution that are not health related.

9. D Lines 91–97.

10. B Lines 102–108.

11. D This is the best paraphrase of the two passages. Other answers are either
not stated (A and E), extreme (C), or not true (B).

12. A Passage 1 discusses money, while Passage 2 discusses health.

13. A The author of Passage 2 uses a direct quotation in lines 91 to the end of the
passage. The author of Passage 1 does not do this.

186 | © The Princeton Review, Inc.

17 Answers Exp 186 11/16/05 4:14:19 PM


Answers & Explanations

SENTENCE COMPLETIONS
Be sure to look up and memorize any words that you don’t know!

page 118

1. horrible

1. seriousness, gravity

2. excitement

3. standard

5. rebirth

5. soften, give up

6. shouting

7. influx, addition

7. wasteful

7. death, extinction

7. study, scrutiny

9. penniless, broke

10. required, necessary

10. relentless, persistent

4. C

6. A

7. B

pages 119–120

8. Eliminate (A) and (B)

10. Eliminate (B) and (E)

1. A

2. D

3. C

4. A

5. C

6. C

© The Princeton Review, Inc. | 187

17 Answers Exp 187 11/16/05 4:14:20 PM


Refresher Manual for the SAT

7. D

8. D

9. B

WRITING Homework
pages 154–155

Drill One
1. D Correction: no

2. C Correction: Liz and her

3. E No error

4. B Correction: swipe

5. A Correction: who are

6. E No error

7. C Correction: her

8. C Correction: enjoy

9. A Correction: frequently

10. D Correction: grow

11. C Correction: or

12. C Correction: but also

13. E No error

14. A Correction: had lain

15. C Correction: it was

16. B Correction: they are becoming

17. D Correction: was the most

188 | © The Princeton Review, Inc.

17 Answers Exp 188 11/16/05 4:14:20 PM


Answers & Explanations

pages 156–158

Drill Two
1. D Correction: exist

2. E No error

3. D Correction: reads the local

4. E No error

5. C Correction: drive more slowly

6. E No error

7. D Correction: have been

8. E No error

9. B Correction: such as

10. A Correction: a hardworking and

11. E No error

12. D Correction: that of gasoline-powered

13. A Correction: consider

14. A Correction: should have

15. E No error

16. A Correction: Domestic

17. E No error

18. A Correction: shaken up

19. E No error

20. C Correction: moved

21. B Correction: is essential

22. A Correction: is going

23. E No error

24. D Correction: responsible for planting

25. B Correction: doing so

26. E No error

© The Princeton Review, Inc. | 189

17 Answers Exp 189 11/16/05 4:14:20 PM


Refresher Manual for the SAT

page 159–160

Improving Sentences Drill


1. E The original sentence is passive and incomplete.

2. A The original sentence is correct as written.

3. A The original sentence is correct as written.

4. B The original sentence uses the wrong conjunction.

5. C The original sentence uses the wrong verb tense.

6. E The original sentence is passive.

7. A The original sentence is correct as written.

8. A The original sentence is correct as written.

9. B The original sentence uses the wrong conjunction.

10. C The original sentence uses the wrong idiom.

11. A The original sentence is correct as written.

12. D The original sentence contains a noun agreement error.

13. D The original sentence contains a parallelism error.

190 | © The Princeton Review, Inc.

17 Answers Exp 190 11/16/05 4:14:21 PM


17 Answers Exp 191 11/16/05 4:14:21 PM
17 Answers Exp 192 11/16/05 4:14:21 PM
17 Answers Exp 193 11/16/05 4:14:21 PM
17 Answers Exp 194 11/16/05 4:14:21 PM

You might also like